You are on page 1of 320

Financial Mathematics Lecture Notes

Using the SOA/CAS FM Syllabus and


Problems
Version 2-2

Russell Jay Hendel, Ph.D.,A.S.A, Dept of Math, Towson University


(c) Fall 2015
Table of Contents
0 - Preface (includes, goals, features of the book,
abbreviations, approach to pedagogy)
1 – Calculators
2 - Money Growth
3 – (IRR) – Internal Rate of Return
4 – Time and Dollar Weighted Methods
5 - Periodic Payments
6 – Increasing Decreasing Annuity
7 - Inflation
8 – Miscellaneous Annuities
9 – Amortization
10 – Amortization Problems
11 – Bonds
12 - Reinvestment
13 - Stocks, Margin, Spot, Forward, Term Structure
14 – Callable Bonds
15 – Duration Convexity
16 –Full Immunization
17 – Insurance, Puts, Calls, Forwards
18 – Introduction to Derivatives
19 – Derivatives - Graphing Methods
20 – Call Put Parity
21 –Arbitrage
22 – Swaps
23 – Appendix (two dozen derivatives)
Solutions to Selected Problems
0.1. PREFACE TO VERSION 2-1 i

0.1 Preface to Version 2-1


• This preface has 5 goals. To present:

• I) Important differences from the first version, notably removal of all


typos and sharpening of proofs

• II)The unique features of this book

• III)The good features of this book ( found in other books also)

• IV) The lists of abbreviations used throughout the book

• V) The URLs of lists of free problem resources

0.2 I: Important differences from the first version


• I published the first version of my book in Spring 2012

• My intentions were to go through the book during the semester and


remove all typos

• However I was badly hospitalized in Summer 2012 and did not get to
correct the many typos that semester

• I didn’t fully recover from the anemia I had till Dec 2014

• So I went through the book this semester and published this version
2-1

• I believe this version relatively free from errors with correct worked
out solutions and good verbal proofs

0.3 This version has many new features and im-


provements
• This version

• is free from many annoying typos

• has a complete set of good verbal proofs

• includes a classification of the new updated SOA problems


ii

0.4 Dedicated URL


• I plan to update this book every semester

• At the very least I will update with new problem solutions

• I am attempting to repost this on the CBT website

• However to make sure the lastest version is immediately available with


all improvements

• I am posting this on my own website: http://www.Rashiyomi.com/Math/Math312.pdf

• This URL will always contain the latest version

• I additionally intend to eventually post this on my Towson Website


home page

0.5 II: Unique features of this e-Book


• The following feautures of this book cannot be found in other current
books:

• Consistent emphasis on problems requiring and developing exec-


utive brain function

• Consistent use of the Rule of 6 - Verbal, Algebraic, Graphical,


Calculator, Name/Classification, English Conventions

• Consistently using Modularization to tackle complex problems

• The Database-Join-Method for Derivative Tables

• Qualitative Graphical methods for Derivatives

• Bernoulli’s rule as a heuristic for approximation

• The book is formatted in a bullet fashion makeing use of the


internet rule of F

• These items are each explained in the slides that follows


0.6. II: GOOD FEATURES OF THIS BOOK (FOUND IN OTHER BOOKS)iii

0.6 II: Good features of this book (found in other


books)
• Emphasis on live SOA problems

• Avoiding different interest and pay period using conversions

• Avoidance of Geometric Series in inflation problems

• Use of verbal derivations

• Review of needed Calculator skills

0.7 II: Good organizational things of the book


• This book can be used for a 15 week, 2-day a week course

• 22 Chapters - Each section fits one class day of 75 minutes

• Remaining days can be used for review and tests

0.8 General Learning Features of this book


• We now describe in more details the unique features mentioned above

0.9 Executive function


• Executive function refers to the brain’s ability to solve a problem with
two or more cognitive areas

• I sometimes refer to these problems as Two Step Problems or TSP

• In other words, the hard part of a problem, might be its twoness

• Each separate component might be easy and doable

• Almost all problems in this course are TSP

• Executive function is a critical skill needed by actuaries


iv

0.10 Publication on my views on executive func-


tion
• The following paper of mine outlines my views on executive function
and pedagogy

• A Discipline-Independent Approach to a Higher Cognitive Pedagogy,

• Journal of Systemics, Cybernetics and Informatics,

• Vol 12 Number 5, 2014 pp 16-21

• http://www.iiisci.org/journal/sci/FullText.asp?var=&id=SA135EN14

0.11 Modularization
• Throughout the text we emphasize breaking big problems into smaller
component parts

• For example a reinvestment problem can typicallly be broken into 2-4


timelines

• One timeline for each investment vehicle

• And a summary timeline

• Such an approach facilitates solving hard problems

• Modularization is consistent with the principle of executive function


and TSP

0.12 Rule of 6
• Every concept in the course should be approached from 6 points of
view

• Its Name and classification

• Its Graph or Time Diagram

• Its Formulae and actuarial symbols

• The verbal-algebraic correspondences needed to understand the


verbal problems associated with this concept
0.13. USE OF VERBAL DERIVATIONS v

• The Calculator keystroke solutions associated with the problem

• The English conventions associated with this problem

• Some problems are difficult because students are looking for the for-
mula

• When instead the hard part of the problem are the English conventions
or its classification

0.13 Use of verbal derivations


• We derive many important formulae verbally

• The verbal technique is common in the mortality portions of actuarial


math

• But not frequently found in the interest theory books

• Formulae for annuities can elegantly be derived verbally

• Without the need for algebraic formulae or geometric series

0.14 Section Length


• The book has 22 chapters

• It was used in a two-day a week, 15-week semester

• We comfortably covered one chapter every 105 minute class (4 credit


course)

• We frequently had time for in-class quizzes

• We also had time to verify computations and ask questions

0.15 Bulleted formatting - Web Rule of F


• This book is not written in prose; rather it is written in a bulleted
fashion

• Such a bulleted fashion is easier on the reader and aids retention


vi

• Studies have shown that when people read web pages they do not read
it as prose

• Instead their eyeball movements trace an F

• They read down the page skimming the first few words of each line

• Thus their eye movement traces out an F; hr

• This book is formatted to reflect this natural reading tendency

0.16 Specific Learning Features of this book - con-


tent specific items
• We next describe features of this book addressing actuarial content

0.17 Emphasis on SOA Problems


• In some books problems are too easy, for example, plug ins

• In other books, some problems are project-like; too hard

• This book draws problems exclusively from the SOA exams

• However, not all problems are accepted

• There is an emphasis on problems with multiple parts that must be


integrated

• Problems are challenging but not overwhelming

• This corresponds to the emphasis on executive function

• Additionally, this book gives a classification of SOA problems by topics

• The SOA online exams are built off these problems

• Examples of lists of SOA problems organized by topic are found in


slides 4.16, 5.32, 9.19
0.18. CALCULATOR - REVIEW OF NEEDED SKILLS vii

0.18 Calculator - Review of needed skills


• Several books have review of needed calculator skills

• We have a short chapter going over basics

• Importantly, we emphasize several methods of doing complex but fre-


quently seen arithmetic functions

• We emphasize only 3 calculator worksheets: Time Value line, Time


Value, IRR, and Amortization

0.19 Use of conversions when interest and pay pe-


riod are different
• Traditional interest theory knows of monthly payments

• One approach to these problems is to convert the annual rate to a


monthly rate

• I refer to this approach as the conversion method

• It unifies all problems

• It obviates the need to become familiar with special monthly symbols

• The special symbols are however defined but their formulae are not
necessary to solve verbal problems

0.20 Avoidance of Geometric Series


• We cover all course material without ever mentioning geometric series

• For example inflation annuities can be converted to level annuities-due


at the adjusted inflation rate

• This is pedagogically superior for the student

• The student need not learn complicated formulae

• Instead the student learns to manipulate timelines in a skillful manner


viii

0.21 Qualitative Graphical Methods for Deriva-


tives
• Many derivative problems can be solved without profit tables

• A general consideration of slope (up and down) and behavior at infinity


is sufficient

• We provide a new semi-mechanical method to approach these problems

• This method enables quick solution of problems that otherwise might


require profit tables

• This method is similar to use of qualitative methods for differential


equations

0.22 Database-Join-Method for Derivatives


• All derivative problems can be broken into 3 building blocks

• Call(Short/long) puts (short/long) and buys(short/long/forwards)

• The building blocks can then mechanically be unified for a final table

• The method of doing this can eleganntly be formulated using Database


joins

• The benefits of this are similar to the benefits of modularization

0.23 III: Abbreviations - Past Exams


• Throughout this book we use abbrevitations

• In the following slides we present these abbreviations

• The abbreviations refer to items on past exams or the Sample questions

• I recommend downloading both questions and solutions

• Past exams may be obtained at


0.24. ABBEVIATIONS ix

• http://www.soa.org/education/exam-req/syllabus-study-materials/edu-
multiple-choice-exam.aspx

• Look for FM or Course 2.

0.24 Abbeviations
• e.g. M01#3 means Question 3 on the May 2001 exam

• Similarly N05#15 means Question 15 on the November 2005 exam

0.25 IV: URLs - Resources for free downloads


• In addition to past exams

• The SOA provides Sample Problems

• They are used throughout the book

• We recommend you download them also.

• Here are the urls and corresponding abbeviations

• Q-IT: http://www.soa.org/files/edu/2015/edu-2015-exam-fm-ques-theory.pdf

• S-IT: http://www.soa.org/files/edu/2015/edu-2015-exam-fm-sol-theory.pdf

• D-SQ: http://www.soa.org/files/edu/edu-2014-10-exam-fm-ques.pdf

• D-SA: http://www.soa.org/files/edu/edu-2014-10-exam-fm-sol.pdf
x
Chapter 1

Calculators

1.1 Chapter Overview


• I) Calculator Importance

• II) Calculator Notation

• III) A) Formatting

• IV) B) Order of Operations (Review)

• V) C) 3 Methods for negative exponents

• VI) D) 3 Methods for Calculations

• VII) Da) Traditional (Parenthesis) Calculation Method

• VIII) Db) Memory Calculation method

• IX) Dc) Operation-order calculation method

• X) D) Mentally checking

• XI) E) Timeline - Compound interest

• XII) E) Time Value Worksheet

• XIII) E) Important notation

• XIV) Advanced memory (Optional)

1
2 CHAPTER 1. CALCULATORS

1.2 I) Calculator Importance


• Course goals include both

• Mastery of mathematical content

• Mastery of calculator techniques

1.3 Why both: Content and Calculators


• Each decade has different technologies available

• E.g. Tables, calculators, special calculators

• Hence, challenging mathematical content is relative to calculator knowl-


edge

• If a problem cannot be done by pushing a button it may be


challenging

• But if that same problem can now be done by pushing a button


it is no longer challenging

1.4 Approved Calculators


• BA II Plus is preferred

• BA II Plus Professional allowed, but not necessary

• On SOA and CAS exams they allow: BA 35, TI-30XA, TI-30X II,
TI-30X

• To check the latest list visit: https://www.soa.org/education/exam-


req/exam-day-info/edu-calculators.aspx

• Multiple calculators are allowed per student on each exam

• No other calculators will be allowed in this course.


1.5. MINIMUM CALCULATOR KNOWLEDGE 3

1.5 Minimum Calculator Knowledge


• Do we have to be calculator experts?

• If not, what is minimum we have to know?

• This is the approach adopted here.

1.6 Overview - 5 Things to know


• A) Formatting

• B) Order of operations

• C) Three methods for negative exponents

• D) Three approaches for calculation: i) Natural-Parenthesis, ii) Mem-


ory, iii) Order of Operations

• E) TV (Time value worksheet)


item The above five items - A - E - will be used throughout the course

• Two other special calculator items will be covered later when dis-
cussing their specific content

• F) IRR, Internal Rate of Return calculation and NPV, Net Present


Value

• G) Amortization Worksheet

1.7 II) Calculator Notation


• Calculator has 9 rows and 5 columns

• In teaching calcualtors I use my own x-y notation

• x refers to row and y refers to column

• For example, 1 refers to top row; -1 refers to bottom row

• A 2 prior to coordinates indicates a requirement to press the 2nd key


first
4 CHAPTER 1. CALCULATORS

1.8 Calculator Notation Examples


• Examples in this section follow the BA II Plus/Professional Keyboard
• Those with other keyboards should adjust their notes
• Some important buttons are: on-off, clear, 2nd function key, ex-
ponential, logarithm, power
• You should know where the buttons for each of these functions is
• On-off at (1,5); clear at (-1,1)
• 2nd Function key at (2,1);
• So (-4,1) gives logarithm; 2(-4,1) gives exponential function
(5,4) gives powers

1.9 III) A) Formatting


• Goal: Readability, rounding, accuracy (Discussion: Why?)
• Wrong: 1/3 = .33
• Wrong: 1/3=.33333333333
• Preferred: 1/3 = 0.3333
• A course requirement on exams and homework for a perfect score is
4-place accuracy.

1.10 Setting Format


• To Enter Format window: 2(-1,3)
• To Navigate with Control Arrows: (1,3), (1,4)
• Use arrows until DEC appears
• Hit keys: 4 ENTER (1,2)
• This sets all computations to 4 decimal places.
• To leave formatting window: QUIT 2(1,1)
1.11. IV) B) ORDER OF OPERATIONS - (REVIEW) 5

1.11 IV) B) Order of Operations - (Review)


• Goal: Make calculator usage consistent with the way we think

• Example: Consider 2 + 3 × 4.

• Is the answer 14 or 20.

1.12 Order of Operations - Two Methods


• There are two methods to compute 2 + 3 × 4.

• Method #1: Addition 1st. 5 × 4 = 20

• Method #2: Multiplication 1st. 2 + 12 = 14

1.13 Order of Operations - 2 Methods


• CHN: Chain method refers to sequential operations

• The operations are sequentially performed from left to right

• In the above example, the Chain method does addition 1st, multipli-
cation 2nd

• AOS: Algebraic Order of Operations is the way humans compute

• We discuss the principles of AOS in the next slide

1.14 Theory of Algebraic Order of Operations


• AOS is Based on 5 principles

• 1) Do Exponentation 1st

• 2) Do Multiplication/Divisions 2nd

• 3) Do Additions / Subtractions 3rd

• 4a) If there are many additions and multiplications compute sequen-


tially left to right

• 4b) If there are many exponentiations compute sequentially right to


left
6 CHAPTER 1. CALCULATORS

• 5) One can override the above rules with parenthesis

• Perform the items in parenthesis before applying rules 1-4

• Example: 2 + 3 × 4 AOS requires

• multiplication 1st

• addition 2nd

• (2 + 12 = 14)

1.15 To Format calculator for Order of Operations


• Enter FORMAT, 2(-1,3)

• Use ARROWS, (1,3) (1,4) until CHN,AOS appears

• Toggle CHN, AOS with SET, 2(1,2)

• Select AOS by stopping when AOS appears in window

• Quit, 2(1,1)

• Always test to make sure ok

• Hit keys 2 + 3 × 4 = The window should display 14.

1.16 V) C) 3 Methods for negative exponents


• Example: Calculate 1
1.013
= 1.01−3

• We present three methods

• Please master this now as it will facilitate following course examples


1.17. THE THREE METHODS - A,B,C 7

1.17 The three methods - a,b,c


• a) 1.01yx 3± =
• The keystrokes are 1.01 (5,4) (-2,4) (-1,4) (-1,5)
• yx is found at (5,4)
• b) 1.01yx 3=; 1/x
• 1/x is found at (4,4)
• Note: You do not have to enter = after 1/x
• c) 1/1.01yx 3=
• Note the different locations of = in methods (a) and (b)

1.18 VI) D) 3 Methods for Calculations


• There are 3 primary calculation methods
• a) Natural-Parenthesis. This method is a frequent source of error
on calculators
• b) Memory. This is an alternative to Parenthesis
• c) Order of Operations This is another alternative to Parenthesis
• Best practice is to select a method and develop habits now

1.19 Illustrative Example


1−1.01−3
• Example: 0.01

• Answer: 2.9410

1.20 VII) Da) Traditional (Parenthesis) Calcula-


tion Method
• Correct: (1 − 1.01yx 3±)/.01=
• Common Parenthesis error. 1 − 1.01yx 3±/.01= − 96.0590
8 CHAPTER 1. CALCULATORS

1.21 (VIII) Db) Memory Calculation method


• The key to the Memory method is use of the STO, RCL keys (store,
recall )

• STO (-3,1); RCL (-2,1)

• 1 − 1.01yx 3± = STO7

• 0.01 = STO 4

• Comment: You can see the 0.01 and be sure no error

• RCL 7 / RCL 4 =

• Comment: Gives correct answer of 2.9410

1.22 Memory vs. writing down


• Some students use this method but do not use the STO keys

• Instead these student write down the numerator and denominator

• Writing down will not hurt in this example

• Writing down will change the accuracy in more complicated examples

• Why? Because the calculator stores more places than it displays

• So using memory vs. jotting down will increase accuracy

• Make a habit of using STO and RCL

1.23 Advanced Memory-Optional


• See the last section, section XIV, in this chapter for advanced memory
methods
1.24. IX) DC) OPERATION-ORDER CALCULATION METHOD 9

1.24 IX) Dc) Operation-order calculation method


• To use the operation-order calculation method one has to enumerate
operations
1−1.01−3
• The example 0.01 uses 3 operations . . .

• i) The exponentiation

• ii) The subtraction

• iii) The division

1.25 Order of Operations


• The order of operations is as listed in the last slide

• The division is done last because the fraction symbol is like parenthesis

1.26 Order of Operations - Illustration


• After performing an analysis of order of operations, we sequentially
perform them

• 1.01yx 3 ± = is done first

• We next perform minus operation

• But how do we perform it if the 0.9706 which we want to subtract is


in the display window?

• We use the ± key

• ±+1=

• Division is done 3rd:

• / .01=
10 CHAPTER 1. CALCULATORS

1.27 X) Mentally Checking


• Mentally checking is not part of the syllabus

• Nevertheless, mentally checking is a very important part of becoming


a good actuary.

• Why check?

• Good actuaries always mentally approximate and check

• Use of Parenthesis can cause errors because you have no feel

• Bernoulli’s law is a key tool

• Bernoulli’s law is a required part of the syllabus for one course topic

1.28 Bernoulli’s Law


• Theorem: For small values of x, y,(1 + y)x ≈ 1 + yx

• Proof is by the binomial expansion


x(x−1) 2
• (1 + y)x = 1 + xy + 2 y ...

• If x, y are small the main term is 1 + xy

1.29 Examples of Bernoulli’s Law


• Example 1. 1.015 ≈ 1.05
√ 1
1
• Example 2. .9 = (1 − .1) 2 ≈ 1 − 2 × 0.1 = 0.95.

1.30 Approximating in a Complicated Example


• How would we check the following
1−1.01−3
• Example: .01 = 2.9410.

• Step #1) Calculate 1.01−3

• #1 continued) Solution: 1.01−3 ≈ 1 − 3 × 0.01 = 0.97

• Step #2) Calculate numerator: 1 − .97 = .03


1.31. XI) TIMELINE - COMPOUND INTEREST 11

.03
• Step #3) Calculate denominator: .01 =3

• Conclusion: We did pretty well! 2.9410 ≈ 3

1.31 XI) Timeline - Compound Interest


• The Timeline is a powerful geometric aid

• It is the single most used aid in financial problems

• It is used for compound interest problems

• The basic setup is given in this section

• Then we show algebraic, numerical and calculator worksheet correlates

• So you must know 4 approaches: timeline, algebra, calculator, numeric

1.32 Basic Timeline Example


• Sample question: How much is 1,000 worth in 2 years in a 10 percent
account?

• Before proceeding we explain the phrase 10 percent account

• Suppose you deposit $1,000 in a bank

• The bank will pay you interest on the $1,000

• Suppose the bank agrees to give you 10 percent of your account value
each year

• Then at the end of the year the bank pays 10% × $1, 000 = $100 in
your account

• This $100 is called interest

• Your total accumulated value in your account is 1000+100 = 1100.

• Now that we understand this basic setup let us look more deeply
12 CHAPTER 1. CALCULATORS

1.33 $1000 deposit and 10% per year


• Year 1. 1000 + 10%1000 = 1.1 × 1000 = 1100

• Year 2. 1100 + 10%1100 = 1.1 × 1100 = 1210

• Note 1.1 × 1100 = 1.1 × 1.1 × 1000 = 1.12 × 1000

• A similar calculation would apply if one left the money for a 3rd year

• At the end of the 3rd year you would have 1.1 × (1.12 × 1000) =
1.13 × 1000.

• This process can be generalized for an arbitrary number of years, t


years.

1.34 Formula - Compound Interest



A(t) = A(0)(1 + i)t (1.1)

• A(t) is the Accumulation function, the amount accumulated at time t

• i is the interest rate per period which is assumed compounded

• The interest is compounded - that is,

• It is reinvested in the account and earns interest

• t is the number of years the money lies in for example the bank

• We will use either t or n to represent time.

1.35 Formula-Compound Interest-Example


• A(1) = 1000(1 + .1)1

• A(2) = A(1)(1 + .1)1 = 1000(1 + .1)2


1.36. THE GEOMETRIC DIAGRAM 13

1.36 The Geometric Diagram


• The following diagram summarizes all calculations

• Timeline

1000 1000(1.1) = 1100 1000(1.1)2 = 1210


0 1 2

• In this timeline one row indicates points of time: t = 1, 2

• The other row indicates the balance in the account at that time

• So for example the account has 1000 at time 0 and 1100 at time 1.

1.37 XII Time Value Worksheet


• The Time Value Worksheet is found on Row 3 of your calculator

• This is the most used calculator worksheet

• I will use the abbreviation TV

• We gave the geometric setup above

• In this section we present algebraic solutions

• We also give the keystrokes using the TV calculator worksheet

1.38 Four Solvable Problems - Algebraic Approach


• All four problems come from one basic situation

• Accumulating $1210 by investing $1000 for 2 years at rate i = 10%

• Here are the four problems

• Find A(2) : Solution: A(0)(1 + i)t = 1000(1.1)2


A(2) 1210
• Find A(0) : Solution: (1+i)2
= 1.21
14 CHAPTER 1. CALCULATORS

 1
2
A(2) A(2)
• Find i : Solution: A(0) = (1 + i)t ; 1 +i= A(0)

 
A(t) A(t)
• Find t : Solution: A(0) = (1 + i)t ; t log(1 + i) = log A(0)

1.39 TV Timeline
• Basic idea. Enter any 3 of 4 numbers; then compute 4th number.

• Find A(0)

• Solution.

2 10 CPT 0 1210
N I/Y PV PMT FV

• Just to be clear: The 2 over the N means entering the following


keystrokes:

• 2 (3,1)

• Similarly the keystrokes 10 (3,2) enter 10% for interest

• Notice that we enter 10 not 0.10

• Hitting CPT(1,1) PV(3,3) yields an answer of -1000.

1.40 TV Signs +, −
• How do you determine +, −.

• Think of me as the bank and you as the lender.

• At t = 0 you give me, the bank, 1000, and hence, you lose, 1000.

• Hence you enter -1000

• But at t = 2 you can withdraw 1210. That is you gain 1210.

• Hence calculator shows +1210.

• You could also reverse the roles of + and -


1.41. TV KEYSTROKES - FURTHER EXAMPLES 15

1.41 TV Keystrokes - Further examples


• Find i
• Solution.
2 CPT − 1000 0 1210
N I/Y PV PMT FV

• Find t
• Solution
CPT 10 − 1000 0 1210
N I/Y PV PMT FV

1.42 Checking data entries


• Suppose I wanted to check what I entered
• You use the RCL button
• For example RCL N places 2 in the display window since N = 2 years
• It is always good to check your entries.

1.43 Summary
• TV allows you to press 3 buttons to solve for a 4th unknown amount
• TV is faster than algebra!
• Using the TV worksheet is a basic skill
• The Timeline is a basic geometric aid

1.44 Advanced Memory - Viewing Memory


• MEMORY 2(-1,2)
• Use ARROWS (1,3) (1,4)
• You can see values in all 10 memory locations
16 CHAPTER 1. CALCULATORS

1.45 XIV) Advanced memory (Optional)


• Three types of clearing

• (1) CLEARWORK 2(-1,1) Clears all memory cells

• (2) But CLEAR (-1,1) Clears a typo during entry

• For example, if I want to calculate 2 + 3 × 4 + 5

• Suppose I have typed 2 + 3 × 1

• Then I can either start over or

• I can continue 2 + 3 × 1 CLEAR 4 + 5;

• Use CLEAR when you want to correct a typo and not start over

• (3) On-off Clears typos/starts over, but does not clear memory cells!
Chapter 2

Money Growth

2.1 Chapter Overview


• We have 13 sub-goals to meet in this chapter

• I) Definitions, Bank Background and terminology

• II) 7 Interest methods: What remains the same? What changes?

• III) Method A: Compound Interest

• IV) Method B: Nominal Compound Interest

• V) Methods C, D: (Nominal) Compound Discount method

• VI) Methods E, F: Simple interest / discount

• VIII) Method G: Force

• IX) Actuarial Equivalence Principle

• X) Discount factor, v

• XI) Good SOA Exam Problems

• XII) Rule of 6

• XIII) A typical comparison problem and solution approach

17
18 CHAPTER 2. MONEY GROWTH

2.2 I) Definitins, Bank Background and terminol-


ogy
• Principle A(0) : You go into a bank and deposit $1,000

• Interest rate i : The bank agrees to pay you 10% per year for your
money

• If you want to be more specific, is indicates the interest rate from


t = s − 1 to t = s.

• The s-th year: The s-th year is from t = s − 1 to t = s.

• For example, the first year is from 0 to 1. There is no zero-th year.

• Interest Period: The interest period - the time interval to which the
10% applies - is the year

• Payment Period: The bank will pay you the interest at the end of
each year.

• The time duration at which payment is made is called the payment


period.

• Note that in our example, the interest period and payment period are
the same.

• Interest amount I : The amount that the bank pays you - I =


$1000 × 10% = $100

• If I want to be more specific, I can use Is for the amount of interest


earned from time t = s − 1 to time t = s.


A(s) − A(s − 1) = Is = is × A(s − 1) (2.1)

• More generally, the amount of interest earned from time s to time t is


indicated by Is,t , with

A(t) − A(s) = Is,t (2.2)

• Accumulated amount at time t, A(t) : For example, at time t = 1


you have accumulated A(1) = $1, 100.
2.2. I) DEFINITINS, BANK BACKGROUND AND TERMINOLOGY 19

• Besides the capital-A accumulation function, A(t) there is a little-a


accumulation function, a(t).
A(t)
• a(t) is defined by a(t) = A(0) .

• You can think of a(t) as A(t) for an initial deposit of 1 (Why?)

• I personally never use a(t)

• Time Value of Money: Notice that the $1,000 at time t = 0, without


further deposits, becomes the $1,100 at time t = 1.

• We say that the value of money is a function of time.

• In other words, without further deposists money has two different


values at two different times

• We next present two different types of interest

• Simple interest: When you withdraw the interest from the bank and
leave the principle

• In such a case you get $100 each year.

• Compound interest: When you leave the interest in the bank

• In such a case you earn interest on the interest

• Example: If you deposit $1,000 in a 10% account then . . .

• You earn $1, 000 × 10% = $100 the first year

• You earn $1, 100 × 10% = $110 the second year.

• The compound interest rate is also called the effective annual rate

• The compound method of accumulating interest is the default method

• This means if you are just told say that i = 5% then . . .

• you are expected to understand that 5% is a compounded rate

• Present Value (PV), Accumulated Value(AV), Future Value(FV),


Current Value(CV):

• We use the English phrase Present Value to refer to A(0) = $1, 000
20 CHAPTER 2. MONEY GROWTH

• We use the English phrase Accumulated value at t or Future value at


t to refer to A(t).

• So A present value of $1,000 accumulates to a future value of $1,210


at time t = 2.

• Also, the current value at t = 1 is 1100

• Interest factor: You can refer to interest in three ways:

• The interest rate is i, the interest amount is I, the interest factor is


1+i

• In some problems, it is easier to compute interest factors vs. interest


rates

• In those problems, you would formulate everything in terms of factors.

2.3 II) 7 Interest methods: What remains the same?


What changes?
• There are seven methods to describe interest or money growth

• In every interest method three things remain the same and two things
change

• #1) In all methods, t refers to time. Both t and n refer to time.

• #2) In all methods A(t) is the amount accumulated in the bank at


time t

• #3 In all methods Is,t = A(t) − A(s), the amount of interest accumu-


lated from s to t.

• The remaining two items change from method to method

• #4) In all methods there is some symbol describing the rate of growh.

• Some typical symbols are i for interest, d for discount and δ for force.

• There are in fact 5 symbols


2.4. 7 INTEREST METHODS 21

• #5) In each problem, the algebraic formula relating A(t) with A(0), t
and i, d, or δ changes.

• To understand a method means to know i) the notation and ii) formula


relating A(0), A(n)

2.4 7 Interest methods


• In this chapter, we will explore the following 7 methods:

• compound interest, compound discount, simple interest, sim-


ple discount, nominal interest, nominal discount, force.

• To recap what was stated in the last slide

• For each method

• There will be separate symbols indicating money growth

• Separate formulae for A(t) in terms of t, A(0)

2.5 III) Method A: Compound Interest


• This is a review of Chapter 1.

• Basic question: Suppose I deposit P = A(n) at time t = n, at


interest rate i,

• How much will I have at time t = n + 1?

• Basic Solution: I have A(n + 1) = P + i × P = (1 + i)P.

• Recall we call 1 + i the interest factor

• If we repeatedly apply the interest factor starting at t = 0 we obtain,

A(n) = A(0)(1 + i)n . (2.3)

• Remember: If nothing is said about i then it is assumed compound.


22 CHAPTER 2. MONEY GROWTH

2.6 IV) Method B: Nominal Compound Interest


• Suppose the bank pays 10% per year payable twice a year.
• By convention, this means that you earn 5% each six months
• We call this 10% interest in name or nominal interest
• Why is it called interest in name?
• Because you never use 10% in any calculations. Rather you use 5%.
• So the 10% is in name only.
• So A(0) = $1, 000, A(0.5) = 1.05 × $1, 000 = $1, 050, . . .
• . . . A(1) = 1.05 × $1, 050 = $1, 102.50, etc.
• We use the symbol i(2) = 10% to indicate nominal interest payable
twice a year.

i(m)
rate of i(m) payable m times a year ←→ interest rate of every m-th of a year
m
(2.4)
• So the accumulation formula becomes
i(m) mn
 
A(n) = A(0) 1 + (2.5)
m

• The following all mean the same thing:


• payable twice a year, nominal rate payable twice a year, compounded
twice a year, convertible twice a year, i(2) .

2.7 V) Methods C, D: (Nominal) Compound Dis-


count method
• Timeline.
A(0) A(1)
0 1
2.8. FORMULA FOR A(N ) 23


I = A(1) − A(0), the interest amount (2.6)


I
i= , the interest rate (2.7)
A(0)


I
d= , the discount rate (2.8)
A(1)

• Discount is payment in advance; common with Bonds (Discussion)

• Interest and discount are two ways to describe the same situation

2.8 Formula for A(n)


A(0)
• A(1) = A(0) + I = A(0) + dA(1) −→ A(1) = 1−d

A(1) A(0)
• A(2) = 1−d = (1−d)2

.
• ..


A(0)
A(n) = (2.9)
(1 − d)n

2.9 Nominal Discount


• This is analogous to nominal compound

• We suppose d(p) is the nominal rate of discount payable p times a year.


A(0)
A(n) = (2.10)
d(p) pn
(1 − p )
24 CHAPTER 2. MONEY GROWTH

2.10 VI) Methods E, F: Simple interest / discount


• Here is a basic question:

• I deposit A(0) dollars at time t = 0 at simple interest rate i

• Interest does not stay in bank (Example: retirement fund)

• What is A(n)? How much total money will I have accrued at time
t = n.

2.11 Simple Interest and Discount - Basic Answer/Formula


• Simple interest

A(0) A(1) = A(0)(1 + i) ... A(n) = A(0)(1 + in)


0 1 ... n

• Simple discount
A(0) A(0)
A(0) A(1) = ... A(n) =
1−d 1 − dn
0 1 ... n

2.12 Formulae Summary



A(n) = A(0)(1 + in) (2.11)


A(0)
A(n) = (2.12)
(1 − dn)

2.13 Compound vs. Simple Interest / Discount


• Compound: Interest rate, i, is constant each year

• Simple: Interest amount, I, is constant each year

• The default, if nothing else is said, is compound and interest


2.14. VIII) METHOD G: FORCE 25

2.14 VIII) Method G: Force


• Here is motivation for the concept of force, instantaneous rate of in-
terest

• We use the symbol δt to denote instantaneous rate of change

• We use the limit definition of instantaneous rate of change presented


in calculus courses.


mIt,t+ 1 
A(t + 1
− A(t)
 1
− A(t) A0 (t)
m m) 1 A(t + m)
δt ≈ =m = 1 −→
A(t) A(t) A(t) m
A(t)
(2.13)
1 i(m)
• But A(t + m) − A(t) = m A(t)

• Putting the last two equations together we obtain

δt ≈ i(m) (2.14)

• Almost identical reasoning can be applied to compound discount yield-


ing the sister approximation

δt ≈ d(p) (2.15)

• As an exercise one should check the following identity, using the chain
rule, (??)
d A0 (t)
log A(t) = = δt (2.16)
dt A(t)

2.15 Force - Formulae


• Integrate both sides of (??) from Beginning time, B, to End time, E

A(E) RE
= e B δt dt (2.17)
A(B)

• But suppose δt = δ a constant in the last equation


A(t) = A(0)eδt = A(0)(1 + i)t (2.18)
26 CHAPTER 2. MONEY GROWTH

• If A(0) = 1 and t = 1 in the last equation we obtain


eδ = 1 + i (2.19)

2.16 Force Summary


• Force Timeline

A(B) δt A(E)
B E

• δt is force or instantaneous rate of interest at time t

• All you need to know are the seven formulae in the last two slides
(which also contain perspectives)

• Force was useful before the advent of the calculator (Discussion)

2.17 IX) Actuarial Equivalence Principle


• Timeline.

A(0) A(n)
0 n

• Two transactions are (actuarially) equivalent if A(0) and A(n) are


equal.

2.18 Equivalence example clarified


• The following two are equivalent

• A deposit at time t = 0 of $1,000, at an annual compound rate of 10%

• A withdrawal at time t = 1, of $1,100 at an annual compound discount


rate of 9.09%
2.19. BASIC I − D FORMULAE 27

• Why are they equivalent? In both cases I = $100, A(0) = 1000, A(1) =
1100
100 100
• But 1000 = 10% while 1100 = 9.09%
• The same timeline has two different but equivalent descriptions.
• So Actuarial Equivalence is a way of saying that two different descrip-
tions are the same

2.19 Basic i − d formulae


• If I deposit $1 for n years, payable m times a year, (discounted p times
a year), how much do I have at t = n?
 m  −p
i(m) 1 d(p)
• 1+ m = 1 + i = 1−d = 1 − p

• The Basic Equivalence.


i(m) mn d(p) −pn
   
A(n) = A(0) 1 + = A(0) 1 − = A(n) (2.20)
m p

• Warning: The Discount formula has two minuses - in exponent and


in parenthesis
• i(m) is called the nominal rate of interest payable or compounded m
times a year
• d(p) is called the nominal rate of discount payable or compounded p
times a year
• In any problem you may want A(0), A(n), i, d, or n.
• You simply use the basic formula relating A(0), A(n).

2.20 X) Discount factor, v


• Timeline.
v 1
0 1
28 CHAPTER 2. MONEY GROWTH

• v is the present value of 1 at 1

1
• Solving, we obtain, v = 1+i (Why?)

• 2 year timeline.

v2 v1 1
0 1 2

• n years

vn 1
0 n


If A(0) = v n X −→ A(n) = X. (2.21)

2.21 v, The Discount Factor - Meaning


• v n is the present value of 1 at n.

• So 1 at time t = n is equivalent to v n at time t = 0

• So I need v n dollars at time t = 0 to obtain 1 dollar at time t = n.

• In other words, the cost at time t = 0 for a payment of 1 at time t = n


is v n .

2.22 Discount factor vs Discount method


• Discount factor refers to v

• Discount method refers to calculating A(n) using d

• Although the same word is used, the meanings are totally different
2.23. USEFUL FORMULAE FOR V, D I AND D 29

2.23 Useful Formulae for v, d i and d



I iA(0) i
d= = = = iv (2.22)
A(1) (1 + i)A(0) 1+i

i 1
1−d=1− = =v (2.23)
1+i 1+i

2.24 XI) Good SOA Exam Problems


• Most texts explain theory well

• If you only apply theory to simple problems you are not learning, not
practicing enough

• I consider a problem in this course good if it has complex organizational


quality

• Such problems are higher cognitive since they require executive brain
function

• Example 1: Problems with 2 or more subproblems are good

• Some typical examples are i) parallel subproblems, ii) sequential sub-


problems, iii) comparisons

• Example 2: Problems with one subproblem with many algebraic


conversions required

• We will explore problems requiring conversions in future chapters

2.25 Good Problem sources


• We shall use live problems from the public SOA/CAS exams as a
source of problems

• There are several public URLs where these problem sources may be
freely downloaded

• The URLs and abbreviations we will be using throughout the book


are as follows:
30 CHAPTER 2. MONEY GROWTH

• Q-IT https://www.soa.org/Files/Edu/2015/edu-2015-Exam-FM-ques-
theory.pdf

• S-IT https://www.soa.org/Files/Edu/2015/edu-2015-Exam-FM-sol-theory.pdf

• D-SQ https://www.soa.org/Files/Edu/edu-2014-10-Exam-FM-ques.pdf

• D-SA https://www.soa.org/Files/Edu/edu-2014-10-Exam-FM-sol.pdf

• Example 1: Q-IT#5 refers to Question #5 in the Questions for


Interest Theory

• Example 2: M05#3 is Question #3 on the May 2005 exam

• We will also use the 7 published SOA/CAS exams

• You can find these exams at URL: http://www.soa.org/education/exam-


req/syllabus-study-materials/edu-multiple-choice-exam.aspx

• The interest theory exams are sometimes called Course 2 and some-
times Course FM

2.26 Problems vs. Theory


• Chapters 1 and 2 presented the theory of 7 types of interest:

• Simple interest-discount, compound interest-discount, nominal interest-


discount, (constant) force

• From a theory point of view, each of these 7 types are equally on the
syllabus

• From an exam problems point of view however, 50% of SOA problems


involve force

• From an exam problems point of view,compound and simple interest


and discount problems are too easy

• Exam problem proportions may indicate different emphasis than the-


ory
2.27. GOOD SOA/CAS EXAM PROBLEMS ON THE SEVEN INTEREST METHODS31

2.27 Good SOA/CAS Exam Problems on The Seven


Interest Methods
• N00#53 - Force - compound (Comparison / Algebra)
• M01#45 - Force - nominal discount (Good comparison problem)
• M03#1 - Force - nominal (Good comparison problem)
• M01#49 -Force-nominal-simple (Comparison problem/Abstract)
• M00#37 - Force-nominal - Piecewise functions
• N01#1 - Force - Interest Amount (Algebra)
• M03#12 - Nominal - Simple Interest - Interest Amount
• Q-IT#1 - Same as M03#1
• Q-IT#21 Same as N01#28
• Q-IT#3 Same as M03#12
• N01#28 - Payment - Deposit are continuous function of t
• Q-IT#61 - Force - Substitution
• Q-IT#77 - Nominal-Nominal (Good conversion problem)
• Q-IT#79 - Force-Compound

2.28 XII) Rule of 6


• The Rule of 6 is an approach to learning and teaching
• The Rule of 6 applies to the entire term and all concepts and examples
in the course
• The concept of Rule of X was first articulated by Prof. Hughes-Hallet
• She made many advances in Calculus pedagogy
• She formulated the rule of 4
• Each course example is approached using i) formalism (algebra), ii)
graphs, iii) numerics iv)verbal
32 CHAPTER 2. MONEY GROWTH

2.29 Rule of 4 in Theory of Interest


• In interest theory, we will use algebraic formulas for formalism (Func-
tions are used in calculus)

• In interest theory, we will use timelines instead of graphs

• In interest theory, we will use calculator timelines instead of numerics

• In interest theory, we use verbal-algebraic correspondences

• We also introduce two more rules: verbal conventions and classifica-


tions/naming

2.30 Rule of 6
• In Interest theory each course concept and example is approached in
6 ways .

• Formula, Verbal, Calculator, Timeline, Name (Classification),


Verbal conventions

2.31 Example of Rule of 6


• Let us illustrate the rule of 6 using compound interest.

• Name/Classification: compound interest

• So we use the notations, verbal conventions and formulae unique to


interest theory

• Verbal-Algebraic Dictionary The idea of verbal is that each En-


glish phrase has a specific algebraic correlate

• We will indicate these correlates with parenthesis using equal signs

• How much (= A(t)) is an initial 1000 deposit (= A(0)) worth in 2


(t = 2) years in a 5% (= i) account

• Verbal Conventions: 5% account means (by convention) compound


interest

• Formula: A(n) = A(0)(1 + i)n


2.32. XIII) A TYPICAL COMPARISON PROBLEM AND SOLUTION APPROACH 33

• Timeline:

A(0) i A(n)
0 n

• Calculator:

n 100i A(0) 0 A(n)


N I/Y PV PMT FV

2.32 XIII) A typical comparison problem and so-


lution approach
• We solve M01#45 in this section

• First lets discuss the general approach to solving problems.

• There are six main components to all problem solutions

• #1) First break the problem into basic sub-problems

• #2) Then each subproblem is studied from three perspectives:

• i) Timelines, ii) Calculator TV Timelines and iii) formulae (of actuarial


equivalence)

• #3) You may have to convert the problem to an algebraically simpler


form.

• Conversions are a very important step in solving complex problems

• #4) Typically there is a link between the various subproblems giving


an equation whose solution solves the original problem; this equation
is called the Equation of Value or EOV

• A typical problem will have several timelines

• Note the subtlety that each timeline has an equation relating its be-
ginning and terminal value

• This is different than the EOV which relates several different timelines
34 CHAPTER 2. MONEY GROWTH

• #5) Identify and algebraically solve each timeline and TV line sepa-
rately

• #6) A rule of thumb is that it is useful to leave numerical computations


to the end

• But that is more a matter of personal taste

2.33 M01#45 - Text


• Since this is our first problem I will give the full text

• Frequently in the future I expect you to look it up

• Notice how the formatting of the problem hints at the timelines needed

• At time t = 0, 1 is deposited into each of Fund X and Fund Y


t2
• Fund X accumulates at a force of interest δt = k.

• Fund Y accumulates at a nominal rate of discount of 8% per annum


convertible semiannually.

• At time t = 5, the accumulated value of Fund X equals the accumulated


value of Fund Y

• Determine k

2.34 M01#45 - Subproblems and TimeLines


• The general procedure mentioned above calls for identifying the sub-
problems first

• The two subproblems are the X fund and Y fund problem

• The general procedure mentioned above calls for construction of time-


lines as the second step

• The 4th step requires identification of the EOV

• The following shows the timelines and EOV


2.35. M01#45 - EQUATIONS Y 35

• X

t2
A(0) = 1 δt = AX (5)
k
0 n

• Y

A(0) = 1 d(2) = 0.08 AY (5)


0 n


AX (5) = AY (5).

2.35 M01#45 - Equations Y


• How do I get an equation for Y?

• Answer: Every topic has key formulae


 −2×5
d(2) =.08
• Use (??). AY (5) = 1 − 2

2.36 M01#45 - Equations X


• How do I get an equation for X?

• Answer: I gave you seven equations for force. Review all seven and
pick the right one

• Note: Selecting the right equation from seven equations is harder than
just plugging into one equation

t2
R5
dt
• e 0 k = AX (5)
36 CHAPTER 2. MONEY GROWTH

2.37 M01#45 EOV


• I have equations for Y and X - what do I do next?

• Answer: Find the Link


 −2×5
t2
R5
dt d(2) =.08
• e 0 k = AX (5) = AY (5) = 1 − 2

2.38 M01#45 - Algebra and Numerics


53
• Solve e 3k = (.96)−10

• k = 102.07

• Check reasonableness using Bernoulli. Approximate (.96)−10 and log((.96)−10 ).


Chapter 3

IRR - Internal Rate of


Return

3.1 Chapter Overview


• We have 8 sub-goals to meet in this chapter
• I) Review of Chapter 1 - single investments under compound interest
• II) Portfolios - Multiple investments - Equation of Value - Equivalence
Principle
• III) IRR: Internal Rate of Return, calculator worksheet
• IV) Comparison of methods: Algebra, TV, IRR
• V) Compound vs. nominal accumulated value
• VI) Trivia on equations of value
• VII) The NPV worksheet
• VIII) Model Exam Problems

3.2 I) Review of Chapter 1 - single investments


under compound interest
• Typical Problem Abe deposits $500 in an account which accumu-
lates to $600 after 10 years. Assume there is a constant, annual,

37
38 CHAPTER 3. IRR - INTERNAL RATE OF RETURN

effective interest rate, i earned by this account. Compute i.

• Let us examine the four adjectives: constant, annual, effective interest


rate

• Under each word we have placed its significance

• For example constant means the same amount each year.

• English

constant annual ef f ective interest rate i


same each year payment period=1 year compound not discount rate

3.3 Solution to Typical Problem


• Rule of 6 states that we should approach the problem solution in mul-
tiple ways

• We present in this slide the EOV and the TV calculator line

• EOV: 600 = 500(1 + i)10 −→ 1.20.1 = 1 + i.

• TV Sheet

10 CPT I −500 0 600


N I/Y PV PMT FV

• i = 1.84%

• Note: TV method faster, more versatile, and less prone to error than
the formula method.

3.4 II) Portfolios - Multiple investments - Equa-


tion of Value - Equivalence Principle
• A portfolio refers to a collection of deposits and withdrawals

• Examples could be your college expenses (and assets) for the semester
or a collection of stocks
3.5. EOV - EQUATION OF VALUE 39

• Any particular item may be worth alot or cost alot; we are interested
in the overall portfolio performance

• We use the letter C to denote either Deposits or Withdrawals

• Here, C stands for Cash Flow, D stands for Deposits, and W stands for
Withdrawals

• Deposit Dk at time tk ⇒ Ck = +Dk

• Withdraw Wj at time tj ⇒ Cj = −Wj

3.5 EOV - Equation of Value


• An important actuarial principle

• Also called in later courses, the Equivalence Principle

• Timeline

C1 C2 C3 ... Cm
t1 t2 t3 ... tm

• EOV N P V = C1 v t1 + C2 v t2 + . . . + Cm v tm

• For each value of i the NPV is different

• So NPV is a function of i, NPV(i).

• Calculating NPV is one major function of an actuary (Discussion:


Why?)

3.6 Yield, IRR


• Sometimes we are interested in the rate for which the N P V (i) = 0.

• Yield, IRR, Rate of Return: A value of i for which (??) equals 0


is called either the yield, the rate of return or IRR, the internal rate
of return.

• One main job of actuaries is to compute the yield of a portfolio of


money flows (Why? Discussion)
40 CHAPTER 3. IRR - INTERNAL RATE OF RETURN

• Alternate version: X X
Dk v tk = Wj v tj (3.1)

• English Version: The present value of deposits equals the present


values of withdrawals

3.7 Typical Multiple PaymentExample


• Bonnie borrows 1000 and agrees to pay back 600 at time t = 1 and an
unknown payment, P at time t = 2. The current interest rate is 10%.
Compute P

• Basic Problem approach: Rule of 6

• Step 0. English, 1. There aren’t 2 subproblems 2. Timeline, 3. PV,


4. No extra EOV, 5. TV Sheet, 6. Numerical Answer

3.8 Solution to Typical Problem


• Timeline

+1000 −600 −P
0 1 2

• EOV 1000 = 600v + P v 2

• Calculator

2 10 1000 − 600 CPT


N I/Y PV PMT FV

• FV = 50

• Real answer is Pay back 550. Why?

3.9 PMT Key


• We have not yet explained the difference between the PMT key and the
FV key
3.10. III) IRR INTERNAL RATE OF RETURN, CALCULATOR WORKSHEET41

• PMT indicates a periodic payment at the end of each payment period,


including the last

• Contrastively, FV indicates a payment at t = n.

• Warning: So both PMT and FV affect net value at time t = n.

• Timeline

+50
1000 −600 −600
0 1 2

• So net payment at time t = 2 is -550.

3.10 III) IRR Internal Rate of Return, calculator


worksheet
• You use the Cash Flow Worksheet for one purpose:

• To find the yield of deposits / withdrawals

• To enter the CF worksheet: (2,2)

• To clear the CF worksheet: 2(-1,1) 2(1,1). 2nd CLR WORK, 2nd QUIT

• To scroll to the time 0,1,2,3 rows: Use  and ; (1,3) and (1,4)

• To enter amounts in a cell: Type number, sign and hit ENTER, (1,2)

• Throughout the course: ENTER and = have similar functions but are
not interchangable

• After spreadsheet is filled, to compute yield: type IRR CPT (2,4), (1,1).

3.11 Slide 3.6 with CF Worksheet


• CF, 2nd CLR WORK, 2nd QUIT, CF

• Scroll to CFo.
42 CHAPTER 3. IRR - INTERNAL RATE OF RETURN

• +1000, ENTER

• Scroll to C01, -600, ENTER

• Scroll to F01, 1, ENTER

• F01 indicates frequency. It will be 1 for now (Discuss later) e

• Scroll to c02, -550, ENTER, F02,1, ENTER

• IRR CPT - Shows yield is 10%

3.12 IV) Comparison of methods: Algebra, TV,


IRR
• We use the following problem

• English: Charles deposits 4000, 3000, 3000 at times t = 0, 1, 2.

• He withdraws 11000 at time t = 3.

• What is his yield on this transaction.

• We will solve this problem in three ways

• Algebra, TV Sheet, CF Worksheet

3.13 Timeline
• TimeLine

−4000 −3000 −3000 11000


0 1 2 3

3.14 Solution by Algebra


• 4000 at t = 0 has present value of 4000 at t = 0

• 3000 at t = 1 has present value of 3000v Why?

• 3000 at t = 2 has present value of 3000v 2


3.15. TV WORKSHEET APPROACH 43

• 11000 withdrawal at t = 3 has present value of 11000v 3 .

• EOV: 4000 + 3000v + 3000v 2 = 11000v 3

• A cubic? Can’t solve. No cubic formula.

3.15 TV Worksheet Approach


• TV Worksheet

3 CPT I −4000 −3000 14000


N I/Y PV PMT FV

• Why did I use 14000 for FV? Why didn’t I use 11000?

• Because both PMT and FV contribute to net value at time t = n.

• TimeLine

−4000 −3000 −3000 11000


−−−−− − − −− − − −− − − −−
14000
−4000 −3000 −3000 −3000
0 1 2 3

3.16 IRR Approach


• 2nd CF CLR WORK, 2nd QUIT, CF

• Scroll to CFo row: -4000, ENTER

• Scroll to C01 row: -3000, ENTER; F01, 1,ENTER

• Scroll to C02 row: -3000, ENTER; F02, 1,ENTER

• Scroll to C03 row: 11000, ENTER; F03, 1,ENTER

• IRR CPT ⇒ 0.0461

• Warning: Note the difference: IRR CPT vs. CPT PV


44 CHAPTER 3. IRR - INTERNAL RATE OF RETURN

3.17 What is the meaning of F01


• If F0# = 1, then

• CF worksheet meaning should be clear

• CFo is timeline value at t = 0

• C01 is timeline value at t = 1

• C02 is timeline value at t = 2

• CF worksheet is simply a timeline.

3.18 F01 not 1


• Still using the same example we have been studying

• 2nd CF CLR WORK, 2nd QUIT, CF

• Scroll to CFo row: -4000, ENTER

• Scroll to C01 row: -3000, ENTER; F01, 2, ENTER

• Scroll to C02 row: 11000, ENTER; F02, 1, ENTER

• IRR CPT ⇒ 0.046083

3.19 What Happened?


• F01 = 2 means use this value on two consecutive times

• It sounds like something powerful

• But I advise not to use it

• Why? Because it gets confusing. C02 now represents t = 3.


3.20. V) COMPOUND VS. NOMINAL ACCUMULATED VALUE 45

3.20 V) Compound vs. nominal accumulated value


• The most frequent error I see among students is the answer to the
following:

• How much is $1 at time t = 0 worth at time t = 0.5, if

• a) the annual effective rate is i = 10%?

• b) the nominal rate is i = 10% payable twice a year?

• c) the rate of discount is i = 10% per year.

3.21 Solution to nominal-compound problem


• In each case we use the formulae from Chapter 2 with that particular
money growth

i(2)
• Nominal: 2 = 5% −→ Anominal (.5) = 1.05

• Compound: Suppose A(0.5) = 1 + j −→ 1 + i = A(1) = (1 + j)2



• So Acompound (.5) = 1.10 = 1.0488.
1
• Discount: Adiscount (.5) = (1−0.10). 5 = 1.0541

• The technique illustrated her is the conversion technique

• It is a technique used when the interest period 6= payment period

3.22 Why
i(2)
• Nominal rate, i(2) by convention means 2 every half year

• But compound rate has no special meaning for half year periods

• We have to use the basic compound formulae to infer its value

• So this problem distinguishes English convention and Accumulated


value formulae
46 CHAPTER 3. IRR - INTERNAL RATE OF RETURN

3.23 Nominal-Compound Formula Summary


• Given i = i(m)

i(m)
 
 
1
Anominal = A(0) 1 + (3.2)
m m
   1
1 m
Acompound = A(0) 1 + i . (3.3)
m

• Note by Bernoulli’s law these two are approximately equal.

3.24 VI) Trivia on equations of value


• The following are facts which are consequences of the definitions and
methods

• The yield need not exist

• The yield need not be unique (There might be two solutions to EOV.)
.72
• It takes (approximately) i time to double your money

3.25 VII) The NPV Worksheet


• We return to the example above with cash flows of -4000, -3000 -3000,
11000 at t = 0, 1, 2, 3

• Enter them in the CF worksheet as shown above.

• Now, instead of asking what IRR will make the NPV 0, we reverse the
question.

• Hit the NPV button and at the I display ENTER 5 (1,2)

• Scroll using the up and down arrows to the NPV display and hit CPT
(1,1)
3.26. VIII) MODEL EXAM PROBLEMS 47

• You get roughly −76

• Now scroll to the I display and ENTER 4 and then scroll to NPV and
hit CPT

• You get 120.67

• These are the values of N P V (i).

• Does it make sense?

• Yes. You incurred alot of debt at t = 01, 2. If interest is higher (5%)


you will lose more money

• If interest is lower (4%) you will make money.

3.26 VIII) Model Exam Problems


• We list useful exam problems on multiple non-periodic investments

• Here and throughout the book Q-IT problems may duplicate exam
problems

• Q-IT#12 Sequential: Two rates

• Q-IT#20 Comparison

• Q-IT#23 Comparison

• Q-IT#32 Sequential: Reinvestment

• M00#1 Comparison: simple/compound

3.27 Q-IT#20
• We sketch the solution

• Please read the problem in the publicly available resources

• We solve as follows Step 0: English, 1: Timeline, 2: EOV, 3: Algebra


48 CHAPTER 3. IRR - INTERNAL RATE OF RETURN

• TimeLine

100 200 300 600


0 n 2n 10

• EOV: 1 + 2v n + 3v 2n = 6v 10 .

• Plug in and solve for v 10 .

• If you have v 10 you can solve for v


1
• If you have v you can solve for i since v = 1+i .

• After all your computations, be sure and answer original question.


Chapter 4

Time and Dollar Weighted


Methods

4.1 Overview
• In this lecture we will go over non-exact methods

• The approximations presented in this chapter are professionally rec-


ognized

• We have 4 subgoals to meet in this chapter.

• I) Overview of Dollar and Time Weighting

• II) The Dollar weighted method

• III) The Time weighted method

• IV) Good Exam Problems

4.2 I) Overview of Dollar and Time Weighting


• These 2 methods apply to yield problems

• Typically, there are many deposits and withdrawals

• Typical examples are bank accounts, stock accounts etc.

49
50 CHAPTER 4. TIME AND DOLLAR WEIGHTED METHODS

• You may want the yield for the entire period of investment or

• You may want the equivalent annual yield

• The approximation methods ease or make possible the calculations

4.3 Two Standard Approximation Methods


• Dollar weighted (also called Money weighted)

• Time weighted

4.4 II) The Dollar weighted method


• Here is a typical problem

• Timeline

10210 DEPOSIT + 4000 WITHDRAWAL − 3000 DEPOSIT + 1000 12982


1 - 1 - 90 5 - 1 - 90 11 - 1 - 90 11 - 1 - 91 12 - 31 - 91
1 5 11
t=0 t= t= t= t=1
6 12 12

• 10210 is the beginning amount in the bank

• 12982 is the end amount in the bank

• The deposits and withdrawals are self explanatory

• Note: Fractions in the above figure indicate fractions of the entire


period

• So for example 11-1-91 occurs 22 months after 1-1-90


11
• 22 months is 12 of the entire 24 month period from 1-1-90 to 12-31-91

4.5 Typical Questions


• Typical Question #1. What is the yield for the entire period?

• Typical Question #2. What is the equivalent annual yield?


4.6. EXACT APPROACH 51

4.6 Exact Approach


• First: Ask for the interest rate for the entire period.
• In other words, entire period is one unit of time
1 5 11
• EOV: 10210 + 4000v 6 − 3000v 12 + 1000v 12 = 12982v
• Multiply through by 1 + i
5 7 1
• 10210(1 + i) + 4000(1 + i) 6 − 3000(1 + i) 12 + 1000(1 + i) 12 = 12982.
• What is the meaning of i?
• i is the interest rate per period, that is for the entire 2 years!

4.7 Approximate
• Bernoulli (1 + i)f ≈ 1 + f i
• Apply Bernoulli to the above
• 10210(1 + i) + 4000(1 + 56 i) − 3000(1 + 7
12 i) + 1000(1 + 1
12 i) = 12982.
• Warning: You must apply Bernoulli to the last equation with i.
• Applying Bernoulli to the equation with present values (v) will give a
different approximation.
• Although this different answer is a correct approximation it is not the
officially recognized dollar weighted approximation
• The approximation we are using, based on i is an industry standard.

4.8 Separate Variables and Numbers


• Let us now solve the equation obtained by using Bernoulli.
• 10210(1 + i) + 4000(1 + 65 i) − 3000(1 + 7
12 i) + 1000(1 + 1
12 i) = 12982.
!
• 10210 + 4000 − 3000 + 1000 +
!
i 10210 + 56 4000 − 7
12 3000 + 1
12 1000

= 12982
52 CHAPTER 4. TIME AND DOLLAR WEIGHTED METHODS

• Easy to solve: This is linear!

4.9 Helpful Intuitions


!
• Interest Amount, I : 12982− 10210 + 4000 − 3000 + 1000 = 772

!
5 7 1
• Exposure: 10210 + 6 4000 − 12 3000 + 12 1000 = 11876

• The exposure is the amount of money exposed to the interest rate


I 772
• Interest rate: i = Exposure = 11876 = 0.065.

4.10 Yield per period vs. Yield per year


• 0.065 is the appoximate (dollar weighted) yield per period
• Let j be the interest rate per year.
• How do you solve for j?
• You use the equation (1 + j)2 = 1.065. (Why?)
• Fundamental Technique: This last equation in j illustrates the
EOV for conversions of interest rates for different periods
• We will cover conversions more thoroughly in a later chapter

4.11 Memorizing vs. Understanding Formulae


• Too many formulae in course to memorize all of them
• For example, in the next few chapters we introduce 50 symbols
• Each symbol has a time line, formula, English, etc.
• The best approach is to memorize a few formulae and then under-
stand/derive the rest.
• We have approached dollar weighted as a Bernoulli approximation to
the exact solution of the equation of value using accumulated values at
t = 1!
4.12. DOLLAR WEIGHTED - METHOD SUMMARY 53

4.12 Dollar Weighted - Method Summary


• Method Summary:

Timeline has begin and end amounts and times, (4.1)


Timeline has deposit and withdrawal amounts and times,
Compute exact EOV,
multiply by 1 + i,
Apply Bernoulli,
Solve linear for i,
solve annual yield vs. i

4.13 III) The Time weighted method


• For the timeline below we ask what is the yield for the period?

• Timeline

BALANCE 100000 105000 60000 115000


DATE 1 - 1 3 - 1 5 - 1 12 - 31
DEP / WITH 0 −50000 50000
NEW BALANCE 100000 55000 110000

• What is the yield factor for the entire period?


105000 60000 115000
• Solution: 100000 × 55000 × 110000 = 1.1975.

4.14 Time Weighted - Method Summary


• Method Summary:

Fill in begin-balance, (4.2)


Fill in deposit/withdrawal,
Fill in end-balance;
Compute interest-factor between each two transactions;
Multiple all interest-factors to obtain period interest factor
54 CHAPTER 4. TIME AND DOLLAR WEIGHTED METHODS

4.15 Subtleties
• The diagrams really help you memorize the formulae

• Time weighted: Time duration of deposits do not matter

• Dollar weighted: Time duration of deposits do matter

• Time weighted: Time of investments does not matter

• Dollar weighted: Time of investments of deposits does matter

• Time weighted: Rate between investments of deposits does matter

• Dollar weighted: Rate between investments of deposits does not


matter

• Time weighted: End of period always used even if no deposit

• Time/dollar weighted: Two possible questions: yield vs. equivalent


annual yield

4.16 V) Exam Problems


• Approximation problems are popular. Almost 1 per exam.

• M05-16 Dollar weighted

• Q-IT-83 Time weighted - too simple

• N00-27 Time weighted 1/2 year vs. whole year

• Q-IT-78 Time weighted - extra information

• ---------------------

• M00-16 time weighted 1st year; dollar 2nd

• M01-31 time/dollar weighting one period

• N01-20 time/dollar weighting one period

• ---------------------
4.16. V) EXAM PROBLEMS 55

• M03-17 Too much computation Bad Problem

• ---------------------

• Q-IT#19 = N01-20

• Q-IT#45 = M01-31

• Q-IT#5 = M03-17

• Q-IT#8 = M03-30
56 CHAPTER 4. TIME AND DOLLAR WEIGHTED METHODS
Chapter 5

Periodic Payments

5.1 Overview
• We have 9 subgoals to meet in this chapter

• I) Annuity Immediate

• II) Rule of Three

• III) Related Symbols: a-s

• IV) Annuity Immediate vs Annuity Due

• V) Annuity-Perpetuity

• VI) Summary of Formula Derivations

• VII) Good Exam Problems

• VIII) Overview of Next Lectures

• IX) Suggested Problem Solving Approach

5.2 I) Annuity Immediate


• Timeline

0 1 1 1 ... 1 (5.1)
0 1 2 3 ... n (5.2)

57
58 CHAPTER 5. PERIODIC PAYMENTS

• English Description. The annuity immediate is the present value


(at time t = 0) of a stream of periodic payments of 1 at the end of the
next n years.
• Symbol. an

5.3 Rule of 6
• Symbol
• Name
• Timeline
• Formula
• Calculator
• English Description

5.4 Timeline
• Done above. See (??)

5.5 Name
• The name is level annuity certain immediate
• Underneath the 4 words of this name we place the implications and
nuances
• Name

Level annuity certain immediate


same amount periodic payments not dependent on survival payment at year end

5.6 Calculator
• Calculator:

n 100i CPT −1 0
N I/Y PV PMT FV
5.7. ENGLISH DESCRIPTION 59

• Example. n = 10, i = 0.01, ⇒ a10 |0.01 =

• Example. n = 5, i = 0.01, ⇒ a5 |0.01 =

• Example. n = 10, i = 0.05, ⇒ a10 |0.05 =

5.7 English Description


• Given above. A level annuity certain immediate is the present value
at time t = 0 of a stream of n payments of 1 at the end of each of the
next n years.

5.8 Proof of Formula for annuities - overview


• Warning. Proofs in Interest Theory are unlike proofs in calculus.

• Proofs in calculus are manipulative, algebraic, analytic

• Proofs in actuarial math are verbal, intuitive, meaningful

• Unlike calculus, actuaries encourage a verbal approach.

• In actual proof, I construct a hypothetical situation

• I then analyze it from two points of view and derive an equation

5.9 Proof of Formula - My viewpoint


• Situation I deposit 1 at t = 0, in a bank account giving i per year,
and withdraw the 1 at time t = n.

• My point of view

1 −1
0 n

• Value of 1 at t = 0 equals 1; Value of 1 at t = n equals v n

• So the present value of my total transaction is 1 − v n .

• Notice: I spent 1 − v n but nothing is left in Bank.


60 CHAPTER 5. PERIODIC PAYMENTS

5.10 Proof of Formula - Bank’s viewpoint


• Bank gives i at end of each year (interest on my deposit of 1 at time
t = 0)

• Bank’s point of view

my 1 ...
0 i i ... i
0 1 2 ... n

• So the present value of the bank’s transactions is ian

5.11 Proof of Formula-inflow=outflow


• We use, as usual, PV to stand for Present Value
• Derivation

Equivalance Principle Outflow = Inflow


(5.3)
English PV my investment = PV of bank’s investment
(5.4)
Formulae 1 − vn = ian
(5.5)
1 − vn
Annuity formula = an
i
(5.6)

• Emphasize: Formula derived verbally, not through geometric series.

5.12 Second Annuity Formula


• Recall the timeline for an annuity immediate

• Timeline

0 1 1 1 ... 1 (5.7)
0 1 2 3 ... n (5.8)

• But the PV of 1 at time t is v t


5.13. NOTATION 61

• So the PV of entire annuity is the present value of the n v t as t goes


from 1 to n


1 − vn
v + v 2 + v 3 + . . . + v n = an = (5.9)
i
• Applying the formula for geometric series to the left side will give the
right side

• The slick verbal derivation is due to Kellison

• As indicated, good actuaries can derive verbally

5.13 Notation
• Given above. an

5.14 II) Rule of Three


• Use of the following three attributes is called the Rule of 3

• Periodic Payment

• Constant Amount

• Deferral

• In problems, we use an approach of building blocks

• Let us now examine each of the three attributes separately

5.15 Constant Periodic Payment


• Timeline

0 2 2 2 ... 2
0 1 2 3 ... n

• PV Present value of above timeline = 2an


62 CHAPTER 5. PERIODIC PAYMENTS

• We can replace 2 by any constant

• Why does this work?

• Interpretation approach. Algebraic Approach.

5.16 Deferral
• Timeline

0 ... 1 1 ... 1
0 1 2 ... 10 11 12 ... 30

• P V = v 10 a20

• v 10 is present value at time t = 0 of a payment of 1 at time t = 10.

• v 10 a20 is present value at time t = 0 of a payment of a20 at time


t = 10.

5.17 Example: Rule of 3


• Timeline

0 ... 5 5 ... 5
0 1 2 ... 10 11 12 ... 30

• English Description 10 year deferred, 20 year annuity, of 5.

• P V = 5v 10 a20

• Present value formula has 3 multiplicands:

• i)deferral (v 10 ), ii) # periods (immediate,20 year), (iii) amount(5).


5.18. III) RELATED SYMBOLS: A-S 63

5.18 III) Related Symbols: a-s


• For each symbol taught such as an we can generate 4-6 more symbols
as follows:

• Dot version e.g. än

• s version e.g. sn

• s dot version e.g. s̈n

• Perpetuity e.g. a∞

• Dot perpetuity e.g. ä∞

• Instead of 48 symbols, we need only learn 8 groups of symbols.

5.19 Meaning of a-s


• Timeline

0 1 1 1 ... 1
0 1 2 3 ... n

• Present value of above stream at time t = 0, is an

• Future value of above stream at time t = n, is sn

5.20 Formula-a-s
• Timeline

a s
0 n

• Use A(n) formula from chapter 2.

• sn = A(n) = (1 + i)n A(0) = (1 + i)n an


1−v n
• But an = i
n n (1+i)n −1
• ⇒ sn = (1 + i)n 1−v
i = (1 + i)n 1−v
i = i
64 CHAPTER 5. PERIODIC PAYMENTS

5.21 Calculator-a-s
• Calculator a:

n 100i CPT −1 0
N I/Y PV PMT CPT

• Calculator s:

n 100i 0 −1 CPT
N I/Y PV PMT FV

• Put in 3 values and obtain 4th value

5.22 IV) Annuity Immediate vs Annuity Due


• Symbol än

• Timeline

än
1 1 1 ... 1 0
0 1 2 ... n−1 n

• English name (Level) Annuity Due (Certain)

• English Description Present value of n periodic payments of 1 at


the beginning of each of n periods.

• Warning. Totally irrational terminology. immediate:due::end of


year:begin of year

5.23 Three Proofs of Formula for Annuity Due


• Use methods of Equation of timelines

• Timeline#1 = 1+Timeline#2
5.24. PROOF 2 - TIMELINE METHODS 65

• Timeline#1
1 1 1 ... 1 0
0 1 2 ... n−1 n

• Timeline#2
0 1 1 ... 1 0
0 1 2 ... n−1 n

• =⇒
• än = 1 + an−1
• Emphasize: Derived without any algebra.

5.24 Proof 2 - Timeline Methods


• an
0 1 1 ... 1 1
0 1 2 ... n−1 n

• än
v v v ... v 0
0 1 2 ... n−1 n

• vän = an
• Here we use, v, the discount factor, from chapter 2.

5.25 Proof 3 - Algebraic Methods



än =(1 + i)an
1 − vn
=(1 + i)
i
1 − vn 1 − vn
= =
iv d
• Exercise: Produce verbal proof, presented in slides 5.8 - 5.11, for
value of än
66 CHAPTER 5. PERIODIC PAYMENTS

5.26 Calculator for annuity due


• BEGIN-END mode

• 2(3,4) BGN 2(1,2) ENTER 2(1,1) QUIT

• Can Toggle on-off

• Recognize calculator is in BEGIN mode by word BGN in upper right


corner of display window

• än

n 100i CPT −1 0
N I/Y PV PMT FV

5.27 Frequent Calculator Error


• Warning: If you were in BGN mode a few weeks ago and forget to get
out of it, the calculator remembers and computes due

• If you found you made such an error, get out of BGN mode

• You need not enter the numbers again after changing BGN mode.

• You can continue to compute whatever you were doing without reen-
tering the numbers.

5.28 V) Annuity-Perpetuity
• Timeline

0 1 1 1 ...
0 1 2 3 ...

• Name Perpetuity (immediate)

• Symbol a∞

• English Description Present value at t = 0 of an infinite stream of


payments of 1 at the end of each year forever.
5.29. PERPETUITY FORMULAE 67

5.29 Perpetuity Formulae


1−v n 1
• a∞ = limn→∞ an = limn→∞ i = i

1−v n 1
• ä∞ = limn→∞ än = limn→∞ d = d

• Calculator trick. a∞ |0.01 ≈ a10000 |0.01

5.30 VI) Summary of Formula Derivations


• Derive perpetuity by taking limits

• Derive s by multiplying by (1 + i)n

• Derive ä - a relation by using v.

5.31 s, s̈, a, ä
• Timeline

0 1 1 1 ... 1
0 1 2 3 ... n
an sn

• Timeline

1 1 1 1 ... 1
0 1 2 3 ... n−1 n
än s̈n

5.32 VII) Good Exam Problems


• We classify exam problems using the following 5 symbols
68 CHAPTER 5. PERIODIC PAYMENTS

• Symbols: I,Increasing,Decreasing
P,Perpetuity
Inf,Inflation
C,Conversion
A,Algebraic Manipulation

• Most duplicate problems in QIT have been caught but not all.

• N05#23 I (Calculator Too easy)

• M00#14 P/C

• N00#20 P/I

• N01#05 P/Inf

• N05#12 P/I

• N05#09 P/Inf

• QIT#25 P/A

• QIT#14 P/Inf

• M01#50 P 3 Timelines (My favorite problem since it emphasizes ex-


ecutive function)

• M05#24 A

• M03#33 A (Very good comparison problem)

• N00#44 I/I

• QIT#6 I/P/A

• M05#14 I - 2 pieces

• M03#45 P/Inf/ 3 Timelines - Reinvestment flavor

• QIT#29 P/C

• QIT#18 I/C

• QIT#11 P/Inf (Almost duplicates M03#45)

• M01#5 P/A (Duplicate QIT)


5.33. VIII) OVERVIEW OF NEXT LECTURES 69

• M01#17 P/C (Duplicate QIT)

• M03#22 I/P/A (Duplicate QIT)

• M05#4 P/A (Duplicate QIT)

• N01#16 P/I/C

• M05#17 P/I - One problem; Calculator easy

• QIT#93 Inf (Level month; increasing yearly)

• QIT#89 A (Retirement problem; Outstanding verbal problem)

• QIT#87 a (deferred annuity; 2 rates)

• QIT#86 I/A

5.33 VIII) Overview of Next Lectures


• Today we learned 6 symbols

• an , sn , a∞ , än , s̈n , ä∞

• Next few lectures: We will learn 44 more symbols

• This module is one of the hard parts of the course since we must . . .

• . . . learn Actuarial Notation

5.34 IX) Suggested Problem Solving Approach


• Please read problem Q-IT#29 to get the most of these next few slides

• Or, you can infer problem from slide solutions

• This problem also shows my preferred approach to payment period vs.


interest period
70 CHAPTER 5. PERIODIC PAYMENTS

• Timeline

32 ...
10 10 10 ...
0 1 2 3 ...

• Conversion Trick: j, rate per 3 years

• EOV: 32 = 10a∞ |j

5.35 QIT#29 -2nd Timeline


• Timeline

X ...
1 1 1 ...
0 1 2 3 ...

• Conversion Trick: k, rate per 4 months, 1/3 year

• EOV:X = a∞ |k

5.36 QIT#29 - Relate j, k, i


• i, rate per year.

• Use basic formula: A(n) = A(0)(1 + i)n .

• (1 + k)3 = 1 + i

• (1 + i)3 = 1 + j ⇒

• (1 + k)9 = 1 + j

5.37 QIT#29 - Solve


• 32 = 10a∞ |j

• Rule of 6. Use Formula


1
• a∞ |j = j
5.38. QIT#29 - HIGHSCHOOL ALGEBRA 71

10
• So j = 32
1
• X = a∞ |k = k

5.38 QIT#29 - HighSchool Algebra


• (1 + k)9 = 1 + j.
10
• j= 32
1
• X= k

• =⇒

• X=
72 CHAPTER 5. PERIODIC PAYMENTS
Chapter 6

Increasing Decreasing
Annuity

6.1 Overview
• We have 9 subgoals to meet in this chapter.

• I) Increasing and Decreasing Annuities

• II) Problem Solving Methods Using a Building Block Approach

• III) N05#23 Illustrates Constant Multiple from Rule of 3

• IV) N01#16 Illustrates Conversion Method for Differing Payment/Interest


Periods

• V) M05#17 Illustrates the Addition of Timeline Approach

• VI) N05#12 Illustrates Use of Variables and the Initial Offset Tech-
nique

• VII) N00#44 Illustrates Multiple Approaches to the Same Problem

• VIII) Decreasing Annuity Formula

• IX) Increasing annuity formula (Brovender’s trick)

73
74 CHAPTER 6. INCREASING DECREASING ANNUITY

6.2 I) Increasing and Decreasing Annuities


• Timeline

1 2 3 ... n
0 1 2 3 ... n

• The notational symbol for the Present Value for an increasing annuity
immediate is: (Ia)n

• The notational symbol for the Future Value (t = n) for an increasing


annuity immediate is: (Is)n

6.3 Other Increasing Annuities


• For an increasing annuity due . . .

• start with a payment of 1 at t = 0 . . . and

• end with a payment of n at t = n − 1.

• The symbol for the Present Value for an increasing annuity due is:
(Iä)n

• The symbol for the Future Value (t = n) for an increasing annuity due
is: (I s̈)n

6.4 Decreasing Annuity


• Timeline

n n−1 n−2 ... 1


0 1 2 3 ... n

• The symbol for the Present Value for a decreasing annuity immediate
is: (Da)n

• The symbol for the Future Value (t = n) for a decreasing annuity


immediate is: (Ds)n
6.5. OTHER DECREASING ANNUITIES 75

6.5 Other Decreasing Annuities


• For a Decreasing annuity due . . .
• start with a payment of n at t = 0 . . . and
• end with a payment of 1 at t = n − 1.
• The symbol for the Present Value for a decreasing annuity due is:
(Dä)n
• The symbol for the Future Value (t = n) for a decreasing annuity due
is: (Ds̈)n

6.6 Review of a − s distinction


• Due starts at 0 while immediate starts at 1
• Both Due and immediate are for n payments
• The a symbol corresponds to the present value at t = 0
• The s symbol corresponds to the accumulated future value at t = n.
• WARNING: For s and due: The last payment is at t = n − 1 but
the accumulated value is at t = n

6.7 II) Problem Solving Methods Using a Building


Block Approach
• The above is all the theory you need!
• We first solve problems in terms of symbols
• We will do a few calculator/formula explanations after we do problems!
• This is the proper approach to problem solving thinking.
• Each problem presented below introduces a basic problem solving tech-
nique.
• See the first slide, 6.1, of this lecture for a summary
• Note: A complete list of annuity problems was given in slide 5.32
76 CHAPTER 6. INCREASING DECREASING ANNUITY

6.8 The General Problem Solving Approach


• 1. Only 3 Building Blocks level, increasing, decreasing

• 2. Subproblems Payment patterns; use 3 building blocks

• 3. Timelines Set up all problem timelines

• 4. EOV Write the EOV Formulae; use actuarial symbols

• 5. Algebra Plug in formula for actuarial symbols; then algebra and


numerics!

6.9 III) N05#23 Illustrates Constant Multiple from


Rule of 3
• Problem The present value of a 25-year annuity-immediate with a
first payment of 2500 and decreasing by 100 each year thereafter is X.
Assuming an annual effective interest rate of 10 percent, calculate X.

• Timeline

2500 2400 2300 ...


25 × 100 24 × 100 23 × 100 ...
0 1 2 3 ...

• Method of factoring out 100

• X = 100 × (Da)25

• Problem is way too easy. Good problems have two parts. This one
has one part!
6.10. IV) N01#16 ILLUSTRATES CONVERSION METHOD FOR DIFFERING PAYMENT/INTEREST

6.10 IV) N01#16 Illustrates Conversion Method


for Differing Payment/Interest Periods
• Olga buys a 5-year increasing annuity for X. Olga will receive 2 at the
end of the first month, 4 at the end of the second month, and for each
month thereafter the payment increases by 2. The nominal interst rate
is 9 percent convertible quarterly. Calculate X.

• Timeline

2 4 6 ...
2×1 2×2 2×3 ...
0 1 2 3 ...

• 5 years = 5 × 12 = 60 months

• =⇒ X = 2 × (Ia)60 |k

6.11 Who is k?
• k is the interest rate per month. (The problem says so!)

• But problem gives 0.09 = i(4) =⇒ Quarterly

• How do you deal with Quarterly vs. Monthly

• Use Chapter 1! A(n) = A(0)(1 + i)n

• 12 months in a year. 4 quarters in a year.


 4
12 i(4)
• =⇒ (1 + k) = 1 + i = 1 + 4

6.12 V) M05#17 Illustrates the Addition of Time-


line Approach
• At an annual effective interest rate of i the present value of a perpetuity-
immediate starting with a payment of 200 in the first year and increas-
ing by 50 each year thereafter is 46,530. Calculate i.
78 CHAPTER 6. INCREASING DECREASING ANNUITY

• Timeline

200 250 300 350 ... =

150 150 150 150 ... +


50 × 1 50 × 2 50 × 3 50 × 4 ...

0 1 2 3 4 ...

• EOV 46350 = 150a∞ + 50(Ia)∞

6.13 Very typical approach


• This is a typical problem of pattern recognition:

• First reduce stream of payments to basic patterns

• Remember: There are only 3 basic patterns; go over them; see which
ones fit best in this problem.

• Then use the technique of addition of timelines.

6.14 VI) N05#12 Illustrates Use of Variables and


the Initial Offset Technique
• Megan purchases a perpetuity-immediate for 3250 with annual pay-
ments of 130. At the same price and interest rate, Chris purchases an
annuity-immediate with 20 annual payments that begin at amount P
and increase by 15 each year thereafter. Calculate P.

• Timeline

130 130 130 130 ...


0 1 2 3 4 ...
6.15. VII) N00#44 ILLUSTRATES MULTIPLE APPROACHES TO THE SAME PROBLEM 79

• Timeline

P P + 15 P + 30 P + 45 ... =

15 15 × 2 15 × 3 15 × 4 ... +
P − 15 P − 15 P − 15 P − 15 ...

0 1 2 3 4 ...

• Notice initial offset: Constant timeline begins at P − 15 instead of P.


Makes formulae smoother.

• =⇒ 3250 = (P − 15)a20 + 15(Ia)20 = 130a∞

• This problem illustrates usual approach:

• Two timelines

• Breakdown of stream of payments into two basic streams.

6.15 VII) N00#44 Illustrates Multiple Approaches


to the Same Problem
• Joe can purchase one of two annuities: Annuity 1: A 10-year decreas-
ing annuity-immediate, with annual payments of 10, 9, 8, . . . , 1.
Annuity 2: A perpetuity-immediate with annual payments. The perpe-
tuity pays 1 in year 1, 2 in year 2, 3 in year 3, . . . , and 11 in year
11 . After year 11, the payments remain constant at 11. At an annual
effective interest rate of i, the present value of Annuity 2 is twice the
present value of Annuity 1 . Calculate the value of Annuity 1.

• Timeline #1

X = (Da)10 ...
10 9 8 ...
0 1 2 3 ...

• For timeline #2 we have two reasonable ways to break it up


80 CHAPTER 6. INCREASING DECREASING ANNUITY

6.16 Timeline #2 - Approach A


• Timeline #2

1 2 3 ... 10 11 11 ...
0 1 2 3 ... 10 11 12 ...

• Approach A

1 2 3 ... 10 | 11 11 ...
0 1 2 3 ... 10 | 11 12 ...

• Approach A (Ia)10 + v 10 11a∞

6.17 Timeline #2 - Approach B


• Timeline #2

1 2 3 ... 10 11 11 ...
0 1 2 3 ... 10 11 12 ...

• Approach B

1 2 3 ... 10 11 | 11 ...
0 1 2 3 ... 10 11 | 12 ...

• Approach B (Ia)11 + v 11 11a∞

• Other approaches using Due also possible.

6.18 Equation of Value


• First: Recall that the present value of annuity two is twice that of
Annuity one
6.19. VIII) DECREASING ANNUITY FORMULA 81

• Approach B: (Ia)11 + v 11 11a∞ = 2(Da)10

• Approach A: (Ia)10 + v 10 11a∞ = 2(Da)10

• Which approach right? Both! Use the approach where the algebra
simplifies!

• For a HW exercise: Repeat the above set up of 2 approaches using


timelines

• Give two more approaches using Increasing Annuity Due

• Try solving all 4 problems.

• Show that algebraically only one works very smoothly

• (You can peek at published solutions to help you)

6.19 VIII) Decreasing Annuity Formula


• We again emphasize the verbal derivation approach

• I deposit n at time t = 0

• I then take out exactly 1 at t = 1, 2, 3 . . .

• Timeline

n − an
n n−1 n−2 n−3 ... 1
0 1 2 3 ... n−1

• At t = n nothing of mine is left in Bank.

• The present value of my transactions are n − an . Why?


82 CHAPTER 6. INCREASING DECREASING ANNUITY

6.20 Formulae: Decreasing Annuity Bank’s view-


piont
• At time t = 1 bank gives me ni because of n at time t = 0.

• At time t = 2 bank gives me (n − 1)i because of n − 1 left at time


t = 1.

• Timeline

i(Da)n
ni (n − 1)i (n − 2)i ... i
0 1 2 3 ... n−1

• So bank spends i(Da)n

6.21 Formulae: Decreasing Annuity


• Equivalence principle: My total payments = Banks total payments.


i(Da)n = n − an (6.1)


n − an
=⇒ (Da)n = (6.2)
i

6.22 Related Formulae


• For (Dä)n replace i by d

• For (Ds)n multiply by (1 + i)n

• For (Ds̈)n do both of above (replace and multiply)

6.23 Calculator
n−an
• Suppose we want to compute (Da)n = i

• #1: Compute an .

• #2: Hit ±
6.24. IX) INCREASING ANNUITY FORMULA (BROVENDER’S TRICK) 83

• #3: Hit +n =.

• So you now have in calculator display window n − an

• #4 Hit ÷ RCL i =

• Note: RCL allows you to peek at input on TV line.

• #5 Hit × 100 =

• This compensates for fact that i key multiplies by 100.

• Example. Apply the above for n = 2, i = 0.10.

6.24 IX) Increasing annuity formula (Brovender’s


trick)
• Timeline

1 2 3 ... n +
n n−1 n−2 ... 1 =
0 n+1 n+1 n+1 ... n+1

• (Da)n + (Ia)n = (n + 1)an Why?

• But we have formulae for (n + 1)an and (Da)n

• So we can compute formula for (Ia)n


än − nv n
(Ia)n = (6.3)
i

6.25 Other Formulae


• For the due version replace i in denominator by d

• For the s version multiply a version by (1 + i)n .


än − nv n än 1
(Ia)∞ = lim = lim = (6.4)
n→∞ i n→∞ i id
84 CHAPTER 6. INCREASING DECREASING ANNUITY

• Similarly
1
(Iä)∞ = (6.5)
d2

6.26 Calculator: Increasing Annuity


• We follow Brovendor’s free calculator e-book,

• http://www.soa.org/files/pdf/FM-23-05.pdf page 29

• Example Use n = 10, i = 0.10 as we go over theory

• Here are the steps to compute an increasing n year annuity immediate

• #1: Enter BGN mode.

• #2: Enter keys to compute än

• #3: Enter n for FV.

• Why? Because this subtracts an amount of n at time t = n

• The present value of this −n at time t = n is −nv n , part of the


Increasing annuity formula

• Calculator:

n 100i CPT −1 n
N I/Y PV PMT FV

• WARNING: Only use -1 for payments. Do not use -2 or -3 (Multiply


afterwards). Why?

• Display window now gives you än − nv n ,

• #4: ÷ RCL i =

• #5: × 100 =
Chapter 7

Inflation

7.1 Overview
• This chapter is about inflation

• We have 7 subgoals to meet in this chapter

• I) Inflation definition

• II) Inflation algebra

• III) Inflation example

• IV) Inflation algorithm

• V) Inflation vs. Increasing Annuity

• VI) Model Example

• VII) Other Inflation problems

7.2 I) Inflation definition


• Suppose Milk/OJ/Beer costs 1 at t = 0

• But costs 1.50 at t = 1.

85
86 CHAPTER 7. INFLATION

• We say the price of milk/oj/beer inflated 50 percent

• Because you need 50 percent more to purchase the item

7.3 Interest and Inflation


• Assume interest is 0.75

• Assume inflation factor is 0.50

• We can ask a Present Value problem

• What is the present value of milk/oj/beer at t = 1

7.4 Solution to Interest/Inflation


• Timeline

1 .50 inflation 1.50


PV .75 interest 1.50
1.50
PV = = .857
1.75
0 1

7.5 Formula for PV of Inflation Makes Sense


• Suppose I have .857 at t = 0.

• Then at t = 1, I have 1.75 × .857 = 1.50

• Now milk/oj/beer costs 1 at t = 0

• But the milk/oj/beer costs 1.50 at t = 1.

• So this method works

• So my .857 at t = 0 gives me 1.50 at t = 1 and I can buy one unit of


milk/oj/beer
7.6. II) INFLATION ALGEBRA 87

7.6 II) Inflation algebra


• Let i be the interest rate

• Let g be the inflation rate

• Define
1 1+g
0
= (7.1)
1+i 1+i

• i0 is called the modified or inflation modified interest rate

7.7 Basic Method for Inflation


• If you are given a problem with an inflation rate g and interest i

• Then follow this algorithm

• 1) First replace i and g with i0

• 2) Replace the stream of payments P, P (1 + g), P (1 + g)2 , . . .

• . . . with the stream of payments P, P, P, P

• This stream of level payments, P , is fictitious, it does not correspond


to anything in the real world

• 3) Compute the stream of P at rate i0 as an annuity due

• Either P än :i0 or P ä∞ :i0

• 4) Then use a deferral factor vim where m is defined as follows:

• Payment at m is P while payment at m + 1 is P (1 + g).

• Notice that the deferral factor uses i not i0 since inflation is not yet
present.

7.8 Basic Method Timelines


• We can compactly summarize this approach by using two equivalent
timelines
88 CHAPTER 7. INFLATION

• Original Timeline

P i P (1 + g) P (1 + g)2 ...
m rate m+1 m+2 ...

• Modified Timeline

P i0 P P ...
m rate m+1 m+2 ...

• The present value of the two timelines is the same

• The Present Value of the modified Timeline is easy to compute

• P vim ä∞ :i0

7.9 IIII) Inflation example


• Let us use the example from the beginning of this chapter

• I need to buy milk at t = 1; it costs 1 at t = 0; inflation is 50%

• g = 0.50. i = 0.75.
1+g 1.50
• Compute 1+i = 1.75 = 0.857
1 1
• Now solve 1+i0 = 0.857 (Hint: Use x key.)

• Obtain 1 + i0 = 1.16666 =⇒ i0 = 0.16666

7.10 Example Continued


• Original Timeline

PV i = 0.75 1.50
P V = 0.857
0 1
7.11. IV) INFLATION ALGORITHM 89

• Modified Timeline

PV i0 = 0.16666 1.00
P V = 0.857
0 1

• So you price milk without the inflation as costing 1

• You find you need 0.86 at t = 0 to purchase milk at t = 1.

• You get same answer 0.86 whether you use . . .

• . . . the inflation factor, 0.50, and the interest rate, i = 0.75, or

• . . . the modified interest rate, i0 = 0.1666 and no inflation

• The method works because you hide the inflation in the modified in-
terest rate

7.11 IV) Inflation algorithm


If you see a problem with i and g


compute (??), i0
Set up timeline with i0 and level payments, P
P is the begin-year payment in the first year of inflation
(7.2)
Compute PV as you ordinarily would with an annuity due
The deferral factor would be vim
where (m, m + 1) is the first year of inflation
The deferral factor uses i not i0 (7.3)

• This method avoids the need for Geometric series

• Warning: If you have to compute accumulated values (s) first com-


pute present values and then multiply by (1 + i)n (Use i not i0 )
90 CHAPTER 7. INFLATION

7.12 V) Inflation vs. Increasing Annuity


• The rule of 6 states that certain parts of the course deal with English
• The next two descriptions both involve the English word increasing
• See if you can find the difference

7.13 Two uses of increasing


• An increasing annuity of 1 per year pays 1 at t = 0, 2 at t = 1, 3 at
t = 2, . . .
• Inflation of 10 percent per year increases from 1 at t = 0 to 1.1 at
t = 1, . . .
• Both of these use the word increasing
• However an Increasing Annuity increases arithmetically; inflation in-
creases geometrically
• You can recognize an Increasing annuity because the amount increases
• Contrastively, inflation has an increasing percent

7.14 VI) Model Example


• Q-IT#14 N01#5 is illustrative
• We approach this problem with the Example method for Timeline
constructions
• This method is very useful in problems when you don’t know how to
begin
• The idea is simple

Make sure the timeline has 2-3 sample values corresponding to each English phrase
(7.4)
• Please read the problem in the course resources
• Then see if you can come up with the payment values at t = 6, 7, 8.
7.15. EXAMPLE: Q-IT#14 N01#5 91

7.15 Example: Q-IT#14 N01#5


• Timeline
10 10 10 10 10 (1 + K)10 (1 + K)2 10 (1 + K)3 10 . . .
0 1 2 3 4 5 6 7 8 ...

• i = 0.092
• Calculate K
• Review: A timeline is correct if it contains all information in English
version of problem.

7.16 Increasing: Arithmetic or Geometric?


• Problem speaks about increasing by K percent
• So use Inflation rather than Increasing Annuity approach

7.17 Solution - Two timelines


• Timeline #1
10 10 10 10
0 1 2 3 4

• Timeline #2
10 (1 + g)10 (1 + g)2 10 (1 + g)3 10 ...
5 6 7 8 ...

• So problem timeline is sum of these two timelines


• I emphasize many times - draw each time line separately

7.18 Solution - Timeline #1


• Timeline #1
10 10 10 10
0 1 2 3 4

• EOV: Easy. P VI = 10a4 0.092


92 CHAPTER 7. INFLATION

7.19 Solution - Timeline #2


• Timeline #2

10 (1 + g)10 (1 + g)2 10 (1 + g)3 10 ...


5 6 7 8 ...

• How do you approach modeling this time line?

• Remember: We want symbolic solution first; formula/numerics second.

7.20 Solution - Timeline #2


• First: Find first year with inflation:

• Year 5 is first inflation year

10 (1 + g)10 (1 + g)2 10 ...


5 6 7 ...

• Second: Compute i0 :

1 1+K
• 1+i0 = 1+.092

• Third: Make all payments level, 10

• Fourth: Use deferral factor v 5 where (5,6) is first year of inflation.

• Year 5

10 10 10 ...
5 6 7 ...

• We next compute present value of this equivalent timeline using the


rule of 3.
7.21. APPLICATION OF RULE OF 3 93

7.21 Application of Rule of 3


• Annuity symbol is

• ä∞ i0

• Notice we use a due approach (Always done with inflation)

• Amount is . . .

• . . . 10.

• . . . This is the amount at the beginning of the annuity due at t = 5.

• Deferral factor is . . .
5
• . . . v0.092

• . . . Why do I use 0.092 rather than i0 ?

• . . . Because prior to t = 5 only 0.092 present. No inflation yet!

7.22 Solution Summary


• The EOV Timeline #2 is modeled with
5 1 1+K
• v0.092 10ä∞ i0 , 1+i0 = 1+.092

• We have done no work but we have complete algebraic solution


5 1 1+K
• 167.50 = 10a4 0.092 + v0.092 10ä∞ i0 , 1+i0 = 1+.092

• This is a typical problem. Two timelines and some non-standard as-


pect (like inflation)

7.23 Solution Algebra


• 10a4 0.092

• Timeline #1

4 9.2 CPT 3.2255 −1 0


N I PV PMT FV
94 CHAPTER 7. INFLATION

n
• ä∞ i0 = limn→∞ än i0 = lim 1−v
d0 =
1
d0

1 1+i0
• d0 = i0

1+i0
• 167.50 = 10 × 3.2255 + 0.6440 × 10 × i0

• Solve. i0 = 1
20 (5 percent)

7.24 Finish Solution


1+g 1
• 1+i = 1+i0

1+K 1
• 1.092 = 1.05

• =⇒ K = 0.04

7.25 VII) Other Inflation problems


• Refer to the list in Chapter 5.32

• Q-IT#14 M03#45 N05#9 are good inflation problems.

• N05#9 is a good practice problem for class quiz or Homework

• Q-IT#60 is an excellent typical problem for SOA exam practice


Chapter 8

Miscellaneous Annuities

8.1 Topics
• Today we cover miscellaneous examples

• We cover continuous annuities and conversions

• We have 8 subgoals to meet in this chapter

• I) Basic verbal convention

• II) Discrete example - Interest Conversion method

• III) Discrete example - Monthly annuity symbol

• IV) Continuous annuities - Daily Approximation

• V) Continuous annuities - Exact method

• VI) Continuously increasing annuities

• VII) Annuities increasing monthly

• VIII) Annuities, level monthly, increasing yearly

8.2 Why are these annuities called Miscellaneous


• We have already learned how to solve problems using interest conver-
sion methods

95
96 CHAPTER 8. MISCELLANEOUS ANNUITIES

• So you don’t really need additional symbols relating different interest


and payment periods

• But you are responsible to recognize notation, timeline, formulae

8.3 I) Basic verbal convention


• The phrase annual payment of P payable m times a year...
P 1
• ...by convention means: That you pay m each m th of a year

8.4 Interest Conversion Discrete


• But what is interest rate, j?

• j is rate per m-th of a year


i(m)
• If given rate is nominal i(m) then use m

• If given rate is annual effective i then use (1 + j)m = 1 + i

• In some problems you must combine nominal-effective methods to get


j

8.5 3 Basic Principles of Conversion


• Principle #1: Never use i(m) .
i(m)
• Always immediately replace i(m) with j = m

• j is interpreted as the interest rate per m-th of a year

• Principle #2: If you have some rate j and need the corresponding d

• Then write d(j) to emphasize it is the d associated with j


j
• Of course d(j) = 1+j .

• Main Conversion Principle #3: Suppose you have two timelines

• One timeline uses periods of length P


8.6. II) DISCRETE EXAMPLE - INTEREST CONVERSION METHOD97

• The other timeline uses periods of length Q

• Further assume, there are n periods of P in one period of Q

• Then the basic conversion formula is

(1 + iP )n = 1 + iQ . (8.1)

8.6 II) Discrete example - Interest Conversion


method
• Find the accumulated value at the end of 10 years if 1000 is invested
annually payable at the end of each quarter at an annual rate of 0.04

• Point: Quarter = Period of payment 6= Period of Interest = Year

• There are two approaches

• First we will review the approach I advocate: Conversion of interest


rates

• Then we will use the approach with quarterly annuity symbols (interest
per year and payments per quarter)

8.7 Conversion approach


• annual payment of 1000 payable quarterly ←→ 250 payable quarterly

• Calculator:

40 100j 0 − 250 CPT


N I/Y PV PMT FV

• N: 10 years × 4 quarters = 40 payment periods

• PMT: Use per payment period, payment 250

• Want accumulated value, FV

• j But what is j
98 CHAPTER 8. MISCELLANEOUS ANNUITIES

8.8 2 Interest Conversions


0.04
• If i = 0.04 is nominal rate per year =⇒ j = 4 is rate per quarter
1
• If i = 0.04 is effective rate per year =⇒ 1 + j = (1 + .04) 4 is rate per
quarter

• This is a very common source of confusion

8.9 Finish of Problem


• In this problem annual rate is 0.04.
1
• So 1 + j = 1.04 4 =⇒ j = 0.009853

• We finish computation: Answer is 250s40 j = 12184.73

• I call this approach the conversion method and advocate its use.

8.10 III) Discrete example - Monthly annuity sym-


bol
• We now present the actuarial symbol and formulae when payment
period 6= interest period

• You should know the symbols

• It is however OK to approach the problem with conversions mentioned


above and not use symbols

8.11 Monthly annuity method


(m)
• Symbol ay i

• English interpretation Annual rate of i; payments for y years; pay-


ments m times a year.
8.12. IV) CONTINUOUS ANNUITIES - DAILY APPROXIMATION 99

• Timeline
1 1 1 1
...
m m m m
1 2 3
0 ... y years
m m m

1−viy
• Formula i(m)

• Solution to Problem
(4)
• Accumulated value at end of 10 years = 1000s10 .04

• Why 1000? Why not 250?


1
• Answer: Because monthly annuity pays m not 1 each payment period

• This is a bit confusing which is why I advocate use of the interest


conversion method

8.12 IV) Continuous annuities - Daily Approxima-


tion
• We now deal with continuous annuities
1000
• An Annual payment of 1000 payable continuously ←→ 365 every day
1000
or 365×24 every hour

• It literally means you pay 1000 × ∆t every ∆t units of time

8.13 English for Continuous annuity payments


1000
• Is using 365 an approximation?

• Answer: Yes and no.

• Yes: It is an approximation

• But the approximation will usually agree on the penny with the exact
answer
100 CHAPTER 8. MISCELLANEOUS ANNUITIES

• Use of approximation methods like these is common in actuarial prac-


tice
• To show how good the approximation is we solve the following problem
in three ways
• Find the accumulated value in 10 years of an annual payment of 1000
payable continuously at an annual effective rate of 0.04
• We will solve this problem 1) Exactly 2) By a daily approximation and
3) compare to the quarterly problem done previously
• All three answers are very close

8.14 Interest Conversion Continuous


P
• So P per year payable continuously can be approximated by 365 as
daily rate
• What about the interest rate?
δ
• We can use 365 as daily force
• If the problem gives you the annual effective rate, i
• Then first convert i to δ using the equation 1 + i = eδ .
δ
• Then use 365 as daily force

8.15 Approximate solution using daily approxima-


tion
• Calculator:
ln(1.04) 1000
365 × 10 100 0 − CPT
365 365
N I/Y PV PMT FV

ln(1.04)
• Note that 1.04 = eδ so that δ = 365

• The answer is 12243.86 which is 21 % bigger than the 12184.73 result


we obtained above for the quarterly payments.
8.16. V) CONTINUOUS ANNUITIES - EXACT METHOD 101

8.16 V) Continuous annuities - Exact method


• Symbol an

• English Description annuity payable continuously at annual effec-


tive rate i, (at constant force of interest δ)

• Formula
n
1 − e−δn
Z
an = e−δt dt = . (8.2)
t=0 δ

• Similarly
eδn − 1
sn = . (8.3)
δ

• Plugging in we obtain for our problem the exact answer: 1000 ×


eln(1.04)×10 −1
ln(1.04) = 12244.66.

• Comparing the 3 answers - payable quarterly, payable daily and payable


continuously - we see

• 12184.73, 12243.86, 12244.66

• As can be seen, the daily approximation differs from the exact answer
by less than 1 dollar.

8.17 Derivation of Integral Formula


Rn
• Let us prove that an i = 0 e−δt dt.

• At instant of time ∆t

• You receive amount 1 × ∆t

• The present value of this requires a discount factor of e−δt .


Rn −δt dt
• The limiting sum of all these present values is t=0 e
102 CHAPTER 8. MISCELLANEOUS ANNUITIES

8.18 VI) Continuously increasing annuities


 
• Symbol Ia
n δ

• English interpretation Increasing annuity of one per year convert-


ible continually at force δ
Rn an i −nv n
• Formula 0 te−δt dt = δ

• Note: Requires integration by parts but you can just memorize formula

• Also note: nv n = ne−δn .

• Finally note:  can let n go to ∞. Get limit formula for increasing


 You
perpetuity Ia
∞ δ

8.19 Verbal derivation of Integral


• At instant of time ∆t

• You receive amount t × ∆t

• The present value of this requires a discount factor of e−δt .

• Adding
R n −δt up all these present values and taking the limit we obtain
0 te dt

• The actual integration uses the technique, integration by parts.

8.20 VII) Annuities Increasing monthly


 (m)
(m)
• Symbol I a
n i

• English interpretation Annual rate of i; payments for n years; pay-


ments increasing m times a year.
8.21. ILLUSTRATIVE EXAMPLE - ANNUITIES INCREASING MONTHLY103

• Timeline
1 2 3 nm
...
m2 m2 m2 m2
1 2 3
0 ... n years
m m m
(m)
än i −nv n
• Formula i(m)

• I would not memorize this formula

• In problems I would advocate the conversion method used above

• We will do an example below


 (m)
• You can derive corresponding formulae for the the following: I (m) s ,
n i
 (m)  (m)
(m)
I ä (m)
, I s̈
n i n i

• You can also derive the formula for the associated perpetuities

8.21 Illustrative example - Annuities increasing monthly


• Compute the present value of an increasing annuity immediate for
10 years with monthly payments that pays 25 at t = 1 and increases
payments by 5 each month. The annual effective rate is 0.04.

• We will solve this problem without using the monthly increasing an-
nuity symbol and formula

• As an exercise you may want to check and redo the problem that way

• English Description Present value; n = 10; end of period payments;


first payment 25, i = 0.04; increasing monthly by 5

• Timeline

25 30 35 ... 20 + 120 × 5
0 1 2 3 ... 120

• 10 years and 12 months per year =⇒ 10 × 12 = 120 periods


104 CHAPTER 8. MISCELLANEOUS ANNUITIES

• Represent as sum of two time lines

• Timeline

20 20 20 ... 20 +
5 10 15 ... 120 × 5
0 1 2 3 ... 120

 
• P V = 20a120 j + 5 Ia , (1 + j)12 = 1.04.
120 j

• As an exercise in using the calculator and the memory keys, compute


a numerical answer.

8.22 VIII) Annuities, level monthly, increasing yearly


• John starts working at age 30 at an annual salary of 36000 payable
monthly. John’s salaary increases 1200 per year payable monthly with
level payments every month. John works until age 55 at which time
he retires. Compute an equivalent level monthly payment X such that
25 years of monthly payments, X, have the same present value of what
John gets paid.

• This problem has the unusual feature of level monthly payments but
increasing annual payments.

• We create a new actuarial symbol for this.

8.23 Algebra - Increasing Annuity payable m-th ly


increasing per year
 (m)
• Symbol Ia
n i

• English interpretation Annual rate of i; level payments at end of


each month; payments increase each year
8.24. ILLUSTRATIVE PROBLEM ANALYZED 105

• Timeline
1 1 1 2 2 2 n
... ... ...
m m m m m m m
1 2 12 13 2 24
0 ... ... ... n years
m m m m m m

• Formula Here memorizing a technique is preferable to memorizing a


formula.

• See the illustrative solution below

8.24 Illustrative problem analyzed


• Let us analyze this as a sum of timelines.

• Timeline

X X ... X X X ... X ... X=


3000 3000 . . . 3000 3100 3100 . . . 3100 . . . 2900 + 25 × 100 =
2900 2900 . . . 2900 2900 2900 . . . 2900 . . . 2900 +
100 100 . . . 100 200 200 . . . 200 . . . 25 × 100
0 1 2 ... 12 13 14 . . . 24 . . . 25 × 12

• The top and 3rd row are easily describable. The 4th row requires a
new technique.

• Our equation of value is

Timeline of X row = Timeline of 2900 row+Timeline of 100 row

• We now analyze the EOV of each row separately.

8.25 Top row


• Timeline of top row = Xa25×12 j

• j is interest rate per month


106 CHAPTER 8. MISCELLANEOUS ANNUITIES

8.26 2900 row


• Timeline 2900 row = 2900a25×12 j

• j is interest rate per month

8.27 100 Row - New Technique


• Timeline Trick

100 100 100 ... 100 =


... 100s12 j
0 1 2 3 ... 12

• Timeline Trick

200 200 200 ... 200 =


... 200s12 j
0 1 2 3 ... 12

• Do you see pattern?

• At t = 12, the end of the first year, we have an equivalent value of


100s12 j

• From t = 13 to t = 24, the end of the second year, we have an equivalent


value of 2 × 100s12 j

• From t = 25 to t = 36, the end of the third year, we have an equivalent


value of 3 × 100s12 j

• So  
Timeline 100 row = 100s12 j Ia
25

8.28 Interest Conversion, j


• j is interest rate per month

• So (1 + j)12 = 1 + i = 1.04
8.29. PUTTING IT ALL TOGETHER 107

8.29 Putting it all together


• Recall Equation of Value

Timeline X row = Timeline 2900 row + Timeline 100 row.

• So,  
2900a25×12 j + 100s12 j Ia = Xa25×12 j
25 i
108 CHAPTER 8. MISCELLANEOUS ANNUITIES
Chapter 9

Amortization

9.1 Overview-Major Topics


• In this chapter 9 we introduce and cover the following 5 major topics

• I) Amortization Table

• II) The 14 payment formulae - 5 general, 5 level, 4 total

• III) Calculator Amortization Spreadsheet

• IV) Sinking funds

• V) SOA Exam Problems

9.2 Overview - Chapter 10


• In chapters 10-12 we do problems

• These problems introduce and cover the following minor topics

• VI) Payment period 6= Interest period (Chapter 11)

• VII) Reinvestment (Chapter 12)

• IX) Non Level payments (Geometric, arithmetic) (Chapter 10)

• X) Total Payments (Chapter 10)

109
110 CHAPTER 9. AMORTIZATION

9.3 I) Amortization Table


• Let us begin with an example

• English You can pay back a loan of 10000 at 5 percent by paying


2820.12 at the end of each year for 4 years

• Formula Ra4 0.05 = 10000 =⇒ R = 2820.12

• Calculator:

4 5 − 10000 CPT 2820.12 0


N I/Y PV PMT FV

9.4 Symbols used


• R, periodic payment

• I, Interest amount

• P, Principle amount

• OLB, Outstanding Loan Balance

• t, time t = 0, 1, 2 . . .

9.5 Amortization Table


• Amortization Table:
R It = i × OLBt−1 P t = R t − It OLBt = OLBt−1 − Pt t
10000 0
R = 2820.12 500 = 0.05 × 10000 2820.12 − 500 = 2320.12 10000 − 2320.12 = 7679.88 1
R = 2820.12 383.99 = 0.05 × 7679.88 2820.12 − 383.99 = 2436.12 7679.88 − 2436.12 = 5243.76 2
.. .. .. .. ..
. . . . .
2820.12 ... ... 0 4

9.6 Why - What does table mean?


• Idea: Separation of interest and principle

• For example, suppose you want to pay off loan at t = 1


9.7. II) THE 14 PAYMENT FORMULAE - 5 GENERAL, 5 LEVEL, 4 TOTAL 111

• You have paid 2820.12. Maybe you owe 10000-2820.12.

• Amortization table says you have paid off only 2320.12

• You owe 7679.88. You can pay that off at t = 1 and owe nothing more

• The other 500 is interest you paid for the right to loan

9.7 II) The 14 payment formulae - 5 general, 5


level, 4 total
• Every amortization question can be solved using 14 formulae

• 4 of these formulae deal with total (raw) payments

• 5 of these formulae are valid when the periodic payment, R, is constant

• If R1 = R2 = R3 = . . . Rn we say payments are level or payments are


constant

• The remaining 5 formulae are general formulae valid whether R is


constant or not

9.8 The 5 general formulae


• The following formulae are general formulae whether R is constant or
not

(OLB)0 = L (9.1)
(OLB)n = 0 (9.2)
It = i × (OLB)t−1 (9.3)
Rt = It + Pt (9.4)
(OLB)t = (OLB)t−1 − Pt (9.5)
112 CHAPTER 9. AMORTIZATION

9.9 5 Formulae for Level Payments



(OLB)t =Ran−t , (9.6)
(OLB)0 =Ran =L ,
(OLB)1 =Ran−1 , . . .
It =R(1 − v n−(t−1) ), (9.7)
1− vn
I1 =i(OLB)0 = iRan = iR = R(1 − v n )
i
Pt =Rv n+1−t , (9.8)
R = I1 + P1 =⇒ P1 = R − R(1 − v n ) = Rv n
(OLB)t = Ran−t , Prospective method, for level payments only
(9.9)
(OLB)t = L(1 + i)t − Rst , Retrospective method, for level paymen
(9.10)
Important: For level payments,Pt forms a geometric series

9.10 4 Formulae for Totals



Total payments: nR (9.11)
Total principle paid: Rv n + Rv n−1 + . . . + Rv 1 = Ran
(9.12)
 
n n−1 n−(m−1)
Total principle t = 1 . . . m: Rv + Rv + . . . + Rv = R an − an−m
(9.13)
Total interest paid: nR − Ran = nR − L (9.14)

• In the amortization context we ask for total payments by summing


without discount factors
• Throughout course: Total means total present value
• But by amortization: total means total raw sum
• Discussion on why.
9.11. III) CALCULATOR AMORTIZATION SPREADSHEET 113

9.11 III) Calculator Amortization Spreadsheet


• The Calculator Worksheet is actually a spreadsheet

• To use it we must first use the TV spreadsheet

• Calculator:

4 5 − 10000 CPT 2820.12 0


N I/Y PV PMT FV

• Next enter Amortization worksheet

• This is found on the calculator at row 3 column 3 2nd key

• Navigate with Arrows; enter with ENTER key

9.12 Reading the Spreadsheet


• The Amortization worksheet presents the Amortization table presented
above in slide 9.5

• Suppose I want to read Row 1 of the Amortization table

• I identify Row 1 using the P1 and P2 keys

• Roughly speaking, P1 refers to the beginning of the 1st row and

• P2 refers to the end of the 1st row

• If I wanted to read row 1, I set P 1 = 1, P 2 = 1, using the ENTER key.

• I then read the window of the spreadsheet from row 1 to row 1

• The arrow keys now allow me to see I1 , P1 , (OLB)1 .

9.13 Row 2
• Simply set P 1 = 2, P 2 = 2.

• I am now reading the window from row 2 to row 2

• By using the Arrow keys I can read I2 , P2 , (OLB)2 .


114 CHAPTER 9. AMORTIZATION

9.14 Rows 1 and 2


• Suppose I ENTER P 1 = 1, P 2 = 1.

• So I am reading the window of the spreadsheet from row 1 to row 2.

• I can still read three items using the Arrow keys

• The OLB gives me (OLB)2 .

• The I gives me total interest, I1 + I2 during this period

• The P gives me total principle paid, P1 + P2 , during this period

9.15 IV) Sinking Fund


• Idea: At time t = 0 I lend L from lender

• But I don’t make periodic payments to the lender

• Rather I make payments say to a bank earning interest iSF

• At the end of n years I accumulate L in the bank account and give it


to the lender

• The name sinking fund refers to the fact that I sink money into the
bank until L is accumulated

• At the end of every year I pay the lender iL × L

• Notice that there are two interest rates, iL and iSF

• Also notice that there are two payees: The lender and the bank into
which I sink money

• We use the same example to illustrate Sinking fund as we did to illus-


trate amortization
9.16. TERMS USED 115

9.16 Terms used


• Interest on Loan iL × L

• SF Deposit Ds4 iSF =L

• SF Interest It = i × (SF )t−1

• SF Amount (SF )t = (SF )t−1 + It + Dt

• SF Amount is the same as SF Balance

• Outstanding Loan Balance (OLB)t = L − (SF )t .

• Total payment to sinking fund iL × L + D

9.17 Set up
• L = 10000, iL = 0.10, iSF = 0.05

• Ds4 0.05 = L = 10000 =⇒ D = 2320.12, the sinking fund payment

• Therefore, interest on loan is = iL × L = 0.10 × 10000 = 1000 at the


end of each year.

• Total payment equals interest payment plus sinking fund payment, =


D + iL × L = 3320.12

9.18 Sinking Fund Table


• Sinking Fund Table:

SF D SF I SF B OLB t
10000 0
D = 2320.12 0 = 0.05 × 0 2320.12 + 0 = 2320.12 = Ds1 10000 − 2320.12 = 7679.88 1
D = 2320.12 116 = 0.05 × 2320.12 2320.12 + 116 + 2320.12 = 4756.24 = Ds2 10000 − 4756.24 = 5243.76 2
.. .. .. .. ..
. . . . .
2320.12 ... 10000 0 4
116 CHAPTER 9. AMORTIZATION

9.19 V) SOA Exam Problems


• M00#26 Refinance

• M05#8 Refinance

• N00#34 Refinance

• M00#24 Refinance Deferral

• Q-IT#64 Refinance Last-payment

• Q-IT#75 Refinance

• Q-IT#84 Refinance-Last Payment

• Q-IT#88 Refinance

• M00#39 Reinvest

• Q-IT#89-Outstanding problem - Loan Refinance Verbal

• M05#2 Sinking Fund

• N00#48 Sinking Fund

• M01#4 Sinking-fund Amortization

• M03#15 Sinking-fund Amortization

• Q-IT#4 Sinking-fund Amortization

• Q-IT#87 Sinking Fund - 2 interest rates

• Q-IT#80

• M01#7 4 Total Formulae

• N00#55 4 Total Formulae


9.19. V) SOA EXAM PROBLEMS 117

• N01#6 4 Total Formulae

• M01#14 5 Level Formulae

• M01#37 5 Level Formulae

• N05#18 5 Level Formulae

• M03#39 5 Level Formulae - 5 General Formulae

• N00#12 5 Level formulae - 3 Total Formulae

• N01#9 Inflation- 5 General Formulae

• QIT#63 5 Level Formulae

• QIT#81 5 Level Formulae-5 General Formulae

• QIT#86 5 Level Formula-5 General Formulae

• Q-IT#15 Duplicate

• Q-IT#16 Duplicate

• Q-IT#24 Duplicate

• Q-IT#26 Duplicate

• Q-IT#28 Duplicate

• Q-IT#46 Duplciate
118 CHAPTER 9. AMORTIZATION
Chapter 10

Amortization Problems

10.1 Overview
• Today we review problems

• We have 3 goals

• I) problem solving methods

• We will look at the following problems which illustrate the following


methods

• II) QIT#9 illustrating the subgoal of non-level payments

• III) QIT#26 illustrating the subgoal of total payments

• These goals were mentioned in Chapter 9, slide 9.2, goals VIII, IX

10.2 I) Problem Solving Approach


• #1) a) Read problem and identify the b) variables and the c) cells,
that is the column and row

• #2) Fill in cells you can see

• #3) Do not jump to abstraction; First do 3-4 examples. Examples are


important

• #4) Look for patterns (Use algebra not numbers)

• #5) Break up into several subproblems

119
120 CHAPTER 10. AMORTIZATION PROBLEMS

Table 10.1: QIT#9: Spreadsheet solution, 1st 10 rows

t R I P OLB
0 0 0 0 1000
1 150 100 50 950
2 142.5 95 47.5 902.5
3 135.38 90.25 45.12 857.38
4 128.61 85.74 42.87 814.51
5 122.18 81.45 40.73 773.78
6 116.07 77.38 38.69 735.09
7 110.26 73.51 36.75 698.34
8 104.75 69.83 34.92 663.42
9 99.51 66.34 33.17 630.25
10 94.54 63.02 31.51 598.74

• In the next few slides we will see how this problem solving strategy is
implemented.

10.3 II) QIT#9 illustrating the subgoal of non-


level payments
• Please read QIT#9 in the problem resources

• The speadsheet solution is found in Table 10.1 and Table 10.2

• We broke the spreadsheet into two linked spreadsheets

10.4 Problem to Cells


• Step #1 in slide 10.2 says to a) read problem, identify b) variables,
and c) cells.

• Let us apply this to QIT#9

• A) 20 year loan −→ n = 20

• B) 20 year loan of 1000 −→ L = OLB0 = 1000


10.5. USE OF 13 BASIC FORMULAE 121

Table 10.2: QIT#9: Spreadsheet solution, 2nd 10 rows

tnew told R I P OLB


0 0 0 0 0 598.74
1 11 97.44 59.87 37.57 561.17
2 12 97.44 56.12 41.32 519.84
3 13 97.44 51.98 45.46 474.39
4 14 97.44 47.44 50 424.38
5 15 97.44 42.44 55 369.38
6 16 97.44 36.94 60.5 308.88
7 17 97.44 30.89 66.55 242.32
8 18 97.44 24.23 73.21 169.11
9 19 97.44 16.91 80.53 88.58
10 20 97.44 8.86 88.58 0

• C) Payments are 150% of interest due −→ Ri = 1.5Ii , i = 1, 2, 3, . . . , 10

• D) The last 10 payments are X −→ Ri = X, i = 11, 12, 13, . . . , 20

10.5 Use of 13 Basic Formulae


• Step #2,#3 of slide 10.2: Fill in cells; if necesssary do 3-4 examples
first.

• Use A on last slide to label −→ t column

• Use B on last slide to get −→ OLB0

• Use C on last slide to fill in row 1: −→ Fill the row in this order,
I1 , R1 , P1 , OLB1 (Use 13 basic formulae)

• Use C on last slide to fill in row 2: −→


Fill the row in in this order, I2 , R2 , P2 , OLB2 (Fill using the 14 basic
formulae)

• Use C on the last slide to fill in row 3: −→


Fill the row this order, I3 , R3 , P3 , OLB3 (Use 14 basic formulae)
122 CHAPTER 10. AMORTIZATION PROBLEMS

10.6 Search for Algebraic Patterns


• Step #4 in slide 10.2

• We just filled in numbers on the spreadsheets for the first 3 rows

• Now with the help of algebra we will obtain important formula

• I1 = 10%L

• R1 = 150%I1 = 150%10%L = 15%L

• P1 = R1 − I1 = 15%L − 10%L = 5%L

• OLB1 = OLB0 − P1 = L − 5%L = 95%L

10.7 Do 2-3 examples for Rows 2 and 3


• Same argument (write it out!) gives

• OLB2 = 95%2 L

• And then the same argument gives OLB3 = 95%3 L

10.8 Recognize Pattern


• Do you see the pattern?

• We can infer OLB10 = 95%10 L

• If you didn’t see pattern you would have to do all 10 rows

• You could still solve the problem but it would take longer

10.9 2nd Part of Problem


• Step #5:

• Think of last 10 rows as new problem

• Old times 11, 12, 13, , 20 becomes New times 1,2,3,. . . , 10


Original Times
• OLB10 = New Loan Amount = OLB0New Times in new count-
ing
10.10. III) QIT#26 ILLUSTRATING THE SUBGOAL OF TOTAL PAYMENTS123

Table 10.3: QIT#9: TV calculator line for 2nd 10 rows.

T V Calculator 0 0 97.44 0
10 10 OLB10 CP T 0
N I PV PMT FV

• So we now have a loan repayment problem with level payments

• This can be solved using the TV calculator timeline (See table 10.3)

10.10 III) QIT#26 illustrating the subgoal of total


payments
• We now do QIT#26

• Please read it

• This problem naturally breaks itself up into 3 subproblems

• We will name the 3 subproblems by the people involved

10.11 Lori
• Lori repays her loan with 10 level payments at the end of every six-
month period.

• This is the Level payment method

• Solve, using TV

10.12 Interest: Calculator


• But how do you compute total interest

• Method #1: Enter amortization sheet

• P1 = 1, P2 = 10 −→ INT gives total interest


124 CHAPTER 10. AMORTIZATION PROBLEMS

Table 10.4: QIT#26 - Lori’s approach


P ERSON t R I P OLB
Lori 0 0 0 0 5000
Lori 1 679.34 300 379.34 4620.66
Lori 2 679.34 277.24 402.1 4218.56
Lori 3 679.34 253.11 426.23 3792.33
Lori 4 679.34 227.54 451.8 3340.53
Lori 5 679.34 200.43 478.91 2861.63
Lori 6 679.34 171.7 507.64 2353.98
Lori 7 679.34 141.24 538.1 1815.88
Lori 8 679.34 108.95 570.39 1245.5
Lori 9 679.34 74.73 604.61 640.89
Lori 10 679.34 38.45 640.89 0
0 0 0 0 0 0
0 0 6793.4 1793.4 5000 0
Solution 0 0 T OT AL Int 0 0

10.13 Interest: Formulae


• Method #2: Level payments −→ Use level payment formulae

• Review: Total payment: 10R

• Total principle:L = Ra10 =Loan amount

• Total Interest: Payment - Principle

• So Total Int: 10R − Ra10 = 10R − L

• See Table 10.4

10.14 Janice
• Janice pays interest due every period

• Janice makes one principle payment at end


10.15. 5 GENERAL FORMULAE 125

10.15 5 General Formulae


• But what is interest due

• Look up 5 General Formulae

• It = I × OLBt−1

10.16 5 General Formulae applied to Row 1


• I1 = 6% × 5000 = 300

• But then P1 = 0 (Why?)

• So R1 = I1 + P1 = I1 = 300

• OLB1 = OLB0 − P1 = OLB0 = 5000

10.17 5 General Formulae applied to Row 2


• So

• I2 = 6% × 5000 = 300

• P2 = 0 (Why?)

• R2 = I2

• OLB2 = OLB1 − P2 = 5000

10.18 The Pattern


• Do you see the pattern?

• It = 6% × 5000 = 300 for t = 0 through 9

• OLBt = 5000 for t = 0 through 9


126 CHAPTER 10. AMORTIZATION PROBLEMS

Table 10.5: QIT#26 - Janice’s Approach


P ERSON t R I P OLB
Janice 0 0 0 0 5000
Janice 1 300 300 0 5000
Janice 2 300 300 0 5000
Janice 3 300 300 0 5000
Janice 4 300 300 0 5000
Janice 5 300 300 0 5000
Janice 6 300 300 0 5000
Janice 7 300 300 0 5000
Janice 8 300 300 0 5000
Janice 9 300 300 0 5000
Janice 10 5300 300 5000 0
0 0 0 0 0 0
T otalInterest 0 0 3000 0 0

10.19 Compute Total interest


• So total interest (in this special case)

• 300 + 300 + 300 + 300 + . . . + 300 = 10 × 300 = 3000

• See Table 10.5

10.20 Seth
• Seth lets interset accumulate over 5 years

• Pays interest and principle in

• One lump sum at end

10.21 Chapter 2
• How do you do this

• Think chapter 2
10.22. CHAPTER 2 - INTEREST 127

Table 10.6: QIT#26 - Seth’s Approach


P ERSON t R I P OLB
Seth 0 0 0 0 5000
Seth 1 0 0 0 5300
Seth 2 0 0 0 5618
Seth 3 0 0 0 5955.08
Seth 4 0 0 0 6312.38
Seth 5 0 0 0 6691.13
Seth 6 0 0 0 7092.6
Seth 7 0 0 0 7518.15
Seth 8 0 0 0 7969.24
Seth 9 0 0 0 8447.39
Seth 10 0 0 0 8954.24
0 0 0 0 0 0
8954.24 - 5000 3954.24 0 0
A(n) - A(0) T otalInt 0 0

• A(10) = A(0)(1 + i)10

10.22 Chapter 2 - Interest


• But what is total interest

• Again: Think Chapter 2

• I = A(Endpoint) − A(Beginning point)

• I = A(10) − A(0) = 5000(1.1)10 − 5000 = 3954.24

• See Table 10.6


128 CHAPTER 10. AMORTIZATION PROBLEMS
Chapter 11

Bonds

11.1 Overview
• We have 8 subgoals to meet in this chapter

• I) What are bonds?

• II) Bond EOV and Timelines

• III) Conversions

• IV) SOA Exam Problems

• V) NO5#4 4 step problem approach

• VI) More Bond symbols and more formulae

• VII) Amortization Table of bonds

• VIII) QIT#10 M03#42 8-Bond-Formulae

11.2 I) What are bonds?


• Suppose I want to start, for example, a pen business

• I need a million dollars to start the business

• With the million dollars I buy a factory, pen manufacturering equip-


ment, etc.

129
130 CHAPTER 11. BONDS

• Once the business is started the profits will sustain the business

• How do I raise the million

• I float/sell 1000 thousand dollar bonds giving me one million

11.3 Bond definition


• A bond is basically an IOU

• You pay me $1,000 now and the IOU says I pay you back $1,000 in 5
or 10 years

• I might pay you more (e.g. $1100 or $1200)

11.4 Bond terms


• The $1000 you pay me is called the Price, P

• The time at which I pay you back - 5 or 10 years - is called the


maturity date

• The amount I pay you at maturity is called the redemption value, C

• The symbol C indicates how much the bond is called for at maturity.

11.5 Coupon bonds


• In the last slide I have described a 0-coupon bond: P, C

• But I might decide to give you coupons, monetary payments, period-


ically

• That way you don’t have to wait to maturity to profit

• The coupon amount equals F × r where

• F is the face amount of the bond and r is the coupon rate

• The sole/only purpose of F and r is to obtain the coupon amount


(Spooky!)
11.6. C VS F VS P 131

11.6 C vs F vs P
• P is what you pay me at time t = 0.

• C is what I pay you at time t = n, the maturity date

• Fr, the coupon amount is what I pay you at the end of each period.

• If the problem gives you P,F,C then they are all different

• If the bond is called a par value bond and nothing else is said then
C = F.

• But the problem might indicate that C and F are different

• In such a case F is called the par value of the bond; C is the redemption
value.

• So straightening out C vs. F can be confusing.

• Remember: English description is one of the Rule of 6

11.7 II) Bond EOV and Timelines


• Bond Timeline

C
−P Fr Fr ... Fr
0 1 2 ... n

• To apply the TV line we need to use all 5 calculator keys since there is
a balloon payment

• Remember the rule: balloon payment goes to FV while periodic pay-


ment goes to PMT

• Calculator TV Line:

n 100i −P Fr C
N I/Y PV PMT FV
132 CHAPTER 11. BONDS

• If you enter any 4 of these you can compute the 5th

• Note: i is not r. The sole purpose of r is to compute the coupons

• i is the overall yield. The equation of value, EOV is


P = F ran i + Cvin . (11.1)

11.8 III) Conversions


• A final word about payment periods vs. interest periods

• Our approach in this course is always to convert first

• The coupon rate is perceived as a nominal rate

• So if the coupons are paid twice a year then


r
• you replace r with 2

• you replace n years with 2n

• and when you compute i you compute the annual rate, j by


1 + j = (1 + i)2 .

11.9 IV) SOA Exam Problems


• N05#4 Plug in (Basic Formula)

• N05#24 Plug in (Basic Formula)

• QIT#09 Plug in (Basic Formula)

• M00#29 Refinancing

• M01#41 Reinvestment

• N05#11 Reinvestment

• N05#16 Reinvestment
11.9. IV) SOA EXAM PROBLEMS 133

• M03#42 8 Formulae

• QIT#62 8 Formulae

• M05#10 Forward / Spot rates

• N05#6 Forward / Spot rates

• QIT#33 Forward / Spot rates

• QIT#34 Forward / Spot Rates

• N01#31 Comparison

• M05#5 Comparison

• QIT#74 Comparison

• QIT#76 Comparison

• N05#19 Trivia

• QIT#50 Bond pricing between coupons

• N05#22 Callable

• M05#11 Callable

• QIT#54 Callable

• QIT#55 Callable

• QIT#56 Callable

• QIT#57 Callable
134 CHAPTER 11. BONDS

• QIT#91 Callable

• QIT#10 Duplicate

• QIT#22 Duplicate

• QIT#47 Duplicate

11.10 V) NO5#4 4 step problem approach


• This is a plug in problem and in my opinion way to easy. But it did
occur on an exam.

• Problem Statement: A ten year 100 par value bond pays 8% coupons
seminannually. The bond is priced at 118.20 to yield an annual nom-
inal rate of 6% convertible semiannually. Calculate the redemption
value of the bond.

• In performing the solution we use four steps . . .

11.11 Four steps of the problem


• 1) We use the English to identify P, i, r, n, F, C.

• 2) We set up the actual timeline

• 3) We then set up the EOV,

• and 4) Set up the calculator timeline

11.12 N05#4 Solution Step 1: Variable values


• 100 par value → F = 100.

• semiannually → n = 2 × 10 = 20.
.08
• semiannually → r = 2 = 0.04.

• P = 118.20
11.13. N05#4 SOLUTION STEP 2: TIMELINE 135

.06
• i= 2 = 0.03.

• C is unknown

• Note: Why doesn’t C = F ?

• Because the problem asks for C and therefore it is assumed that F


and C are different

• I however consider this confusing!

• To communicate the difference of C and F . . .

• I would have phrased the problem differently:

• A ten year bond with a par-value of 8%

11.13 N05#4 Solution Step 2: Timeline


• Timeline

C
−118.20 100 × 0.04 = 4 4 ... 4
0 1 2 ... 20

11.14 N05#4 Solution Step 3: EOV


• Equation of Value, EOV 118.20 = 0.04 × 100a20 20 .
+ Cv.03
.03

11.15 N05#4 Solution Step 4: Calculator Time-


line
• Calculator:

20 3 − 118.20 100 × .04 = 4 CPT FV = 106


N I/Y PV PMT FV
136 CHAPTER 11. BONDS

11.16 VI) More Bond symbols and more formulae


• Modified coupon rate, g is defined by

Cg = F r. (11.2)

• You use this rate if you are given C but not F

• Base rate, G, is defined by

Gi = F r. (11.3)

• So you have three ways to give the coupon: F r, Cg, Gi.

• Or, you could just state the coupon amount.

11.17 Three basic formulae



P = F ran i + Cvin (11.4)

• P = Cgan i + Cvin = Cgan i + C(1 − ian )

• So
P = C(g − i)an i + C (11.5)

• Try and use the first formula, called the basic formula whenever pos-
sible

11.18 VII) Amortization Table of bonds


• The Bond amortization schedule is almost identical to the Loan Amor-
tization Schedule

• You set up a spreadsheet with 5 columns: t, Rt , It , Pt , OLBt .

• The five formulae for non-level payments are almost the same

• i) OLB0 = P, ii)It = i × OLBt−1 , iii)It + Pt = Rt , iv)OLBt−1 − Pt =


OLBt , v)OLBn = C
11.19. THE 5 BASIC BOND FORMULAE WHEN COUPONS ARE LEVEL 137

• There are two key differences

• #1) Rt = F r is the coupon rate (This belongs in the next section, the
formulae for level coupon rates)

• #2) OLBn is equal to C (for Bonds) but to 0 (for amortized loans)

• Refinancing and reinvestment follow the same rules for Loan amorti-
zations

• OLBt is call BVt the Book Value of the bond

• BVt is the price on the books; what someone would pay at t for the
bond

• The BVt is present value at t of all future coupons and the redemption
value

11.19 The 5 Basic Bond Formulae When Coupons


are level
• i)coupons = F r = Cg

• Prospective Formulaii) OLBt = F ran−t i + Cv n−t = C + C(g −


i)an−t i

• iii) Retrospective formula complicated because of coupons and is there-


fore omitted

• iv) Pt = C(g − i)v n+1−t

• v) The formula for It is derived from the formula for Pt (just given)
and the relation It + Pt = Rt = F r = Cg

• Since this is complicated we omit it also (So there are only 3 formulae
for the level case)

• So there are a total of 8 Bond formulae (5 for any case and 3 for the
level case)
138 CHAPTER 11. BONDS

11.20 VIII) QIT#10 M03#42 8-Bond-Formulae


• Problem Statement A 10,000 par value 10-year bond with 8% an-
nual coupons is bought at a premium to yield an annual effective rate
of 6%. Calculate the interest portion in the 7th coupon.

11.21 Step 1: English to Variables


• par value → C = F = 10000.

• n = 10, no conversions!

• r = .08

• i = 0.06

• P, price is unknown

11.22 8-Bond-Formulae
• We present two solutions, both based on the 8 Bond Formulae

• In this slide we present the 1st solution

• Want interest portion in 7th payment

• How do you compute this? How do you approach this?

• You must review the 8 formulae

• Which of 8 formula talks about It

• Aha! I7 = i × OLB6

• Now which formula will you use for OLB

• Aha! OLB6 = F ra4 4 .


+ Cv.06
0.06

• The rest is computation since all variables on the right side are known.

• OLB6 = 10693.02, I7 = 0.06 × OLB6 = 641.58.


11.23. THE SECOND SOLUTION 139

11.23 The Second Solution


• In this slide we present the 2nd solution

• We still want interest portion in 7th payment

• How do you compute this? How do you approach this?

• You must still review the 8 formulae

• Which of 8 formula talks about It

• Aha! Rt = It + Pt

• But Rt = F r = 10000 ∗ 8% = 800

• And Pt = C(g − i)v n+1−t → P7 = 10000(8% − 6%)v0.06


4 = 158.42

• Hence I7 = 800 − 158.42 = 641.58

• Note that F r = Cg → r = g

• This is true in general whenever C = F


140 CHAPTER 11. BONDS
Chapter 12

Reinvestment

12.1 Overview
• We have one main goal for today: Reinvestment problems

• Although the SOA has eliminated Replacement of Capital from the


SOA syllabus

• Reinvestment itself may still be on it

• In fact reinvestment occurs in certain sections covered by the syllabus

• I am therefore retaining this topic in the notes

• It is also an excellent topic to learn to deal with multiple timelines

• We will cover the following 5 subgoals in this lecture

• I) What is a reinvestment problem

• II) Solution method: Distinct timeline for each interest rate

• III) Model Problem: M01#41

• IV) Model Problem: N05#11

• V) Quiz problem: N05#16

141
142 CHAPTER 12. REINVESTMENT

12.2 I) What is a reinvestment problem


• Typical problem - a) loan with pay back or b) buy bond and get
redemption

• But then reinvest say coupons in a different bank, different rates

• A typical problem may have 2-3 banks (2-3 investment rates)

• You may be asked to find overall yield

12.3 II) Solution method: Distinct timeline for


each interest rate
• Each investment rate component gets its own timeline, its own EOV

• The overall yield is determined using Chapter 2 methods

• What did you pay at time 0, A(0),

• What did you receive at time n, A(n),

• Relate the two quantities with the fundamental equation: A(0)(1 +


i)n = A(n).

12.4 III) Model Problem: M01#41


• Bill buys a 10-year 1000 par value 6% bond with semi-annual coupons.
The price assumes a nominal yield of 6%, compounded semi-annually.

• As Bill receives each coupon payment, he immediately puts the money


into an account earning interest at an annual effective rate of i .

• At the end of 10 years, immediately after Bill receives the final coupon
payment and the redemption value of the bond, Bill has earned an
annual effective yield of 7% on his investment in the bond.

• Calculate i
12.5. SOLUTION - COUNT BANKS / INTEREST RATES 143

12.5 Solution - Count banks / interest rates


• (Timeline #1) 10 year bond is at a 3% rate

• (Timeline #2) Coupons are in a bank earning i

• Bill pays P, price of bond and receives

• the redemption value from the 3% rate bank

• and the accumulated coupons from the i bank

• So 3rd timeline relates P vs. redemption and accumulated coupons at


7% rate

12.6 Timeline #1
• Timeline #1

1000
−P 3% × 1000 = 30 30 ... 30
0 1 2 ... 20

• Note: Semiannual for 10 years −→ n = 20.

• 6% semiannual coupons −→ 3% per half year

• EOV: P = 30a20 20 .
+ 1000v3%
3%

• Calculator Timeline #1

N I PV PMT FV
020 3 CPT 30 1000

• P V = 1000.

12.7 When r = i
• In the last slide we saw an example where

• C=F

• r=i
144 CHAPTER 12. REINVESTMENT

• P =C

• This is true in general

• If r = i then P = C

• We can derive this from the formula P = Cr%an r% + Cvrn

12.8 Timeline #2
• Timeline #2

A(20)
3% × 1000 = 30 30 ... 30
0 1 2 ... 20

• Note: i is rate per year; but we need j, rate per half year

• We relate them using factor, not nominal, approach: (1 + j)2 = 1 + i.

• EOV: A(20) = 30s20 :j

• Also possible to compute everything at t = 0.

• The calculator timeline approach does not work because there are too
many unknowns

• Calculator Timeline #2

N I PV PMT FV
20 CPT 0 −30 A(20)

12.9 Timeline #3
• Remember: This is a chapter 2 problem

• You must ask: what happened at t = 0 and what happened at t = n


12.10. IV) MODEL PROBLEM: N05#11 145

• Bond Timeline

C = 1000
A(20)
P
0 1 2 ... 20

• EOV: P (1.07)10 = 1000 + A(20)

• We can substitute numbers

• P = 1000 −→ 1000(1.07)10 = 1000 + 30s20 j

• Now we only have one unknown

• Calculator Timeline #2 revisited

N I PV PMT FV
20 CPT 0 −30 1000(1.07)10 − 1000

• So j = 4.7597

• Hence 1 + i = (1 + j)2 −→ i = 9.7459%

12.10 IV) Model Problem: N05#11


• An investor borrows an amount at an annual effective interest rate of
5% and will repay all interest and principal in a lump sum at the end
of 10 years.

• She uses the amount borrowed to purchase a 1000 par value 10-year
bond with 8% semiannual coupons bought to yield 6% convertible
semiannually.

• All coupon payments are reinvested at a nominal rate of 4% convertible


semiannually.

• Calculate the net gain to the investor at the end of 10 years after the
loan is repaid.
146 CHAPTER 12. REINVESTMENT

12.11 Four investment rates


• The 5% loan

• The 3% bond yield

• The 2% coupon bank

• Chapter 2- Compare proceeds of the other three banks / investment


rates

12.12 The 5% loan


• A(0) = P − − − − − − − − − − − − − −A(10) = P (1.05)10

12.13 The 3% bond


• Bond Timeline

... C = 1000
4% × 1000 40 ... 40
p
0 1 2 ... 20

• EOV: P = 40a20 20 −→ P = 1148.77


+ 1000v3%
3%

• Note: the 6% nominal semiannual rate converts to 3% per period (half


year)

• Note: the 8% coupon semiannual rate converts to 4% per period (half


year)

12.14 The 2% coupon bank


• Bond Timeline

4% × 1000 40 ... 40
A(20)
0 1 2 ... 20
12.15. THE FINAL SUMMARY LINE 147

• EOV: A(20) = 40s20 2% −→ A(20) = 971.89

• Note: the 4% nominal semiannual rate converts to 2% per period (half


year)

12.15 The Final Summary line


• Normally, investment problems ask for the overall yield

• For that we construct a Chapter 2 timeline

• But in this problem they ask for the net gain

• That is: The problem does not want the annual effective percentage
rate,

• But rather wants the total dollar amount gained

12.16 Factors for Computation of Gain


• How do we compute gain

• Well first: We do not care about t = 0.

• Why? Because the person loaned P and then spent P on the bond

• So at t = 0 the person spent nothing

• All we care about is the overall gain at time t = n; What was spent
and earned?

12.17 Tally of spent and gains


• We separately tally the loan, the bond and the coupon timelines

• The Loan: The investor must pay back (loss) at t = n,

• the loan amount with interest, 1871.23.

• The Bond: The investor receives at t = n, the redemption amount,


1000.

• The coupon bank: The investor receives at t = n, A(20) = 971.89.


148 CHAPTER 12. REINVESTMENT

• Total received: -1871.23+1000+971.89 = 100.66

12.18 V) Quiz problem: N05#16


• Dan purchases a 1000 par value 10-year bond with 9% semiannual
coupons for 925.

• He is able to reinvest his coupon payments at a nominal rate of 7%


convertible semiannually.

• Calculate his nominal annual yield rate convertible semiannually over


the ten-year period.

12.19 Hints
• How many distinct investment rates / banks are there?

• Did you make a separate Chapter 2 line (t = 0, t = n) for overall yield


if appropriate?
Chapter 13

Stocks, Margin, Spot,


Forward, Term Structure

13.1 Overview
• We have 8 brief subgoals in todays chapter

• I) Stocks - What are they

• II) Stocks - Dividend discount model

• III) Short sales of Stocks and Margin requirements

• IV) Good exam problems on Margin

• V) Spot rates, strips and yield curve

• VI) Good exam problems on spot and forward rates, yield, and term
structure

• VII) Spot rates and implied forward rates

• VIII) M05-#10, a good exam problem on forward rates

13.2 I) Stocks - What are they


• Say I want to create the Dr. Hendel Pen company

149
150CHAPTER 13. STOCKS, MARGIN, SPOT, FORWARD, TERM STRUCTURE

• I need a million dollars to create the company


• Then I would sell many pens and earn money
• Bonds - loaning money - are one approach to obtaining million dollars
• Stocks are another approach
• #1) Each share of stock represents co-ownership in the company
• Stock are different than bonds since stock holders co-own the
company.
• If a stock holder owns enough shares (s)he can determine policy
• #2) Also share-owners (stock holders) may get percentages of
profits
• These period profit payments are called dividends

13.3 II) Stocks - Dividend discount model


• I could not find any problems in the exams on the Dividend Discount
model
• The issue is how to price stocks
• In the real world many factors influence price - for example what other
investors think the stock will do

13.4 Stocks - Dividend Discount Model


• The Dividend Discount model says that the value of the stock is the
PV of all dividends, D

P = Da∞ i% (13.1)

• This formula can be modified to reflect an inflation factor


• This formula can also be modified to reflect non-level dividend amounts
13.5. III) SHORT SALES OF STOCKS AND MARGIN REQUIREMENTS151

13.5 III) Short sales of Stocks and Margin re-


quirements
• When you sell long the buyer
• pays P at t = 0
• receives the stock bought at t = 0
• When you sell short you
• receive P at t = 0 for a stock you don’t own!
• but you agree to give the buyer the stock say at t = 1
• Alternate formulation: You receive P at t = 0
• You fictitiously borrow the stock you were suppose to sell
• and agree to pay back the stock say at t = 1
• Why sell short? Because you expect the stock to go down. You make
P at t = 0 but lose only the lower value of the stock at t = 1.

13.6 Yield on Short sale


• You receive P at t = 0
• You buy back the stock - that is you cover the short - at t = 1 for C
• You also have to give the buyer at t = 1 any dividends, D, earned
between t = 0 and t = 1.
• So your profit amount is P − C − D while the amount you spent is 0.
• If you spend 0 and roughly earn P − C − D, your yield is infinite.

13.7 Yield on Short sale - Continued


• This is not good: if people can earn money without spending
• This was the cause of the great depression
• Why? Suppose C > P
• Then you can’t cover the short sale - so you go bankrupt
152CHAPTER 13. STOCKS, MARGIN, SPOT, FORWARD, TERM STRUCTURE

13.8 Solution: Yield on Short sale - Margin


• To protect the economy we require depositing margin, M at t = 0

• You get interest on margin, I

• Typicallly margin and interest are expressed as percentages

• So your total gain is P − C − D + I at t = 1

• Your total expense at t = 0 is M

• Your yield is the ratio

Gains − Losses P −C −D+I


i= = . (13.2)
M M

• Note: Sometimes there are other losses / expenses such as commissions

• A commission is an amount you pay a broker, an agent of a stock firm,


to do sales and buys

• You can work any new expenses or gains into the basic formula above

• The problems below will illustrate all these principles

13.9 IV) Good exam problems on Margin


• One formula - all problems are the same

• M01#27 Margin

• M03#36 Margin

• N00#24 Margin

• M05#22 Margin
13.10. M01#27 153

13.10 M01#27
• Jose and Chris each sell a different stock short for the same price.

• For each investor, the margin requirement is 50% and

• Interest on the margin debt is paid at an annual effective rate of 6% .

• Each investor buys back his stock one year later at a price of 760 .

• The stock owned by Jose stock paid a dividend of 32 at the end of the
year while the stock owned by Chris paid no dividends.

• During the 1-year period, the return of Chris on the short sale is i,
which is twice the return earned by Jose.

• Calculate i

13.11 Solution - Jose


• At t = 0, Jose spends 50% × P = 50%P

• At t = 0, Jose receives P for selling short

• At t = 1, Jose covers short sale with 760

• At t = 1, Jose receives the following interest on margin, 6% × 50% × P.

• At t = 1, Jose must pay dividend of 32

• Total profit is P − 760 + 6% × 50% × P − 32

P −760+6%×50%×P −32
• Yield is iJ = 50%P

13.12 Solution - Chris


• Only difference between Yose and Chris is no dividend

P −760+6%×50%×P
• So yield is iC = 50%P
154CHAPTER 13. STOCKS, MARGIN, SPOT, FORWARD, TERM STRUCTURE

13.13 Finish Problem


• iC = 2iJ

• Plug in formulae for iJ and iC and solve

13.14 V) Spot rates, strips and yield curve


• Spot rates rn describe annual yields for n year bonds


1
1 (13.3)
(1 + rn )n
0 n (13.4)

• In words The price (PV) of an n-year zero coupon bond with redemp-
tion amount 1 is (1+r1n )n .

• So Prices of zero coupon n year bonds are an alternate method of


giving spot rates

13.15 Strips
• Given any bond you can sell the coupons and redemption separately

• The separate coupons and separate redemption are called strips

• You have stripped the bond of its coupons and are selling them sepa-
rately

13.16 Yield vs. Spot rates - Term Structure


• Let us suppose I have different spot rates for different years

• Each coupon sells for different prices

• So coupon rates are determined by spot rates

• Overall yield is determined by entire collection


13.17. VI) GOOD EXAM PROBLEMS ON SPOT AND FORWARD RATES, YIELD, AND TERM STRUCT

• We refer to the collection of spot rates as the term structure

• The graph with the x-axis giving interest rates and the y-axis giving
spot rates is called the yield curve

• A flat yield curve means that r1 = r2 = r3 . . . so that the slope of the


yield curve is 0.

13.17 VI) Good exam problems on spot and for-


ward rates, yield, and term structure
• One formula - all problems are the same

• QIT#33

• QIT#34

13.18 Q-IT#34
• You are given the following information with respect to a bond:

• Par amount: 1000

• Term to maturity: 3 years

• Annual coupon rate: 6% payable annually

• Term structure of spot rates:

Term Annual Spot Interest Rates


1 7%
2 8%
3 9%

Calculate the annual effective yield rate for the bond if the bond is
sold at a price equal to its [present] value.
156CHAPTER 13. STOCKS, MARGIN, SPOT, FORWARD, TERM STRUCTURE

13.19 Solution
• Coupon amount equals 1000 × 6% = 60
60
• Present value of 1-year strip of 60 equals 1.07
60
• Present value of 2-year strip of 60 equals 1.082
60
• Present value of 3-year strip of 60 equals 1.093
1000
• Present value of redemption strip of 1000 equals 1.093
60 60 1060
• Present value, price, of bond: 1.07 + 1.082
+ 1.093
=P
• Yield of bond: P = 60a3 i + 1000vi3 .
• SOA solutions suggest using TV Calculator Timeline
• Important: Notice how spot rates and yield are different.

13.20 VII) Spot rates and implied Forward rates


• Suppose spot rates for two different periods are different
• We say the spot rates imply corresponding forward rates
• The forward rate fi,j is the annual rate from t = i to t = j
• There is only one formula

(1 + ri )i (1 + fi,j )j−i = (1 + rj )j . (13.5)

13.21 Good problems - Spot, forward rates


• One formula - all problems are the same
• N05#15
• M05#10
• N05#19 (Problem illustrates English terms yield rates, risk-free yield
curve (term structure), forward rates,spot rates,strips)
13.22. VIII) M05-#10,A GOOD EXAM PROBLEM ON FORWARD RATES157

13.22 VIII) M05-#10,a good exam problem on


forward rates
• Yield rates to maturity for zero coupon bonds are currently quoted at
8.5% for one-year maturity, 9.5% for two-year maturity, and 10.5%
for three-year maturity. Let i be the one-year forward rate for year
two implied by current yields of these bonds. Calculate i.

13.23 Solution
• Use the one formula (??)

• (1 + ri )i (1 + fi,j )j−i = (1 + rj )j .

• Note: Year 2 is from t = 1 to t = 2

• So let i = 1, j = 2. Then

• (1 + r1 )(1 + f1,2 ) = (1 + r2 )2 .


1.085 × (1 + f1,2 ) = 1.0952 −→ 1 + f1,2 = 1.105.
158CHAPTER 13. STOCKS, MARGIN, SPOT, FORWARD, TERM STRUCTURE
Chapter 14

Callable Bonds

14.1 Overview
• We cover one topic today, Callable bonds

• We have the following 7 subtopics

• I) Definition: What are Callable bonds

• II) The difference between Premium vs. Discount

• III) A Good Motivating Example

• IV) The Basic Theorem

• V) Good Exam Problems

• VI) QIT#54

• VII) Helpful Summarizing Table

14.2 I) Definition: What are Callable bonds


• An ordinary bond is called or redeemed at t = n for the maturity value,
C

• The bond seller must redeem the bond at t = n

• The bond buyer must wait until t = n to redeem the bond

159
160 CHAPTER 14. CALLABLE BONDS

14.3 Definition: Callable bond - continued


• Contastively, a callable bond may be called earlier than t = n

• The decision on when to redeem the bond is the seller’s not the buyer’s

• If the seller does not redeem before t = n then he must redeem at


t=n

• The redemption value for earlier redemption times may equal C or be


different

14.4 The Three callable options


• A callable bond can be done in 3 ways:

• American, Bermuda or European option

• American option: Seller may redeem the bond at any t ≤ n.

• European option: Seller may only redeem the bond at one


extra date besides t = n : Say t = n1 < n.

• Bermuda option: Seller may only redeem the bond at a finite


set of dates, t = n1 , n2 , . . . , n

• Typically, Bermuda options are redeemable when coupons are being


paid

• Most examples in this course use a Bermuda option

14.5 The callable bond problem


• What is the yield?

• How do we compute yield if we don’t know the redemption date?


14.6. II) THE DIFFERENCE BETWEEN PREMIUM VS. DISCOUNT 161

14.6 II) The difference between Premium vs. Dis-


count
• In solving the callable bond problem we introduce one more concept

• A bond is bought at a discount if either P < C or g < i

• We call C − P the amount of discount

14.7 Premium - Discount continued


• A bond is bought at a premium if either P > C or g > i

• We call P − C the amount of premium

14.8 P − C vs. g − i
• The definition was formulated in terms of P, C or g, i

• Why are these two definitions the same?

• Note P = F ran i% + Cvin = Cgan i% + C(1 − ian i% )

• Hence P − C = C(g − i)an i%

• So P > C ↔ g > i.

• Also note: If the bond is a par value bond then g = r, (Because


Cg = F r), and g > i is equivalent to r > i.

14.9 More English conventions on Premium and


Discount
• The following English conventions are associated with the term Pre-
mium

• P >C

• The bond is bought at a premium

• The amount of premium in the purchase price of the bond is


P −C
162 CHAPTER 14. CALLABLE BONDS

• The amount of (amortization / accumulation of) premium in the


t-th coupon is Pt
• The amount of write down in the t-th coupon is Pt
• The following English conventions are associated with the term Dis-
count
• C>P
• The bond is bought at a discount
• The amount of discount in the purchase price of the bond is C −P
• The amount of (amortization / accumulation of) discount in the
t-th coupon is −Pt
• The amount of write up in the t-th coupon is −Pt

14.10 III) A Good Motivating Example


• Consider a 2 year bond with redemption value 5 and coupons of 1
yielding 10%.
• If the bond is not callable then the price is 5.8678
• In other words 5.8678 = 1 × a2 10% + 5v 2
• This is solved in a traditional manner with calculator

14.11 Motivating Example redeemed at t = 1


• Suppose the seller had the option to redeem the bond at par value at
t=1
• What is the yield on the transaction
• Notice that the investor (buyer) has already paid the 5.8678
• Well the EOV is 5.8678 = 1 × v + 5 × v −→ i = 2.25%
• This can be calculated using a TV line on 5.8678 = 1 × a1 i% + 5vi
1+5
• Alternatively, you can directly calculate 5.8678 = 1.0225
14.12. MOTIVATING EXAMPLE - SO WHAT IS YIELD 163

14.12 Motivating example - So What is yield


• Let us summarize

• The buyer paid P = 5.8678 for the bond

• The redemption value at either t = 1 or t = 2 is C = 5

• The coupons are each 1

• If the seller redeems at t = 1 the yield is 2.25%

• If the seller redeems at t = 2 the yield is 10%

• But the seller choses when to redeem

• Therefore the yield to the buyer is the lowest yield

• That is the yield of the callable bond is 2.25%

• A more precise way of saying this is as follows

• The buyer can achieve a yield of at least 2.25%

14.13 IV) The Basic Theorem


• In this example we satisfy the following three assumptions:

• 1) P = 5.867 > 5 = C −→ The bond is bought at a premium

• 2) we used a Bermuda option: The bond could only be redeemed


on coupon dates

• 3) the redemption price C was the same for each redemption date

• Under these assumptions we have y1 = 2.25% < y2 = 10%

• Where yt is the yield on the bond if redemption occurs at time t

• This is a general theorem

• It implies that the yield to the investor is the earliest yield

• Earliest yield refers to the yt with t as small as possible


164 CHAPTER 14. CALLABLE BONDS

14.14 Theorem continued - Discount version


• 1) Suppose the bond is bought at a discount

• 2) The bond can only be redeemed on coupon dates

• 3) The redemption price C is the same for each redemption date

• Then y1 > y2 > . . . > yn

• So the yield to the investor is the latest yield

• Latest yield refers to yt with t as big as possible

14.15 Watered Down Theorem


• 1) Suppose the bond can only be redeemed on coupon dates

• 2) The redemption price C is the same for each redemption date

• Then the minimal yield is either the earliest or latest.

• That is, the minimal yield is yt with t as small or as big as possible


over the range of coupon dates for which C is the same

14.16 Summary of techniques


• Theorem approach: You can use the theorems provided the criteria
are met

• Brute Force approach: You can also compute the yield for each
possible t and compare them

• Sometimes you can combine the two techniques

14.17 Proof of the theorem for the Premium ver-


sion
• Let timeline #1 have redemption at time t, have yield yt and present
value, P
14.18. A HEURISTIC RESULT 165

• Timeline #1

C
−P Cg Cg ... Cg
0 1 2 ... t

• Let timeline #2 have redemption at time t + 1, have yield yt+1 , and


present value, P

• Timeline #2

C
−P Cg Cg ... Cg Cg
0 1 2 ... t t+1

• It suffices to prove yt+1 > yt for any t

• Notice that P is the same for both timelines; This is important;

• The point is that the buyer has already paid for the bond

• We must calculate the yield for each redemption date

14.18 A heuristic result


• We need a preliminary heuristic result

• Fix a timeline of amounts and times

• As the yield goes to infinity the present value of the timeline goes down
to 0

• As the yield goes to 0 the present value of the timeline goes up to the
sum of all amounts (since interest is 0)

• So as yield goes up, PV goes down and as yield goes down, PV goes
up
166 CHAPTER 14. CALLABLE BONDS

14.19 Completion of the Proof


• When timeline #2 uses P it has yield yt+1

• If yt+1 would go down then the PV would go up

• In other words, proving yt+1 > yt ⇔ P V2 > P V1 = P, if both tinelines are evaluated at yt

• But the two timelines have identical amounts except at the end

• So

yt+1 > yt ⇔ P V2 > P V1 = P ⇔ C + Cg > C(1 + i) ⇔ g > i.

• This last inequality is the theorem assumption that the bond is bought
at a premium, that is, P > C or equivalently g > i.

• Hence, the proof is completed.

14.20 V) Good Exam Problems


• N05#22

• M05#11 Conceptual, hard

• QIT#54 Premium, r > i Find C

• QIT#55 Have to 1st find price at several dates and for each price find
minimal yield

• QIT#56 Have to 1st find redemption price at several dates and for
each price find minimal yield

• QIT#57 Discount, P < C Find i

• QIT#91 Redemption possible at every other coupon date


14.21. VI) QIT#54 167

14.21 VI) QIT#54


• Matt purchased a 20-year par value bond with semiannual coupons at
a nominal annual rate of 8% convertible semiannually at a price of
1722.25. The bond can be called at par value X on any coupon date
starting at the end of year 15 after the coupon is paid. The price
guarantees that Matt will receive a nominal annual rate of interest
convertible semiannually of at least 6%. Calculate X.

14.22 QIT#54 - Solution


• r = 4% per half year
• i = 3% per half year
• So i = 3% < 4% = r −→ Bond bought at a Premium
• Apply theorem: Check assumption #1: Bond bought at a Premium
• Check assumption #2: Redemption price the same
• Check assumption #3: Can be redeemed on coupon dates (Notice
problem only cares about t ≥ 15.)
• Conclusion: y1 < y2 < . . . < y30
• Minimal yield to buyer is earliest yield on which redemption can take
place
• So minimal yield occurs at t = 15.

14.23 QIT#54 - Solution Conversions


• t = 15 is really n = 30 because we count half years
• r = 4%, i = 3%
30
• EOV 1722.25 = 4%Xa30 3% + Xv3%
1722.25 30 .
• So X = 4%a30 3% + v3%
1722.25
• Can use calculator to solve for the expression X

• Then immediately conclude X = 1440


168 CHAPTER 14. CALLABLE BONDS

14.24 VII) Helpful Summarizing Table


Type of purchase Option Redemption uniform? Yield curve Investor yield


—– —– —– —– —–
Premium Bermuda Always C y1 < y2 < . . . < yn Earliest date
Discount Bermuda Always C y1 > y2 > . . . > yn Latest date

• We again emphasize that the watered down theorem is very possible

• You can apply the theorem to a collection of callable coupon dates


with the same redemption value

• Simply check the earliest and latest coupon date and take the minimum

• You don’t have to worry about premium and discount


Chapter 15

Duration, Convexity

15.1 Overview
• In this chapter we have the following 10 subgoals

• I) General Multiple Investment/Debt Situation

• II) Basis Points - Definition and why used

• III) Calculus Review Extrema

• IV) Definitions, Formulae, and Concepts - Duration

• V) Affect of small change in i on P

• VI) Good SOA Exam Problems

• VII) M05#3

• VIII) QIT#36 - Good Quiz Problem

• IX) Convexity and its uses

• X) Convexity formulae

15.2 I) General Multiple Investment/Debt Situa-


tion
• Sequence of investments/debts that is deposits and withdrawals

169
170 CHAPTER 15. DURATION, CONVEXITY

• Timeline

C1 C2 ... Cj ...
t1 t2 ... tj ...

• Cj is a deposit (if Cj > 0) or a withdrawal (or debt) (if Cj < 0)

• The present value of these investments at time t = 0 is a function of i


j=n
t
X
P V (i) = Cj v i j (15.1)
j=1

15.3 Sign of P V (i)


• If P V (i) > 0 then you make a profit

• If P V (i) < 0 then you make a loss

• If P V (i) = 0 then you break even.

15.4 Investment Timeline - Problem


• The problem is that you don’t know how interest rates, i, will fluctuate

• Your primary goal is to make a profit (P V (i) > 0.)

• Reddington Immunization: Your secondary goal is to avoid loss if


i fluctuates a little bit.

• Full Immunization: A third goal (sometimes achievable) is to avoid


loss even if i fluctuates alot

15.5 II) Basis Points - Definition and why used


• First we introduce a problem of language

• Suppose the yield increases from 5% to 6%

• What was the percent increase?


15.6. BASIS POINTS - SOLUTION 171

6%−5%
• You might say 5% = 20%

• But if you say the 5% yield increased 20%

• It sounds like the new yield is 25%

• Rather than 6%

15.6 Basis points - Solution


• We introduce Basis points

• 100 basis points equal 1%

• If a 5% yield increases 100 basis points

• Then this unambiguously means that the new yield is 6%

15.7 III) Calculus Review Extrema


• Solution approach to problem of obtaining profit uses calculus to iden-
tify minima

• Let us review calculus minima theorems

• Suppose

• 1) P V 0 (i0 ) = 0

• 2) P V (i0 ) = 0

• 3) P V (i) is concave upward (holding water) (P V 00 > 0)

• Then i = i0 is a local minima

• You have no loss

• And furthermore, small fluctuations in i don’t cause a loss.

• To graphically picture this, think parabola


172 CHAPTER 15. DURATION, CONVEXITY

15.8 Extrema - Algebraic Details


• Warning: Taylor series approach not usually emphasized alot in cal-
culus courses
2
• Taylor Series: P (i) = P V (i) = P (i0 )+P 0 (i0 )(i−i0 )+P ”(i0 ) (i−i20 ) . . .

• We only need the first two terms of the Taylor series to develop the
definitions and theorems

• −→ The linear approximation is P (i) ≈ P (i0 ) + P 0 (i0 )(i − i0 )

• −→ The quadratic approximation is P (i) ≈ P (i0 ) + P 0 (i0 )(i − i0 ) +


2
P 00 (i0 ) (i−i20 )

15.9 IV) Definitions, Formulae, and Concepts -


Duration
• Linear Approximation: P (i) ≈ P (i0 ) + P 0 (i0 )(i − i0 )
P (i)−P (i0 ) P 0 (i0 )
• Normalized P (i0 ) ≈ P (i0 ) (i − i0 ) ≡ −D(i0 , 1)(i − i0 )

• Definition - modified duration


P 0 (i0 ) d
D(i0 , 1) = − = − log P (i0 ) (15.2)
P (i0 ) di

• Definition: (Macaulay) Duration:


dP
dδ0
(15.3)
P (i0 )

dCvit dCeδt
• Note: dt = dt = tCeδt = tCvit . Hence,

• Formula: (Macaulay) Duration:

X Cj vitj
!
0
tj (15.4)
P (i0 )
allj

• English The term duration by itself refers to Macaulay duration


15.10. IN PRACTICE 173

• Duration Relationships:

Modified Duration = v × Macaulay Duration, Why? (15.5)

• Portfolio Formula Duration of several portfolios is weighted average


of durations
n
P V (0)
Pn j
X
• DAll = Dj ,
j=1 i=1 P Vj (0)

• Where Dj is the duration of the j-th portfolio and P Vj (0) is the pur-
chase price (See problem M 05#6 for an example)

15.10 In Practice
• Modified duration is what you theoretically need to measure changes
in PV

• But in practice, (Macaulay) duration is computationally easier to com-


pute (weighted average)

• An increase in duration results in a decrease in price and hence the


minus sign in the definition of duration

• You must be simultaneously aware of three related items - modified


duration, (Macaulay) duration and slope of price

15.11 V) Affect of small change in i on P


• Recall that negative modified duration measures first derivative or
slope of relative price change function

• This allows you to estimate how changes in interest rates affect changes
in profits

• We use standard calculus approach: Approximate P V (i) with the lin-


ear slope line.
174 CHAPTER 15. DURATION, CONVEXITY

15.12 Formulae
• It is confusing to memorize three similar formulae and their uses

• Simply derive them by using the 1st order Taylor approximation


P (i) ≈ P (i0 ) + P 0 (i0 )(i − i0 ) (15.6)

• Consequently,

• For every 100 basis points of increase in the yield, (i − i0 )


P 0 (i0 )
• The approximate relative price change is P (i0 ) = −D(i0 , 1),

• Where D(i0 , 1) is the modified duration

• Summary: Duration allows you to asses the impact of interest rate


volatility

15.13 Illustrative Example and Project I


• Suppose P is the price of a 10-year, zero-coupon bond, with a current
yield of 10%

• Suppose further you wish to sell this bond at t = 3

• We obtain the following (almost linear) graph of the price of the bond
at t = 3 as a function i interest yield

• Table on which Graph is based:

i 9.5% 9.75% 10% 10.25% 10.5%


P (i) 529.79 521.40 513.16 505.07 497.12
P (i) − P (10%)
3.24% 1.61% 0% −1.58% −3.13%
P (10%)

• Answer the following 10 questions assuming the yield increases 100


basis points
15.14. THE TEN PROJECT QUESTIONS 175

15.14 The Ten Project Questions


• 1) What is the exact price

• 2) What is the exact new price

• 3) What is the exact price change

• 4) What is the exact relative price change

• −−−−−−−−−

• 5) What is the duration

• 6) What is the modified duration

• 7) What is the slope of the relative price change (at i = 10%)

• −−−−−−−−−

• 8) What is the approximate relative price change

• 9) What is the approximate price change

• 10) What is the approximate new price

• How good are your approximations for the absolute price change

15.15 Answer Tips to Ten Project Questions


• We assume the yield, i, increases 100 basis points (from 10% to 11%)
7
• 1)What is the exact price Answer 1000v10%
7
• 2)What is the exact new price Answer1000v11%

• 3)What is exact price change Answer Exact New price − exact Price

• 4)What is exact relative price change Answer Divide last answer by


exact price
7
7×1000v10%
• 5) What is the duration Answer 7
1000v10%

• 6) What is the modified duration Answer Divide the last answer by


1.10
176 CHAPTER 15. DURATION, CONVEXITY

• 7) What is the slope of the relative price change Answer Multiply the
last answer by minus 1

• 8) What is the approximate relative price change Answer Slope × 1%

• 9) What is the approximate price change Answer Multiple the last


answer by the exact price

• What is the approximate new price Answer Add exact price to the
last answer

• How good are your approximations for the absolute price change An-
swer Divide the approximate answer by the exact answer and subtract
1

15.16 Project I
• I will post individual Project I-s for each student

• Each student will receive values ot C, n, t, i and the number of basis


points in the change

• So in the above 2 slides: C = 1000, n = 10, t = 3, i = 10%, 100 basis


point change

• This is good practice

• Note the following: To calculate the approximate new price you must
sequentially calculate

• i) duration ii) modified duration iii) slope and then calculate

• iv) approximate relative price change, v) approximate price change vi)


approximate new price

• This sequence is hidden in the project; this sequence is why students


may get simple problems like this wrong
15.17. VI) GOOD SOA EXAM PROBLEMS 177

15.17 VI) Good SOA Exam Problems


• M05#3 Duration - Bond

• N05#2 Duration - Bond

• QIT#65 Duration - Bond

• QIT#68 Duration - Bond - Before / After Coupons

• QIT#66 Duration - Change in Price

• QIT#35 Duration - Derivative Approach

• QIT#36 Duration - Stock (Derivative approach faster)

• QIT#37 Duration - Stock Inflation

• M05#6 Portfolio Duration

• N05#21 Immunization

• QIT#59 Immunization

• QIT#70 Immunization

• QIT#71 Immunization

• QIT#72 Immunization

• QIT#73 Immunization

• M05#15 Asset Matching

• N05#10 Asset Matching

• QIT#51 Asset Matching

• QIT#52 Asset Matching

• QIT#53 Asset Matching

• QIT#69 Asset Matching

• Project 1 (last 3 slides)

• Project 2 (Next section)


178 CHAPTER 15. DURATION, CONVEXITY

• The Daniel-Valeer book gives formulae for many types of durations

• The SOA handout on Immunization gives a very good exposition and


many challenging problems

15.18 VII) M05#3


• A bond will pay a coupon of 100 at the end of each of the next three
years and will pay the face value of 1000 at the end of the three-year
period. The bonds duration (Macaulay duration) when valued using an
annual effective interest rate of 20% is X. Calculate X.

15.19 Solution
• Step 1: Timeline

1000
100 100 100
0 1 2 3

• Step 2: EOV Price P = 100a3 3


+ 1000v20%
20%

• Step 3: TV line

789.35
3 20 CP T −100 −1000
N I PV PMT FV

X Ct v t
• Step 4a: EOV Duration t
P
100v 2 3
• Step 4b: EOV Duration Numerics: P × 1 + 100v 100v
P ×2+ P ×
3
3 + 1000v
P × 3 = 2.7
15.20. VIII) QIT#36 - GOOD QUIZ PROBLEM 179

15.20 VIII) QIT#36 - Good Quiz Problem


• Please read problem in SOA resources

• First we outline the SOA solution

• Hints: #1) Denominator is Stock price: Perpetuity

• Hints: #2) Numerator is price of increasing perpetuity

• Hints: #3) Formulae you should have memorized: a∞ i = 1i , (Ia)∞ i =


1
id

• If you can do quiz with hints but not without them then you must
memorize

• Second we outline a faster solution using derivatives.

• Hints #1) Price of stock as a function of interest rates is P (i) = D 1i


Why?

• Hints #2) Take the derivative of P (i) with respect to i (Piece of cake)
P0
• Hints #3) Take ratio P (Piece of cake)

P0
• Hints. Fill in #4) P gives Duration of fill in amount.

• To Duration of fill in amount I need to multiply by fill in amount

• Hints #5 Now plug in i = 10% and get your answer.

• In the process you have derived a useful formula for stock duration
without inflated dividends

15.21 IX) Convexity and its uses


• Duration approximates the tangent line to the present value curve P (i)

• Convexity approximates the quadratic fit to the present value curve


180 CHAPTER 15. DURATION, CONVEXITY

15.22 Why convexity


• If locally you look like y = x2 −→ local minima −→

• −→ fluctuations don’t cause loss −→ You are immunized against loss

• Immunization important (no loss). It is discussed next time

15.23 X) Convexity Formulae


• We start with 2 term Taylor
2
• P (i) ≈ P V (i) = P (i0 ) + P 0 (i0 )(i − i0 ) + P 00 (i0 ) (i−i20 )

• Subtract the constant term and divide by P (i0 )


P (i)−P (i0 ) P 0 (i0 ) 1 P 00 (i0 )
• P (i0 ) = P (i0 (i − i0 ) + 2 P (i0 (i − i0 )2

• We can now produce 3 convexity definitions analogous to the duration


definitions

15.24 Summary of Definitions and Formula


• Macaulay Duration ×v = Modified Duration

Adjective Duration Convexity


P 0 (i0 ) P 00 (i0 )
M odif ied D(i, 1) = − C(i, 1) =
P (i0 ) P (i0 )
dP d2 P
dδ dδ 2
(M acaulay)
P (i) P (i)
t t
! !
X Cj v j X Cj v j
F ormula (M acaulay) i
tj i
t2j
P (i) P (i)
all j all j

• Footnotes: Unspecified duration is macaulay duration;

• But unspecified convexity is modified convexity

• There is a nice relationship between macaulaly duration and modified


duration given above.
15.25. GOOD EXAM PROBLEMS 181

• But there is no nice relationship between macaulay convexity and mod-


ified convexity

15.25 Good Exam Problems


• There are no exam problems on convexity!!!

• The SOA recently remedied this by adopting a new book with a great
new handout

• Hint: If you wanted to test someone on new material where would you
go?

• Daniel-Valeer has excellent problems

• Another approach: Redo all duration problems with duration replaced


by convexity.
182 CHAPTER 15. DURATION, CONVEXITY
Chapter 16

Full Immunization

16.1 Overview
• We have five subgoals in todays chapter

• I) Reddington Immunization, Assets, Liabilities,

• II) Full immunization Theorem

• III) Extended Illustrative Examples (Full Immunization)

• IV) Project 2 Immunization Project

• V) Asset Matching

16.2 I) Reddington Immunization, Assets, Liabil-


ities,
• Immunization means you are immunized or protected against lost

• Reddington immunization means you are protected against small


fluctuations in i

• Full immunization means you are protected against any fluctuations


in i

• Graphically, Reddington immunization corresponds to a local minima


(Why?)

183
184 CHAPTER 16. FULL IMMUNIZATION

• Graphically, full immunization corresponds to a global minima (Why?)

16.3 Reddington Immunization Theorem


• Theorem Suppose P (i0 ) ≥ 0, P 0 (i0 ) = 0, P 00 (i0 ) ≥ 0

• Then you are immunized against small fluctuations in i

• Proof is by calculus graphing

16.4 Reddington Immunization - Assets Liabilities


t
X
• Recall P (i) = Cj v i j
allj

• Assets are investments for which Cj > 0

t
X
• A(i) = Cj v i j
Cj >0

• Liabilities are investments for which Cj < 0

t
X
• L(i) = |Cj |vi j
Cj <0

• So P (i) = A(i) − L(i)

16.5 Reddington Immunization Theorem Reformu-


lated
• Theorem If A(i) ≥ L(i), A0 (i) = L0 (i), and A00 (i) ≥ L00 (i)

• Then you are locally immunized against small fluctuations.

• Proof 0 ≤ P 00 = (A − L)00 ⇐⇒ L00 ≤ A00


16.6. II) FULL IMMUNIZATION THEOREM 185

16.6 II) Full immunization Theorem


• Rather exciting. Mathematics assures you that you can’t lose

• Note: Theorem assumption: You have one liability with two assets

• One before the liability and one after the liability

• Timeline

interest rate i0 , eδ0 = 1 + i0


A1 −L1 A2
0 t1 t t2

• Theorem: If P (δ0 ) = 0 = P 0 (δ0 ) =⇒ P (δ) > P (δ0 ); fully immunized!

• Idea of proof: Show that graph of P (i) looks like a Parabola

16.7 III) Extended Illustrative Example (Full Im-


munization)
• We present some numbers and then show how to fully immunize

• Liability: Must pay $100,000 at t = 7. Interest rates are 2%.

• Assets: Buy zero coupon bonds before and after the liability date:
Say t = 3, t = 10.

• Problem: How much of each type of zero coupon bond should be


purchased to guarantee payment?

• The technique of matching the assets to the liability is called asset


matching

• Solution: To immunize we will solve two equations: P V (liability) =


P V (Assets); and P V 0 (Liability) = P V 0 (Assets).
186 CHAPTER 16. FULL IMMUNIZATION

16.8 Word Problem - Variables


• Unknowns: Buy $x worth of three year zero coupon bonds
• Unknowns: Buy $y worth of 10 year zero coupon bonds
• Note: x and y are prices at time t = 0, and therefore are present
values.
• An alternate approach lets x and y be the number of bonds of each
type
• But if you approach that way you must separately compute Present
Value
• The way I have approached, the present values are already computed
x, y, and x + y
• Still another alternate approach lets x, y be the values at maturity of
the total redemption
• When solving problems it is important to carefully select variables:
PV, number, FV

16.9 Strategy
• We have two unknowns −→ We need two equations
• We obtain one equation for P (i0 ) = 0
• We obtain one equation for P 0 (i0 ) = 0
• We then solve the two equations in two unknowns
• And find the value of x and y

16.10 First Equation is P (i0 ) = 0.


• Recall P (i0 ) = 0 =⇒ A(i0 ) = L(i0 ).
• But A(i0 ) = x + y (Why?)
7 = 87056.02 (Why?)
• L(i0 ) = 100, 000v2%
7 .
• So first equation is x + y = 100, 000v2%
16.11. EQUATION 2: ASSETS: NUMERATOR, DENOMINATOR 187

16.11 Equation 2: Assets: Numerator, Denomi-


nator
• Second equation is P 0 = 0 or A0 = L0

• Let us do A0 first

• We could compute A0 directly.


0
• But a slick trick is to compute the Macaulay Duration which is − AA

• Notice that the denominator of A doesn’t bother us

• The fraction equals 0 iff the numerator equals 0

• So we will set durations equal rather than the derivatives of A and L

16.12 Equation 2: Assets: Duration Computation


   
C1 v t 1 C2 v t2
• Duration is P t1 + P t2

• But what is t1 , t2

• Well t1 = 3, t2 = 10 since they are 3-year and 10-year zero coupon


bonds

• What is P ? Well P = x + y because x and y are present values at time


0.

• What is C1 v t1 and C2 v t2 ? They are x and y respectively since they


are prices
x y
• Putting it all together: Duration = x+y ×3+ x+y × 10.

16.13 Equation 2: Liabilities: Duration


• What is the Macaulay duration of the liability

• Well the liability is in effect a 7 year zero coupon bond of $100,000

• We know that the duration of such a bond is n = 7


188 CHAPTER 16. FULL IMMUNIZATION

16.14 Equation Summary


10y
• P 0 (i) = 0 =⇒ 3x
x+y + x+y = 7.

• P (i) = 0 =⇒ x + y = 100000(1.02)−7

• Need to solve

• Then need to check we are ok with convexity

16.15 Solve Equations - How?


• We have two linear equations in two unknowns

• Clear denominators of second equation by multiplying by x + y

• Gather terms till x is related to y by a constant

• Go back to first equation and substitute value of x or y in terms of


the other

• You now have a linear equation in one variable which is easy to solve

• So x = 37309.72, y = 49746.30

16.16 Are we immunized


• We only used two equations corresponding to P and P 0 but are fully
immunized

• We can check convexity

• Convexity of zero coupon liability bond is n2 = 72 = 49


37309 49746
• Convexity of assets is 87056 × 32 + 87056 × 102 = 61 > 49

• So we indeed have P 00 > 0 showing we have full immunization (It looks


like a parabola)
16.17. IV) PROJECT 2 IMMUNIZATION PROJECT 189

16.17 IV) Project 2 Immunization Project


• For Project 2 each of you will do your own personal project
• You will be given a) Liability b) liability date c) two dates for assets
d) interest rate
• You will then recommend purchase of zero coupon bonds to fully im-
munize liability

16.18 V) Asset Matching


• There are asset matching problems that don’t require immunization
• What do they require?
• They require buying assets to pay off a liability
• In symbols we want A(i) = L(i)
• We just want to match the liability
• We are not concerned about fluctuations in interest rates
• We assume the interest rate is constant and just want to match assets
to liabilities
• Very often this type of problem is called exact asset-liability matching

16.19 M05-#15
• An insurance company accepts an obligation
• To pay 10,000 at the end of each year for 2 years.
• The insurance company purchases a combination of the following two
bonds
• At a total cost of X in order
• To exactly match its obligation:
• (i) 1-year 4% annual coupon bond with a yield rate of 5%
• (ii) 2-year 6% annual coupon bond with a yield rate of 5%
190 CHAPTER 16. FULL IMMUNIZATION

16.20 Asset Matching Timeline


• We must adjust our strategies

• The two unknowns will be F, G

• F, G are the total face values of the two bonds

• Timeline illustrating exact asset-liability matching

t 0 1 2
Liability 10000 10000
F 1.04F 0
G 0.06G 1.06G

16.21 Equations
• The basic EOV is Assets = Liabilities

• We have two equations one for each liability

• To solve them we start at right side at t = 2

• Why? Because you will have one equation in one unknown and you
can solve

• 1.06G = 10000

• Next go to t = 1

• 0.06G + 1.04F = 10000

• Two equations in two unknowns; piece of cake; solve for G; then solve
for F

• But F, G are face values. So then calculate prices PF , PG

• Final answer is PF + PG
Chapter 17

Insurance, Puts, Calls,


Forwards

17.1 Overview
• We have the following 9 subgoals in this chapter.

• I) Insurance, Derivatives

• II) Derivative Overview - Graphs

• III) Derivative Overview - 6 Building Blocks

• IV) Long Buy

• V) Short sale

• VI) Forwards Futures

• VII) Profit / Payoff Tables /Graphs

• VIII) Options - Calls

• IX) Options - Puts

• For Homework I want you to think how you would solve DS-Q#3

• We will do DS-Q#3 next time.

191
192 CHAPTER 17. INSURANCE, PUTS, CALLS, FORWARDS

17.2 I) Insurance, Derivatives


• What is the difference: Buy insurance vs. Buy a car

• When you buy a car: You receive the actual car

• When you buy insurance: You buy the right to buy a car if your
current car is destroyed

• Insurance typifies not buying objects but buying rights to buying ob-
jects

• We call such transactions Derivatives

• What you buy is derived from the actual buy

17.3 Types, Issues and Goals of Derivatives


• What are the types of derivatives

• What are the issues of derivatives

• What are the goals of interest for derivatives

17.4 Goals
• We will discuss puts,call,forwards-futures-buys-sales

• We will discuss long, short positions

• We are interested in possible profits, losses

17.5 II) Derivative Overview - Graphs


• Think of buying insurance for your car

• Your future profits/losses depend on the value of your car

• If your car is fine −→ you don’t exercise the insurace −→ you lose
your insurance premium
17.6. DERIVATIVE OVERVIEW - PROFIT DIAGRAMS/GRAPHS 193

• If your car is destroyed −→ you do exercise the insurance −→ you get


a new car

• If the new car is worth more than your old car you profit

• You spent premiums and now have a better car

17.6 Derivative Overview - Profit Diagrams/Graphs


• The example in the last slide illustrates what profits depend on

• They depend on possible future values of the underlying asset (in this
case the car)

• In the general case we are interested in a Profit graph

• How does profit (y axis) depend on

• The underlying asset value (x axis)

• P rof it = f (AssetV alue)

17.7 Summary of Possibilities with Derivatives


• Possible Actions: Buy/Sell Long/Short

• Possible things to Buy/Sell: Forwards-futures-buys-sales/Options:


Calls / puts

• Consequent Graphs: Payoff / Profit

• Combinations of the above

17.8 III) Derivative Overview - 6 Building blocks


• As in other parts of the course we emphasize a building block approach

• For example we built up annuity problems from 3 basic building blocks


(level, increasing, decreasing)

• Similarly we studied amortization from 3 sets of basic formulae (gen-


eral, level, total)
194 CHAPTER 17. INSURANCE, PUTS, CALLS, FORWARDS

17.9 Derivative Building Blocks


• There are 6 basic Derivative building blocks

• Every derivative component is either a i) forward/future/bu0y/sale ii)


call or iii) put

• Every Derivative component is either sold a) long or b) short

• So the 6 derivative building blocks are combinations of long/short and


forward-future-buy-sale/call/put

17.10 This Lecture and Next lectures


• We will cover the 6 building blocks today

• In the next two lectures we will learn how to combine them

17.11 IV) Long Buy


• Standard Example: I sell you my watch

• At t = 0 you give me $30

• At t = 0 I give you the watch

• The item sold is called the Asset

• The money you give me for the asset is call the Price

• We call a buy/sale long if the Asset and price are both exchanged at
time t = 0

• The person receiving the money is called the Seller

• The person receiving the object is called the Buyer


17.12. PROFIT ON LONG SALE 195

17.12 Profit on Long Sale


• Suppose the Asset goes up: t = 0, P rice(0) = 30; t = 1, P rice(1) =
40

• You lose 30 at t = 0;

• You acquire the watch at t = 0

• You sell the watch at t = 1 for 40

• You have made a profit amount (Interest amount) of 10.

17.13 Profit on Long Sale


• If the asset price goes up −→

• Buyer in long transaction made a profit of 10.

• Seller in long transaction loses 10

• If the asset price goes down −→

• Buyer in long transaction loses 10.

• Seller in long transaction profits 10

• Again this is all rough since we must take into account interest

17.14 V) Short Sale


• I sell my 30 dollar watch short

• This means

• I receive the 30 at t = 0

• I promise to deliver you a watch at t = 1

• qquad However, at t = 0 I am short of the watch and do not yet have


it.
196 CHAPTER 17. INSURANCE, PUTS, CALLS, FORWARDS

17.15 Profit analysis if asset goes up


• Suppose watches are worth 40 at t = 1

• Then I pay 40 at t = 1, obtain a watch, and give it to you

• Notice that the asset has gone up but I, the seller, have lost 10

• Although the asset has gone up, you, the buyer, have gained 10

17.16 Profit analysis if asset goes down


• Suppose watches are worth 20 at t = 1

• I only need to pay 20 to cover my short and obtain a watch for which
I received 30

• So my profit amount is 10: I received 30 and paid 20

• You, the short buyer, lose 10 if asset price goes down

17.17 Profit-Loss Summary


• Long buyer makes money if asset price goes up

• Short seller makes money if asset price goes down

• So long and short are opposites (one profits when the other loses)

17.18 Warning on terminology


• Here the word Short refers to the transaction

• The word Short can also be used to describe a position

• We will see this below

• But it is confusing because the same word can be used in two ways
17.19. STRATEGIES 197

17.19 Strategies
• Items like watches don’t fluctuate that much in price

• Stocks and bonds do fluctuate

• If you think a stock will go up you buy long and expect to profit

• If you think a stock will go down you sell short and expect to profit

17.20 IV) Forwards and Futures


• What is a forward?

• At time t = 0 we (buyer and seller) enter a contract

• I the buyer agree to pay you P at t = 1

• You the seller agree to transfer to me stock X at t = 1 for P

• There is no fee (Premium) at t = 0 for this contract

17.21 Forward terms


• Again: The stock is the underlying asset

• The price, P , is the Price

• The time of exchange (in this example t = 1) is the expiration date

• We say that my position in the forward is long (I make money if asset


goes up)

• We say that your position in the foward is short (You make money if
asset goes down)

• Important: No Premium money is paid at t = 0 for entering the


contract
198 CHAPTER 17. INSURANCE, PUTS, CALLS, FORWARDS

17.22 Four types of contracts


• We have covered forwards and ordinary sales

• They differ based on what happens at t = 0 and t = 1

• There are actually 4 possibilities.

17.23 Four types of contracts


• In all 4 cases I give money and you give me stock

• That is the transaction is actual rather than derived

• In all 4 cases an agreement is made at t = 0

• The transfers may happen at t = 0 or t = 1

Description P ay Receive asset


Outright purchase t=0 t=0
F orward t=1 t=1
P repaid f orward t=0 t=1
F ully leveraged purchase t=1 t=0 (17.1)

• t = 1 can refer to any unit of time

• In a future lecture we will discuss the fair price of forwards and how
to adjust for dividends

17.24 Forwards vs. Futures


• Future and Forwards are almost the same thing

• They both are contracts for exchanges at t = 1.

• The differences between them are subtle

• Futures are done on an exchange; forward contracts are not

• An exchange is an official business place for stocks, bonds etc.


17.25. WHY USE FORWARDS / FUTURES 199

• Exchanges have rules which affect futures; forwards have less rules

• The rules give guidance on how to deal with a situation in which

• One basic difference is that futures are settled daily

• The determination of who owes whom is called Market to market

• There are other differences but they are consequences of the daily
settlement.

• Contrastively, a forward is a sort of private contract. It is off-market.

17.25 Why use forwards / futures


• Typical Example: I raise hogs, cattle, sheep, etc.

• It may take me a half a year or year to raise them

• These animals have to be fed wheat or alfafa

• If the price of wheat or alfafa fluctuates I pay more for alfafa, eating
up my profits

• So I am interested in locking into rates to guarantee my profit

17.26 VII) Profit / Payoff Tables / Graphs


• Let us deal with a forward with expiration date at t = 1

• Remember: P rof it = f (Asset value)

• Payoff Table Long Forward

Agreed price Actual Price-X My profit-Y


t=1 t=1
1000 900 −100
1000 1000 0
1000 1100 100
200 CHAPTER 17. INSURANCE, PUTS, CALLS, FORWARDS

• Explanation: (Row 1) If I agree to buy for a 1000 (at t = 1) but the


asset is worth 900 (at t = 1) then I have lost 100
• To make a profit table you can use excel and make many rows
• However you only need three points: Below, at, above agreed price
• Note: Payoff is what you get at t = 1; profit is payoff − expenses.
• But there are no expenses (Premium) for a forward or future.
• So for a forward / future, P rof it = P ayof f

17.27 Payoff/Profit Diagram Long Forward


• X axis: The price of the underlying asset at t = 1
• Y axis: The payoff or profit from the transaction at t = 1
• Description of Graph: Line slanting upwards
• I use clock positions to describe graphs
• a line slanting upwards resembles the clock position of 7:05
• I also use letters: U indicates a slanting upwards line
• The slope is 1; Can you justify this from the above table?

17.28 Options - Calls


• What does it mean for me to buy a Call on stock X
• It means I buy the right or option to do one of two things
• (Possibility #1)
• Buy the underlying asset, X,
• At the future expiration date,
• At the agreed strike price,
• (Possibility #2)
• I have the right to do nothing (no buy; no sale)
17.29. WHY USE A CALL 201

17.29 Why use a call


• Think of the hog and alfafa example mentioned above
• A call would give me the option of buying alfafa at P if the price goes
up
• But if the price goes down I buy at the lower price
• So my future expenses for alfafa are capped at P
• Calls are therefore a sort of insurance - I can’t spend more than P
• Like an insurance policy I have to pay for the right to have this option
• The payment price is called the premium

17.30 Call Terminology


• If I decide to buy the asset at expiration date we say I exercise the
option
• The other terms - price, underlying asset, premium, expiration
date - are mentioned above

17.31 Buying style


• These are the same three styles we discussed for callable bonds
• An American style allows me to exercise the call at any time
• A Bermuda style allows me to exercise the call at certain times
• An European style allows me to exercise the call only at the expira-
tion date
• For the most part we will deal with European styles.

17.32 Example
• Premium: 93.81
• Interest rate for 6 months: 2%
• Strike Price: $1000
202 CHAPTER 17. INSURANCE, PUTS, CALLS, FORWARDS

17.33 Call Table/Diagram


• Payoff-Profit Table Long Call

Actual Price-X Agreed Price Buy? Reason Payoff Premium Premium Profit-Y
t=6 t=6 t=6 t=6 t=0 t=6 t=6
900 1000 No No buy for more − −93.81 −95.68 −95.68
1000 1000 − − − −93.81 −95.68 −95.68
1100 1000 Y es Buy cheap 100 −93.81 −95.68 4.32

• The Call Payoff Diagram/graph looks like the clock position TIME 9:10

• We can also use letter notation: HU, a horizontal line (slope 0) followed by an upwards slanting line

• Call Profit Diagram Same shape but lower (The two graphs are translations)

• Why. Because Profit = Payoff − Premium.

• Note the horizontal line of graph =⇒ no exercise of options; the slanting line =⇒ exercise of option

17.34 Graph Table Basics


• The following slide gives important advice on how to create a table for
a call graph

• Graph tables should have 3 rows (before, at, after)

• Tables should have at least 4 columns

• i) Actual price (X-axis), 2) Payoff (Possibly 0) 3) Cost=Premium at


t = expiration date 4) Profit (Y -axis)

• You might also benefit from using the following 3 more columns

• 5) Should you exercise 6) Why? 7) Cost at t = 0

• The purpose of the extra columns is to prevent errors (e.g. confusing


t = 0 and t = expiration date

• Furthermore, label each column with two header rows, the second row
indicating t = 0 or t = expiration date
17.35. IX) OPTIONS - PUT 203

17.35 IX) Options - Put


• What does it mean for me to buy a Put on stock X

• It means I buy the right or option to do one of two things

• (Possibility #1)

• Sell the underlying asset, X,

• At the future expiration date,

• At the agreed strike price,

• (Possibility #2)

• I have the right to do nothing (no buy; no sale)

17.36 Call Put Diagrams


• Long Call Diagram/Graph looks like TIME 9:10 (horizontal line,
followed by slanting upwards line)

• Using letter notation we would abbreviate this as HU

• Long Put diagram looks like TIME 2:50 (Slanting downwards line,
followed by horizontal line)

• Using letter notation we would abbreviate this as DH

• Here D means a sloping down line with slope -1

• Important point: Short call and short put diagrams are x-axis re-
flections of the corresponding long positions

• x-axis Reflection means for each (x, y) in long diagram we have


(x, −y) in short diagram.

17.37 Summary: Call Put Graphs


• We use the terms write a call, sell a call, sell a put, or write a
put to indicate short sales
204 CHAPTER 17. INSURANCE, PUTS, CALLS, FORWARDS

Description Graph Graph Buyer has Right to (17.2)


I buy a Call TIME 9 : 10 HU buy asset (17.3)
I buy a P ut TIME 2 : 50 DH sell asset (17.4)
I write a Call TIME 9 : 20 HD f orce me to sell (17.5)
I write a put TIME 2 : 40 UH f orce me to buy (17.6)

17.38 Thinking HW for Thursday


• Find in the following question, the basic building blocks: puts, calls,
buy-sell-forwards-future. Describe each transaction - who is buyer and
seller, when will transfers take place etc.

• DS-Q#3. Happy Jalapenos, LLC has an exclusive contract to sup-


ply jalapeno peppers to the organizers of the annual jalapeno eating
contest. The contract states that the contest organizers will take de-
livery of 10,000 jalapenos in one year at the market price. It will cost
Happy Jalapenos 1,000 to provide 10,000 jalapenos and todays market
price is 0.12 for one jalapeno. The continuously compounded risk-free
interest rate is 6%. Happy Jalapenos has decided to hedge as follows
(both options are one-year, European): Buy 10,000 0.12-strike put op-
tions for 84.30 and sell 10,000 0.14-stike call options for 74.80. Happy
Jalapenos believes the market price in one year will be somewhere be-
tween 0.10 and 0.15 per pepper. Which interval represents the range
of possible profit one year from now for Happy Jalapenos?

• The entire next lecture will be spent on this problem.


Chapter 18

Introduction to Derivatives

18.1 Overview
• We have one goal today: Coverage of DS-Q#3

• This example illustrates how basic business applications come from


the building blocks

• Accordingly we have 11 subgoals in todays lecture

• I) Building blocks: Puts/options, long/short, buy-sale-forward-future

• II) Typical multi-step problems DS-Q#3

• III) Solution Approach: i) Find/list all building blocks in the problem

• IV) ii) Create profit table/graph for each building block

• V) iii) Join them together

• VI) iv) Calculate Grand profit and draw graph

• VII) Non Computational Graphing method approach

• VIII) Basic graph components: U,D,H

• IX) Basic Graph component addition laws

• X) Application to DS-Q#3

• XI) Peachy example of non computational graphing: DS-Q#9

205
206 CHAPTER 18. INTRODUCTION TO DERIVATIVES

• Note that subgoals 7,8,9,11 form a unit teaching non computational


graphing
• Currently this topic can not be found in any other textbook
• It is however a very useful technique.

18.2 I) Building blocks: Puts/options, long/short,


forward-future-buy-sale
• We learned the six basic building blocks in Chapter 17
• puts, calls, forward-purchases
• long,short
• You must know their profit diagrams and graphs
• Everything else in Chapter 18 and 19 is built from them

18.3 II) Typical multi-step problems DS-Q#3


• This is a great multi-part problem
• We presented it last time
• We state it here and then analyze its components
• DS-Q#3. Happy Jalapenos, LLC has an exclusive contract to supply
jalapeno peppers to the organizers of the annual jalapeno eating con-
test. The contract states that the contest organizers will take delivery
of 10,000 jalapenos in one year at the market price. It will cost Happy
Jalapenos 1,000 to provide 10,000 jalapenos and todays market price
is 0.12 for one jalapeno. The continuously compounded risk-free in-
terest rate is 6%. Happy Jalapenos has decided to hedge as follows
(both options are one-year, European): Buy 10,000 0.12-strike put op-
tions for 84.30 and sell 10,000 0.14-stike call options for 74.80. Happy
Jalapenos believes the market price in one year will be somewhere be-
tween 0.10 and 0.15 per pepper. Which interval represents the range
of possible profit one year from now for Happy Jalapenos?
18.4. III) SOLUTION APPROACH: I) FIND/LIST ALL BUILDING BLOCKS IN THE PROBLEM207

18.4 III) Solution Approach: i) Find/list all build-


ing blocks in the problem
• Sell call - so we take a short position

• 10000 items are sold

• The Strike price is 14 cents per unit

• Our goal will be to analyze profit and graph

18.5 Component 2 - Put


• Buy put - so we take a long position

• 10000 items are bought

• The Strike price is 12 cents per unit

• Our goal will be to analyze profit and graph

18.6 Component 3 - Purchase


• Happy Jalapenos is seller; Eating Contest is buyer

• At t = 1, Happy Jalapenos supplies 10000 peppers to Eating contest

• Eating contest pays market price as of t = 1

• Transportation of 10000 peppers will cost Happy Jalapenos, $1000

• In computing the profit diagram, the $1000 is an added cost subtracted


from profit

18.7 Additional information - Interest rate


• δ = 6%

• The problem uses the English word continuously


208 CHAPTER 18. INTRODUCTION TO DERIVATIVES

• −→ Hence the rate is continuously compounded not annual effective

18.8 IV) Create profit table/graph for each build-


ing block

SHORT CALL Should other P ayof f on 1 P ayof f on all COST COST P ROF IT − Y
M arket price − X Strike price buy to me to me P remium P remium at
T =1 t=0 t=1 t=1 t=0 t=1 t=1
0.1 0.14 No 0 0 74.8 79.43 79.43
0.11 0.14 No 0 0 74.8 79.43 79.43
0.12 0.14 No 0 0 74.8 79.43 79.43
0.13 0.14 No 0 0 74.8 79.43 79.43
0.14 0.14 No 0 0 74.8 79.43 79.43
0.15 0.14 Y es −0.01 −100 74.8 79.43 −20.57

18.9 Call Profit Table Comments


• Position is short. I sell call. Other person is buyer

• So I receive premium; hence premium is positive, a profit

• In short sale, buyer determines whether to exercise option

• Buyer exercises option if profitable for him/her

• So Buyer exercises option if a loss for me

• Remember: For both puts and calls, the slanting part of graph corre-
sponds to exercise of option

• Graph of P uts looks like time position 2 : 50 Call Graphs =


time position 9 : 10

• Important to value premium at t = 0 and t = 1.

• To get value at t = 1 multiply value at t = 0 by 1 + i = e0.06

• We have two ways to do table: Strike price equals i) 12 cents ii) 1200
dollars

• This is a matter of taste; we will have to eventually deal with the


10000 peppers
18.10. LONG PUT 209

18.10 Long put



Long put Should I P ayof f on 1 P ayof f on all P remium P remium P rof it − Y
M arket price − X Strike price sell to me to me At At at
T =1 t=1 t=1 t=0 t=1 t=1
0.1 0.12 Y es 0.02 200 −84.3 −89.51 110.49
0.11 0.12 Y es 0.01 100 −84.3 −89.51 10.49
0.12 0.12 Y es 0 0 −84.3 −89.51 −89.51
0.13 0.12 No 0 0 −84.3 −89.51 −89.51
0.14 0.12 No 0 0 −84.3 −89.51 −89.51
0.15 0.12 No 0 0 −84.3 −89.51 −89.51

• Premiums are negative since they represent a cost to the buyer

• Put graphs look like time position 2:50; exercising of option occurs
on slanting part of graph

18.11 Forward at t = 1

Jalapeno Sale P rof it at 1
M arket price − X Cost to me P ayof f to me P rof it to me − Y
T =1 T =1 T =1 t=1
0.1 −1000 1000 0
0.11 −1000 1100 100
0.12 −1000 1200 200
0.13 −1000 1300 300
0.14 −1000 1400 400
0.15 −1000 1500 500

• Note that cost (of transportation) is fixed at $1000

• The fact that the 1000 is paid at t = 1 is not explicit in problem

• You have to infer it based on your knowledge of how transactions occur

• The payoff is 10000 × market price

18.12 V) Join tables together


• How do you join together the above tables

• Join is a technical term in database theory


210 CHAPTER 18. INTRODUCTION TO DERIVATIVES

• You join different tables on a common column

• All three tables have in common the market value column

• When joining: We take from each table two columns,

• The market value = X column and the profit = Y column

18.13 Join the Tables continued


• The results are shown in the next slide

• Note since we anticipated the need for a join, −→

• −→ therefore we used in the tables, 5 market values 11-15 cents

• Ordinarily we only need 3 market values.

• Note: The idea of using database theory to join tables is a contribution


of this book

• This perspective does not occur in other books

18.14 Grand Joined Table


• Join Table

M arket price P rof it P rof it P rof it T otal


T =1 Call P ut Jalapenos P rof it
T = 1 T = 1 T = 1 T = 1 T = 1
0.1 79.43 110.49 0 189.91
0.11 79.43 10.49 100 189.91
0.12 79.43 −89.51 200 189.91
0.13 79.43 −89.51 300 289.91
0.14 79.43 −89.51 400 389.91
0.15 −20.57 −89.51 500 389.91
18.15. COMPLETION OF PROBLEM 211

18.15 Completion of problem


• We can draw graph directly

• We can see that profit is guaranteed no matter what market price is

• Profit always lies between 189.91 and 389.91

• How can profit be guaranteed?

• Because experience taught that prices lie between 10 cents and 15


cents

• We will examine this in more detail after introducing the graph method

18.16 VII) Non Computational Graphing method


approach
• We can draw graph directly without ANY computation!

• We now introduce this non computational graph method

• We first need a method to describe piecewise linear graphs

• We describe graph components and how to add them

• We can construct the form of a composite graph without any compu-


tation

18.17 VIII) Basic graph components: U,D,H


• The graphs of puts,calls,buy-sell-forward-future have 3 components

• U refers to a line segment slanting upwards (positive slope of 1)

• D refers to a line segment slanting downwards (negative slope of -1)

• H refers to a horizontal line segment (slope of 0)


212 CHAPTER 18. INTRODUCTION TO DERIVATIVES

18.18 Examples
• A long call graph is abbreviated HU corresponding to a 9:10 clock
position

• A long put graph is abbreviated DH corresponding to a 2:50 clock


position

• A short call graph is abbreviated HD corresponding to a 9:20 clock


position

• A short put graph is abbreviated UH corresponding to a 2:40 clock


position

18.19 Slope Interpretation of U,D,H


• Consider a U of a graph

• Recall that the X axis represents possible underlying-asset values

• Recall that the Y axis represents profit.

• Why is the graph a slanting upward line, U,

• Because in this graph component each 1 dollar increase in underlying


assets results in a 1 dollar increase in profit

• In other words, the slope is 1

• Similarly the slope of a D component is -1; the slope of a H component


is 0

• This unit slope of 1 or -1 is valid for long and short calls and puts as
well as buys and sales

18.20 IX) Basic Graph component addition laws


• Suppose you have a combination of puts, calls and forwards

• Its graph is a piecewise linear graph

• To describe its form you have to identify two things:


18.21. X) APPLICATION TO DS-Q#3 213

• 1st) You have to identify the number of total distinct regions

• For example, in a call or put there are 2 regions (one H and the
2nd either U,D)

• 2nd) You have to identify the slope of each line segment

• If you want the actual graph vs. its form you have to identify the
y-intercept

• The rule for slope is as follows

Slope of line segment = Total number of U − Total number of D


(18.1)

• This rule follows from the way we have defined U,D,H for basic com-
ponents

• Example: D + H + D + U + D = 2D indicating a slope of -2.

18.21 X) Application to DS-Q#3

• Piece of Cake!

• A Short call is HD

• A long put is DH

• A purchase is U

• To add these we have to specify the regions where they apply


214 CHAPTER 18. INTRODUCTION TO DERIVATIVES

18.22 Graph of DS-Q#3


M arket P rice Below 12 Between 12 14 Above 14


Short Call H H D
Long P ut D H H
F orward U U U
Sum H +D+U H +H +U D+H +U
Join Graph H U H
10000 units 10000H 10000U 10000H

• The construction of this table should be obvious

• Note how we infer the graph form without any computation

• If we now needed numbers we would simply compute profit at X = 0.


We would find Y (0) = 189.91

• But graph is 100H on (0, 0.12) implying that Y (0.12) = 189.91.

• What is Y (13) Well ∆X = 0.13 − 0.12 = 0.01


∆X
• But slope = 10000 = ∆Y and hence ∆Y = 10000 × 0.01 = 100

• So Y (0.13) = 100 + Y (0.12) = 289.91

18.23 DS-Q#3 Revisited


• We can now discuss DS-Q#3 more thoroughly

• Suppose you wanted to make a Jalapeno deal

• You might make a lot of profit

• But you might lose

18.24 Why buy a Put


• The loss is indicated by the Upward line below the x axis

• To cancel this line we need a downward slanting line


18.25. WHY SELL A CALL 215

• Look over the basic 6 components and find the basic component with
a downward slanting line

• The Long Put has a downward slanting line

• Hence we buy a long put as insurance to prevent loss

• The downward put line and upward forward line cancel

18.25 Why sell a Call


• But buying the put costs money and that is loss

• A standard option trick is to sell a call bringing in premium

• The premiums of the sold call and long put almost cancel

• You cut your profits (Downward short call line + upward forward line)

• But if you dont expect peppers to advance very much this is ok

18.26 XI) Peachy example of non computational


graphing: DS-Q#9
• Please read DS-Q#9 in the class resources.

• Let us study part (A) Buy a 90-put, buy a 110 put, sell two 100 puts

• The graph is shown in the next slide

• The important point: We know the graph is HUDH

• We know this without doing any computations

• For HW apply this graphical method to DS-Q#9B,C,D,E.

• Only one of these does not have the form HUDH

• So you can compute the answer to DS-Q#9 without making numerical


computations.
216 CHAPTER 18. INTRODUCTION TO DERIVATIVES

18.27 Graph of DS-Q#9b



M arket P rice Below 90 Between 90 100 Between 100 110 Above 110
Buy 90 call H U U U
Buy 110 call H H H U
sell two 100 calls H H D D
2nd 100 call H H D D
Symbolic sum H+H+H+H U +H+H+H U +H+D+D U +U +D+D
Join Graph H U D H
Chapter 19

Derivatives - Graphing
Methods

19.1 Overview
• This chapter deals with Derivatives

• We cover the following 5 subtopics

• I) Derivative Definition

• II) Important Derivative Properties

• III) Good Derivative Problems

• IV) General Approach to Graphical Problems

• V) Illustrative Examples

19.2 I) Derivative Definition


• A derivative is a simultaneous purchase/sale of several puts/calls

• Sometimes the underlying asset is also bought or sold (short)

• You can find a description of two dozen derivatives in the Appendix

• The appendix contains for each derivative: definitions, graphs, and


other details

217
218 CHAPTER 19. DERIVATIVES - GRAPHING METHODS

• You have to memorize or be familiar with all these derivatives

• In this chapter we illustrate the appendix by studying a typical row

19.3 What do you have to know about derivatives


• A typical row of the Appendix gives various attributes of each deriva-
tive

• Their name, graph

• Their definition: That is, what combinations of buying/selling puts/calls


you use to obtain this derivative

19.4 Which properties of derivatives do you have


to know
• In addition to knowing the Names, Definitions, Graphs . . . of
derivatives,

• You must also know their Properties.

• You must know how to infer their properties from their Graphs

• The properties you must know are summarized in the following terms:

• moneyness, profit, loss, volatility, (un)limited profit/loss, superiority

19.5 II) Important Derivative Properties


• Let us now go over each derivative property and explain it

• You make a Profit at a given market (x) value if the y value is positive
(above x-axis)

• The profit/loss is (potentially) unlimited if lim y = ∞ as x goes to


infinity

• You can recognize unlimited profit by a slanting upward line (positive


slope) at the end of the graph
19.6. SUPERIORITY 219

• More precisely, as x goes to infinity the slope eventually becomes pos-


itive

• Using our graphing scheme from chapter 18 this would correspond to


a (nU) at the end of the graph for some integer n

• The profit/loss is capped if there is maximum or minimum

• You can typically recognize caps by a terminal horizontal line, (H), in


the graph description as x goes to infinity

• Note that x is never negative: So an initial slanting-upward line may


cap losses.

• That is, the value of y(0) may be a minimum

19.6 Superiority
• Derivative I is superior to Derivative II at market value x, if yI (x) >
yII (x)

• Profits, caps and limits can typically be done without tables, compu-
tation of numerical values, by general graph methods

• However, superiority may require tables (or at least a few computed


values)

19.7 Volatility
• Volatility refers to changes in market value x

• A volatile stock’s market values will typically move away (left or right)
from the strike price

• A non-volatile stock’s market values will stay near the strike price

• The typical phrase to describe volatility is more volatile than the mar-
ket

• This emphasizes that although the market is volatile you expect higher
volatility in a given stock
220 CHAPTER 19. DERIVATIVES - GRAPHING METHODS

19.8 Moneyness
• Every option can be classified in one of three ways

• At the money, in the money, out of the money

• We can use abbreviations ATM, ITM, OTM

19.9 Moneyness computation


• To classify an option we compare the current market value and strike
price

• If they are about the same the option is ATM

• If selling the option at the current market price would bring a profit,
the option is ITM

• Note that ITM is different for calls and puts (Since their profit regions
are different)

• If the current market price would motivate not exercising the option,
the option is OTM

19.10 Why Moneyness


• Remember in lecture 18 how we used a put for insurance in the Happy
Jalapeno example

• We also sold a call: The short call brought in premium

• This call premium compensated for the loss of premium in buying the
put

• This combo: Buying one option and selling another to offset premium
costs is common

• ITM options have higher premiums (Because you can make money if
the price persists)

• OTM options have lower premiums (Because you can’t make money if
the price persists)
19.11. III) GOOD DERIVATIVE PROBLEMS 221

19.11 III) Good Derivative Problems


• Some derivative problems can be done solely by graphs without com-
putational tables

• Othe graph problems need computational tables

19.12 Good Problems - Graph only


• D-SQ#8

• D-SQ#9

• D-SQ#15

• D-SQ#17

• D-SQ#26

19.13 Good Problems - Require tables


• D-SQ#3

• D-SQ#11

• D-SQ#13

• D-SQ#14

• D-SQ#16

19.14 IV) General Approach to Graphical prob-


lems
• First, we discuss how to graph a derivative without computation

• Use the graph addition method of Lecture 18

• Use the definition of derivatives in the appendix

• Use the definition of the half dozen properties presented above


222 CHAPTER 19. DERIVATIVES - GRAPHING METHODS

• Give the answer based on qualitative properties

• Note: This method is also a good start even when numerical tables
are necessary

19.15 General Approach to computational prob-


lems
• After using the graphical approach mentioned in the last slide,

• then, if necessary, make a table

• However, you might have to compute only 1-2 values

• To make the table, you can break the derivative into component build-
ing blocks

• Then use the database join method of the previous chapter

19.16 Recommended: The fastest computational


graph method
• The fastest computational method does the following

• First compute the graph form using qualitative methods

• Then compute the value of the graph at 0

• Since you know the slope of each graph segment you can quickly
compute any point

• The method is illustrated in Slide 27-29 below

19.17 V) Illustrative Examples


• We do the five graphical problems listed above in order. We start with
D-SQ#8
19.18. D-SQ#8 - SOLUTION 223

• You believe that the volatility of a stock is higher than indicated by


market prices for options on that stock. You want to speculate on that
belief by buying or selling at-themoney options. What should you do?
A. Buy a strangle
B. Buy a straddle
C. Sell a straddle
D. Buy a butterfly spread
E. Sell a butterfly spread

19.18 D-SQ#8 - Solution


• Volatile means you expect stock market values to go away from the
strike price

• Butterflys have graphical form HUDH; Written Butterflys have graph-


ical form HDUH.

• So if you are away from the strike price the graph is horizontal

• That means you would not make a profit if market is very volatile

• Written straddles have graphical form UD,

• This would not give you a profit if market values were away from strike
price

• Only strangles, DHU and straddles DU would give profit if away


from the market price

• According to the appendix, only straddles use ATM options. So the


correct answer is buying a straddle, B

19.19 D-SQ#9 - Solution


• This pretty problem was done in lecture 18

• Notice, that the problem can be done without computing tables

• The published solutions seemed to overlook this

• The qualitative graphical approach advocated in these notes is power-


ful
224 CHAPTER 19. DERIVATIVES - GRAPHING METHODS

19.20 D-SQ#15
• The current price of a non-dividend paying stock is 40 and the con-
tinuously compounded risk-free rate of return is 8%. You enter into
a short position on 3 call options, each with 3 months to maturity, a
strike price of 35, and an option premium of 6.13. Simultaneously,
you enter into a long position on 5 call options, each with 3 months to
maturity, a strike price of 40, and an option premium of 2.78. Assum-
ing all 8 options are held until maturity, what is the maximum possible
profit and loss for the entire option portfolio?

P rof it Loss
A. 3.42 4.58
B. 4.58 10.42
C. U nlimited 10.42
D. 4.58 U nlimited
E. U nlimited U nlimited

19.21 D-SQ#15 - Solution


• This is another problem that unexpectedly can be done soley by graphs

• The first step is to analyze the graph using the graphing method

• This analysis is presented in the next slide

• After doing the problem by graphs we approach it using two other


methods

• We approach it using tables

• We also show how to combine the graphical method with one computed
point at 0 to get the table

• This last approach is very quick and easy

19.22 Method #1: Graphical Analysis


• The graph is HDU
19.23. GRAPHICAL ANALYSIS 225

• The graphical solution showing this is found in the next slide


• The terminal U indicates a potential for unlimited profit as x goes to
infinity
• The vertex of the DU segment is the minimum of the graph
• It represents the maximum loss
• Remarkably only answer C allows maximum profit and capped losses

19.23 Graphical Analysis


M arket value Below 35 Between 35 40 Above 40


Short Call H D D
Short Call H D D
Short Call H D D
Long Call H H U
Long Call H H U
Long Call H H U
Long Call H H U
Long Call H H U

Join Graph H 3D 2U

19.24 Method #2: Table Analysis


• Although we don’t need to, it is good to review table creation
• The techniques were illustrated in the solution to D-SQ#3, Chapter
18
• Step 1: Create two building blocks, the Short call, Long call
• Step 2: Each building block uses the columns in our standard set up
• Step 3: Create the Join table
• Be sure and multiply the short and long calls by 3 and 5
226 CHAPTER 19. DERIVATIVES - GRAPHING METHODS

19.25 The Short Call Table


M arket Strike Buyer P ayof f P remium P remium P rof it


V alue P rice Exercise? t=1 t=0 t=1 t=1
0 35 No 0 6.13 6.25 6.25
35 35 No 0 6.13 6.25 6.25
40 35 Y es −5 6.13 6.25 1.25
100 35 Y es −65 6.13 6.25 −58.75

19.26 The Long Call Table


M arket Strike Buyer P ayof f P remium P remium P rof it


V alue P rice Exercise? t=1 t=0 t=1 t=1
0 40 No 0 −2.78 −2.84 −2.84
35 40 No 0 −2.78 −2.84 −2.84
40 40 No 0 −2.78 −2.84 −2.84
100 40 Y es 60 −2.78 −2.84 57.16

19.27 The Join Table


M arket 1 Short 3 Short 1 Long 5 Long Grand P rof it


V alue Call Calls Call Calls
0 6.25 18.76 −2.84 −14.18 4.58
35 6.25 18.76 −2.84 −14.18 4.58
40 1.25 3.76 −2.84 −14.18 −10.42
100 −58.75 −176.24 57.16 285.82 109.58

• So one can see that the graph minimum occurs at the point y(40) =
−10.42
19.28. METHOD #3: FAST TABLES BY COMPUTING ONLY ONE POINT PLUS GRAPH227

19.28 Method #3: Fast tables by computing only


one point plus Graph
• We already have the graph shape and all relevant slopes

• We only need one point to get a complete numerical graph

• Conveniently that point is 0

• In the next slide we compute the numerical value of y at x = 0

• In the slide following that we compute the derivative minimum of 10.42

• All this is accomplished with one computation

19.29 Computation of y value at x = 0


• Well there are only 2 basic items: Short call and Long call

• The easiest point to compute is at x = 0, since there is option exercise

• For the short call there is no exercise at 0

• So the only contribution of profit is the premium

• Profit at t = 0, is 3 × e.08 × 6.13 = 3 × 6.25 = 18.75

• Note the factor of 3 is because there are 3 short calls

• For the long call there is also no exercise at 0, again, because the graph
is flat

• Profit at t = 0 is 5 × e.08 × −2.78 = 5 × −2.84 = −14.20.

• Hence, the value of the derivative at x = 0 is 18.75 − 14.20 = 4.55

19.30 At x = 40 we have y = −10.42


• How do we compute the minimum

• Remember we have one point on the graph (0, 4.55)

• We know the graph shape HDU or more precisely H3D2U

• The important point is that the graph is a 3-piecewise linear function


228 CHAPTER 19. DERIVATIVES - GRAPHING METHODS

• We also know the slopes

• The turn from H to 3D happens at x = 35

• Since the graph is flat until x = 35 the y value is still 4.55

• We know the slope for the 3D is -3.

• We also know the turning point from D to U happens at x = 40

• The distance from x = 35 to x = 40 is 40 − 35 = 5

• Since the slope is −3 we see that the y value at x = 40 is y = 4.55 −


5 × 3 = −10.42

• This method, one point plus graph form, is generalizable to other


derivatives:

19.31 D-SQ#17
• Assume the current price for a stock index is 1,000, and the following
premiums exist for various options to buy or sell the stock index 6
months from now: Strike Price Call Premium Put Premium

StrikeP rice CallP remium P utP remium


950 120.41 51.78
1, 000 93.81 74.20
1, 050 71.80 101.21

Strategy I is to buy the 1,050-strike call and to sell the 950-strike call.
Strategy II is to buy the 1,050-strike put and to sell the 950-strike put.
Strategy III is to buy the 950-strike call, sell the 1,000-strike call, sell
the 950-strike put, and buy the 1,000-strike put.
Assume that the price of the stock index in 6 months will be between
950 and 1,050. In 6 months, which of the three strategies will have
greater payoffs for lower prices of the stock index than for relatively
higher prices?
A. I only
B. I and II only
C. I and III only
D. II and III only
E. I, II, and III
19.32. D-SQ#17-ANALYSIS 229

19.32 D-SQ#17-Analysis
• (Sketch) We analyze each derivative - I,II,III - using the graphical
methods

• Derivatives I and II have graphs HDH

• Derivative III has graph H

• So clearly, Derivatives I and II make more money when market value


goes down then when up

• For homework please complete the details of this problem


230 CHAPTER 19. DERIVATIVES - GRAPHING METHODS
Chapter 20

Call Put Parity

20.1 Overview
• We do a variety of short topics today

• There are 5 goals, each with several examples

• I) Call-Put Parity

• 1a) Illustrative example: D-SQ#2

• 1b) Illustrative example: D-SQ#5

• 1c) Illustrative example: D-SQ#14

• II) Four Types of Purchase (Review)

• 2a) Illustrative example: D-SQ#29

• III) Dividends and Forward Rates

• 3a) Illustrative Example D-SQ#20

• IV) Hedging

• 4a) Illustrative Example D-SQ#19

• 4b) Quiz/HW: Illustrative Example D-SQ#18

231
232 CHAPTER 20. CALL PUT PARITY

20.2 I) Call-Put Parity


• This is a very important principle. The equation is as follows.


C(K, T ) − P (K, T ) = S(0) − P V (K) (20.1)

• C(K, T ) is the premium for a long call, with strike price K and expi-
ration date T

• −P (K, T ) is the premium for a short put, with strike price K and
expiration date T

• S(0) is the price of the underlying asset, the stock, at t = 0

• P V (K) is the present value, at t = 0, of the strike price, K

• An important assumption of put-call parity is that the underlying


stock has no dividends

• You can however adjust the formula if it does have dividends

20.3 Proof-Right side


• We now verbally prove the important call-put parity formula.

• Right side: I buy stock at t = 0 and pay/lose present value of K at


t=0

• −→ I own stock at t = T and have lost K at t = T.

20.4 Proof-Left Side


• Long Call: I will buy stock and pay K, at time T - if price at T
exceeds K

• Short Put: Buyer (of short) will sell stock and receive K, at time T
- if price at T is less than K

• Short Put: Equivalently, Seller of short (me) will buy stock and pay
K, at T - if price at T less than K

• −→ So, I buy stock at T and pay K whether


20.5. PROOF-EQUALITY 233

• stock price at T exceeds or is less than K, that is,


• In all cases, at time t = T, I buy/own the stock and pay K

20.5 Proof-Equality
• So right side and left side describe the following identical situation
• I own stock and pay K at time T
• −→ Consequently, the cost of the two sides of the equation must be
equal
• The right side deals with stock; the left side deals with options
• This is called the call-put parity principle
• It is a single principle that enables you to solve any situation relating
the following
• premiums of call and put options with the same strike price and
expiration, and the underyling stock, provided no dividends are paid
• Let us now look at some examples.

20.6 D-SQ#2
• You are given the following information:
• The current price to buy one share of XYZ stock is 500.
• The stock does not pay dividends.
• The risk-free interest rate, compounded continuously, is 6%.
• For both the call and put:
• Strike price is K; expiration date is one year; option is European
• Put premium is 18.64
• Call premium is 66.59
• Using put-call parity, determine the strike price, K.
234 CHAPTER 20. CALL PUT PARITY

20.7 Plug in Put Call Parity


C(K, T ) −P (K, T ) = S(0) −P V (K)


66.59 −18.65 = 500 −Ke−0.06

20.8 Solution
• We solve this equation for K

• We obtain K = 480

20.9 D-SQ#5
• You are given the following information:

• One share of the PS index currently sells for 1,000.

• The PS index does not pay dividends.

• The effective annual risk-free interest rate is 5%.

• You want to buy this index in one year for a price of 1,025.

• You do this with put and call options with a strike price of 1,025.

• Describe the transactions (buy/sell); describe cost.

20.10 Application of Put Call Parity


C(K, T ) −P (K, T ) = S(0) −P V (K)


1025
Buy the call Sell the put = 1000 −
1.05
20.11. INTERPRETATION 235

20.11 Interpretation
• Tricky question. Published SOA solutions do not solve this using put-
call parity

• But remember: Put-call parity is all you need when dealing with op-
tions/stocks at same strike price and expiration date

• I should buy the call (long) and sell the put (Short)

20.12 Rule of 6: English Conventions


1025
• The premium cost is 1000 − 1.05 = 23.81

• Why is the answer positive, not negative

• In a table for a graph we always list cost as negative

• However in put call parity the symbols C(K, T ), P (K, T ) represent


premium

• Although premium is a cost and negative in graph tables,

• The actual premium is a positive number

• This is an English convention: You for example say that the cost of a
pen is 10, not -10.

20.13 Stock Index


• DS-Q#5 mentions the index

• What is a stock index?

• Very often you want to know the trend of the market

• For example if stocks are going up you want to buy

• If stocks have been going down for a while you may want to sell short

• But stocks fluctuate alot

• An index is an average of a collection of stocks

• Because it is an average, it is subject to less fluctuation


236 CHAPTER 20. CALL PUT PARITY

• So stock indexes are good indicators of market direction

• Typical stock indexes are the DOW, SP500, Russell

• You can google these to find more information

• For example the DOW is an average of the 30 leading companies

• When people speak about the market trend they may be speaking
about the DOW

20.14 D-SQ#14
• The current price of a non-dividend paying stock is 40 and the con-
tinuously compounded risk-free rate of return is 8%. You are given
that the price of a 35-strike call option is 3.35 higher than the price
of a 40-strike call option, where both options expire in 3 months. How
much does the price of an otherwise equivalent 40-strike put option
exceed the price of an otherwise equivalent 35-strike put option?

20.15 The first 2 equations


C(K, T ) −P (K, T ) = S(0) −P V (K)


C(35, T ) −P (35, T ) = 40 −35e−.02
C(40, T ) −P (40, T ) = 40 −40e−.02

20.16 Two Further algeraic expressions


• To solve this problem we need algebraic expressions for two more En-
glish phrases.

• English phrase The price of a 35-strike call option

• is 3.35 higher than the price of a 40-strike call option

• Algebraic Expression C(35, T ) − C(40, T ) = 3.35


20.17. ALGEBRA 237

• The question asks to solve for the following

• English phrase How much does the price of an otherwise equivalent


40-strike put option exceed the price of an otherwise equivalent 35-
strike put option?

• Algebraic Expression P (40, T ) − P (35, T )

20.17 Algebra
• We have left to solve

• Remember: We are not solving for individual values of C and P

• Rather we are solving for the algebraic expressions, C(35) − C(40) and
P (35) − P (40)

• We start with the first two equations presented two slides ago

• Subtract the two put call equations from each other


   
• C(35, T ) − C(40, T ) − P (35, T ) − P (40, T ) = 5e−.02
 
• −→ P (40, T ) − P (35, T ) = 5e−.02 − 3.35 = 1.55

• Note that the expiration date is 3 months, one quarter of a year


0.08
• So the continuous interest rate for 3 months is 4 .

20.18 II) Four Types of Purchase (Review)


N ame W hen you pay W hen you receive security P rice F ormula
P urchase 0 0 S0
Leveraged P urchase T 0 S0 ert
P
P repaid F orward 0 T S0 = F0,T
F orward T T S0 ert = F0,T
238 CHAPTER 20. CALL PUT PARITY

20.19 D-SQ#29
• Good problem.

• Illustrates computations with four methods

• It is a straightforward plug in

• So we will not cover it here;

• I encourage you to do it by yourself

• Compare your answer with the published solution

• You should complete the lecture, especially the part about Dividends
prior to attempting this problem

20.20 Dividends and Forward Rates


• In this slide we present the definition of dividend

• A dividend is a certain amount you get from the stock corporation

• Why? Because you bought stock and hence are a co-owner in the
corporation

• Hence you should share in profits

• Your share in profits as owner is called dividends

20.21 Dividend timing


• Dividends are frequently paid quarterly (at end of quarter)

• They can also be paid annually

• They can even be paid continuously


20.22. DIVIDEND REINVESTMENT 239

20.22 Dividend Reinvestment


• Many stocks allow automatic reinvestment of monetary dividends
• Here is an example of how this works
• Suppose the stock is worth 100 and pays a dividend of 10 every year
• Then you have the option of automatically converting this 10 into new
stock
• So in our example you would own 1.1 shares of stock (worth 110)
• The (dis)advantage of owning stock versus money is that the stock
may go up/down
• A further consequence of automatic reinvestment is that you automat-
ically acquire the stock
• Normally, to acquire or sell a stock you have to pay the stock broker
a commision
• Commision is an expense
• The commission is the payment to the broker for his work in buying
or selling the stock for you
• When you automatically reinvest you don’t have to pay any commi-
sions
• Not paying commisions saves you money.

20.23 Continous and discrete dividend rates


• Let us use the example just given
• The stock is worth 100 and pays one dividend of 10 every year
• We could say that the dividend rate is 10%
• This would imply that the dividend in year 2 is 11, not 10 (Why?)
• If the dividend rate is continuous then we would use the continuous
rate 9.53% (Why?)
• We will present an example below using discrete quarterly dividends
240 CHAPTER 20. CALL PUT PARITY

20.24 Interest Rates


• When dealing with stocks and derivatives the symbol traditionally
used to indicate interest is r, not i

• r is the rate at which money grows

• The symbol δ is the traditional symbol for the dividend rate

20.25 Confusion between interest and derivative


portion of course
• In the interest theory portion of this course

• we used the symbol i, δ to indicate discrete and continous com-


pound rates respectively

• In the derivative portion of this course

• r, δ indicate continuous monetary interest and continuous divi-


dend interest

20.26 Risk free rate vs. Dividend rate


• We have two interest rates

• An interest rate indicating growth of money

• An interest rate indicating growth of stock

• The interest rate indicating growth of stock has an element of risk

• If the stock goes up you get more dividends; if the stock goes down
you get less

• But the interest rate indicating growth of money has no element of


risk

• It indicates how the money grows

• Traditionally the rate of monetary growth is called the risk free rate
20.27. FAIRNESS ISSUES 241

20.27 Fairness issues


• In a Forward contract I receive the stock at time T

• So the seller (you) has the stock until time T

• So the seller (you) receives the Dividends

• The Dividends are part of the stock deal, for example, there might be
automatic reinvestment

• Even without automatic reinvestment the Dividend Discount Model


relates Dividends to the Stock Price

• You are already receiving the dividends

• Therefore, fairness requires we subtract the present value of dividends


from the forward price

• Otherwise, the seller would receive the value of the dividends twice.

20.28 Forward Rates, F0,T


• We now compute, using the above discussion, the fair price for a for-
ward contract at time T, F0,T

• First suppose there are no dividends

• If the stock is worth S0 now it should be worth F0,T = S0 erT at t = T

• The stock price grows by the risk free interest rate, r, governing money
growth

20.29 Forward Rates, Continued


• Now suppose there are dividends

• The stock value grows by the risk free rate to S0 erT

• But we subtract from the stock the dividends because the buyer al-
ready received them

• There are two cases to consider


242 CHAPTER 20. CALL PUT PARITY

• 1) If the dividend rate is continuous then F0,T = S0 erT e−δT

• 2) If the dividends are discrete then simply subtract

• F0,T = S0 ert − ert Dt v t


P

• The sum is taken over all times t when dividends are given

Dt v t can be evaluated by annuity methods


P
• This sum

20.30 D-SQ#20
• The current price of a stock is 200, and the continuously compounded
interest rate is 4%. A dividend will be paid every quarter for the next
3 years, with the first dividend occurring 3 months from now. The
amount of the first dividend is 1.50, but each subsequent dividend will
be 1% higher than the one previously paid. What is the fair price of a
3-year forward contract on this stock?

20.31 Solution
• Break up the price into two parts

• i) Growth of stock

• ii) Subtraction of Dividends

20.32 Growth of Stock


• δ = 0.04, n = 3 −→ 200e0.04×3 = value of stock at t = 3

• Note this gives the value of the stock without consideration of divi-
dends

• We must subtract the dividends


20.33. TIMELINE - ANNUITY 243

20.33 Timeline - Annuity


1.5 1.50(1.01) 1.50(1.01)2 1.50(1.01)3 ...


0 1 2 3 4 ...

20.34 Inflation Annuity Equivalent Timeline


• The above is equivalent to the following timeline at the modified (in-
flation adjusted) rate i0

1.5 1.5 1.5 1.5 ...


0 1 2 3 4 ...

• The modified (inflation adjusted) interest rate can be found by solving


1 1.01
1+i0 = e0.01

• We compute the present value of the 12 dividends using the rule of 3

• e−.01 ä12 i0 × 1.50

• Conversion Note that δ = 0.04;

• So continuous rate per quarter is 0.01.

20.35 Putting it all together


• The annuity gives us the present value

• But we want the Future Value at t = 3

• So we must multiply by e3r

• F0,T = 200e.04×3 − e0.04×3 e−.01 ä12 i0 × 1.50


244 CHAPTER 20. CALL PUT PARITY

20.36 Numbers
• i0 = 0.004967%

• 200e.04×3 = 225.50

• ä12 i0 × 1.50 = 17.995

• e0.04×3 e−0.01 ä12 i × 1.50 = 20.08

• F0,T = 225.50 − 20.08 = 205.41

20.37 IV) Hedging


• What is a Hedge

• First think of a physical hedge

• The hedge acts as a buffer

• It prevents you from walking and trampling a garden

• An economic hedge also acts as a buffer

• It prevents you from excessive lost

20.38 Hedging-Options
• One type of hedging is insurance hedging

• You buy calls or puts to cap your losses

• These are called floors since it places a floor on losses

20.39 Hedging Probability


• There are hedges based on probability distributions

• You assign probabilities to various market values

• You compute a profit/loss for each such market value


20.40. D-SQ#19 245

• You use the probability distribution to compute an average or expected


loss/profit

• The technique is best illustrated through examples

20.40 D-SQ#19
• You are a producer of gold, and have expenses of 800 per ounce of gold
produced. Assume that the cost of all other production-related expenses
is negligible, and that you will be able to sell all gold produced at the
market price. In 1 year, the market price of gold will be in 1 of 3
possible prices, corresponding to the following probability table: Gold
will either be 750 an ounce with probability 20%, 850 an ounce with
probability 50% or 950 an ounce with probability 30%. You hedge the
price of gold by buying a 1-year put option with an exercise price of 900
per ounce. The option costs 100 per ounce now, and the continuously
compounded interest rate is 6%. Which of the following is closest to
your expected 1-year profit per ounce of gold produced?

20.41 Solution Overview


• We will make 3 tables

• 1) Gold Profit table

• 2) Put Profit Table

• 3) The Combined Join Table

• We then will take a weighted-expected Average

20.42 Probability and Table


• One new column is added to the tables: Probability

• In the final table we take two columns

• The combined profit column

• The probability column


246 CHAPTER 20. CALL PUT PARITY

• We take the sum of the products of these two columns

• This gives us a weighted average or expected profit

• It indicates on the average what to expect

20.43 Gold Profit Table


Gold P rice P robability of price Cost P ayof f = P rice P rof it


T =1 T =1 T =1 T =1
750 0.2 −800 750 −50
850 0.5 −800 850 50
950 0.3 −800 950 150

20.44 Put Profit Table


Gold P rice Exercise P ut? Strike P ayof f P remium P rof it


T =1 T =1 T =1 T =1
750 Y es 900 150 −106.18 43.82
850 Y es 900 50 −106.18 −56.18
950 No 0 0 −106.18 −106.18

20.45 Combined Table



Gold P rice P robability of P rice P rof it on Gold P rice P ut P rof it P rof it T ot P rof it ∗ P rob
T =1 T =1 T =1
750 0.2 −50 43.82 −6.18 −1.24
850 0.5 50 −56.18 −6.18 −3.09
950 0.3 150 −106.18 43.82 13.14

20.46 Expected Average


• We go to the combined table

• We now use the Profit and Probability columns


20.47. GOOD HW EXERCISE 247

• We multiply each profit by its probability

• This product is found in the last column

• We add up these products to obtain the expected (average) profit

• −1.24 − 3.09 + 13.14 = 8.82

• On the average one can expect to make 8.82

20.47 Good HW Exercise


• D-SQ#18 resembles D-SQ#19

• Try doing it yourself using D-SQ#19 as a model solution


248 CHAPTER 20. CALL PUT PARITY
Chapter 21

Arbitrage

21.1 Overview
• We are covering one topic today, Arbitrage

• We have the following 8 subgoals

• I) Simple Pen Arbitrage Example

• II) Revu of Stock Dividends

• III) Stock Arbitrage Overview

• IV) Stock Arbitrage - Fair Forward Price less than Actual Market
Price

• V) Terminology

• VI) Stock Arbitrage - Fair Forward Price more than Actual Market
Price

• VII) Summary comparison of stock arbitrages

• VIII) Exercises

249
250 CHAPTER 21. ARBITRAGE

21.2 I) Simple Pen Arbitrage Example


• Suppose

• Company A sells pens for 10 per pen

• Company B sells pens for 7 per pen

• The same object, pens, are being sold for two prices

• That allows you, the investor to arbitrage and make money

• How?

21.3 Arbitrage Tableau


T ime Activity M oney consequence Consequent N eed


t = 0 Short A, 1 pen Receive 10 N eed pen to cover short
t=0 Long B, 1 pen Receive pen, pay 7 N one
t=1 N one (10 − 7)(1 + i) prof it Cover short, Give A pen

21.4 Arbitrage Definition


• Arbitrage is a guaranteed profit without any money expenditure

• It typically occurs because of 2 prices for the same object

• The strategy is to sell high and buy low

• You implement strategy with longs and shorts

21.5 Tableau technique


• The table in slide 21.3 is called an arbitrage Tableau

• Its purpose is to facilitate the view of the flow of tranactions

• My tableaus use time and need columns


21.6. II) REVU OF STOCK DIVIDENDS 251

• You start at t = 0 by selling high

• Each row creates a need

• That need is met by the following row

21.6 II) Revu of Stock Dividends


• Lets revu stocks, futures and dividends

• The stock price at time t = 0 is S0

• The Forward price at time t (if there are no dividends) is S0 ert

• Here r is the risk free continuous rate currently offered by the Market

21.7 Revu of Dividends - Continued


• Now suppose the stock issues dividends

• Recall we subtract dividend worth since the seller owned the stock till
t

• So the Forward price is:


F0,t = S0 ert e−δt (21.1)

• Here δ is the current market continous dividend rate

• We will refer to this formula as the The Forward Formula

• If r and δ are the current market rates we call the resulting forward
price,Fair

• A similar formula holds if dividends are discrete


252 CHAPTER 21. ARBITRAGE

21.8 Revu of Dividend Reinvestment


• Suppose S0 = 100, dividends are discrete, with discrete rate 10%

• At t = 1, I have two options

• a) I can receive a monetary dividend 10% × 100 = 10

• b) I can have the dividend value automatically reinvested

• So at t = 1, I have 1.1 shares of stock.

• Remember: Automatic reinvestment allows purchase of stock without


paying commissions

21.9 Revu of Dividend Reinvestment Continued


• Why would I choose reinvestment of dividends?

• Suppose the risk free rate is 8% but

• The stock growth rate is 9%

• By reinvesting in stock I earn more than by keeping cash

21.10 III) Stock Arbitrage Overview


• In the real world you typically do not find stocks sold at two different
prices

• But the market may offer a forward price different than the fair forward
price

• So you have two prices for the same object: actual market vs. fair
21.11. STOCK ARBITRAGE - FAIR FORWARD PRICE LESS THAN MARKET FORWARD PRICE253

21.11 Stock Arbitrage - Fair Forward Price less


than Market Forward Price
• Step I: We deal with the following hypothetical problem.

• Current Stock Price 1000

• Market Forward Price 965

• Current market risk free rate is r = 2%

• T = 0.5

• Current market dividend rate is δ = 10%

21.12 Step 2: Test for Arbitrage


• Actual market forward price: 965

• Fair forward price: 960.79

• Actual market forward price is higher than fair forward price

• Two different prices for same entity −→ arbitrage

21.13 Arbitrage Tableau for Dividend Example


T =0 T =0 T =0 T = 0.5 T = 0.5
Action Have/P rof it N eed/Owe P rof it/Have N eed/owe

Buy Stock e−δ T shares −S0 e−δ T $ 1 share (W hy) NA


Borrow S0 e−δ T $ −S0 e−δ T $ N A −S0 e−δ T erT $
Short F orward NA NA F0,T $ −1 Share
254 CHAPTER 21. ARBITRAGE

21.14 Step 4: Profit-Cost analysis


• Cost: I spent nothing at t = 0

• Profit: At t = 0.5, I profitted F0,T −S0 e−δT erT = Actual M arket F orward−
Synthetic F air F orward = 965 − 960.79 = 4.21

• Summary: Cost = 0, P rof it = 4.21 −→ arbitrage

21.15 V) Terminology
• We call this arbitrage a cash and carry arbitrage

• At time T = 0 we borrow cash

• At time T = 0 we carry the 1 share of stock to offset the short


forward

• The first two rows of the tableau are called a synthetic long forward

• You can check that the two rows together have the characteristics
of an actual long forward

• That is, at T = 0.5 you have 1 share of stock and must pay the
forward price S0 e−δT erT

• Using the concept of a synthetic long forward we can neatly explain


the cash and carry arbitrage

• In the pen example, we shorted a pen high and longed a pen low
and profited on the difference

• Similarly in the cash and carry example, we longed a synthetic


fair forward, low, and shorted the actual market forward, high

• In each case the profit is the difference between the high short
and the long low.
21.16. SYNTHETIC FORWARDS 255

21.16 Synthetic Forwards


• At T = 0.5 I have 1 share and paid S0 erT e−δT = 960.79
• This is just like a forward where I buy one share at T = 0.5
• We call this a synthetic forward, because it is made up of two sep-
arate transactions
• You cannot however do this in one step. Why?
• Because no one would sell you one share for 960.79 when they can get
the higher actual market price of 965
• So you need to do it in two steps - that is, you need to synthetically
create a forward

21.17 More on cash and carry


• Cash and Carry Arbitrage: by definition means an arbitrage that
buys the underlying asset and sells/shorts a forward
• You use a cash and carry when Actual market f orward price >
Synthetic F air F orward price
• The next example is a reverse cash and carry
• You use a reverse cash and carry when Actual market f orward price <
Synthetic F air F orward price

21.18 VI) Stock Arbitrage - Expected Price more


than Actual Forward Price
• We use the following information
• Current Stock Price 1000
• Market Forward Price 965
• Market risk free rate is r = 2%
• T = 0.5
• Market dividend rate is δ = 8%
256 CHAPTER 21. ARBITRAGE

21.19 Step 1: Predicted (Expected Dividend) For-


ward Price
• Forward Formula: F0,T = S0 erT e−δT

• We plug in with the market rates

• Numerically: F0,T = 1000 × e.02×0.5 × e−0.08×0.5 = 970.45

• 970.45 is the fair forward rate

21.20 Step 2: Test for Arbitrage


• Actual market forward price: 965

• Fair forward price: 970.45

• Actual market forward price is lower than the fair forward price

• Two different prices for same entity −→ arbitrage

21.21 Step 3: Arbitrage Tableau - Actual market


forward price > Fair forward price
• See the tableau at the bottom of the slide

• Row 3 corresponds to buying the lower price market forward

• This is similear to buying the lower price pen)

• Rows 1 and 2 correspond to selling (short) the higher fair forward

• Indeed: At T = 0.5 in rows 1,2 I have received money and owe one
share stock

• But you cannot do rows 1 and 2 in one step you need two rows

• Why? Because no one would pay me 970.45 for 1 share when they can
buy this share in the market for 965

• We therefore call rows 1 and 2 a synthetic short forward

• The entire table is called a reverse cash and carry


21.22. STEP 4: PROFIT-COST ANALYSIS 257

T =0 T =0 T =0 T = 0.5 T = 0.5
Action Have/P rof it N eed/Owe P rof it/Have N eed/owe

Short Stock +S0 e−δ T $ −e−δ T shares See next row −1 Share
Lend NA −S0 e−δ T $ +S0 e−δ T erT $ NA
Long F orward NA NA +1 Share −F0,T $

21.22 Step 4: Profit-Cost analysis


• Cost: I spent nothing at t = 0

• Profit: At t = 0.5, I profitted S0 e−δT erT −F0,T = F air synthetic F orward−


Actual M arket F orward = 970.45 − 965 = 5.45

• Summary: Cost = 0, P rof it = 5.45 −→ arbitrage

21.23 VII) Summary comparison of stock arbitrages


• Notice the oppositeness of corresponding rows in the two stock arbi-
trage tableaus

• For example let us look at row 2

• It is lend for the reverse cash and carry

• It is borrow for the cash and carry

• Similarly rows 1 and 3 illustrate long vs. short positions.

21.24 VIII) Exercises


• Suppose I had not given dividends
258 CHAPTER 21. ARBITRAGE

• I just gave S0 , r and your expected price

• The above tableaus would still apply

• You could still arbitrage based on F Actual M arket and F Synthetic F air

• See D-SQ#21 which is a good arbitrage problem

• The published solutions do not use an arbitrage tableau

• Please construct an arbitrage tableau in doing this example

• Here is a further simple stock arbitrage example

• Class exercise: F0,1Actual M arket = 100, F Synthetic F air = 110. How


0,1
would you arbitrage
Chapter 22

Swaps

22.1 Overview
• Today we cover swaps

• We have the following 7 subgoals in this chapter

• I) Swaps - basic definition

• II) Illustrative Example D-SQ#22

• III) Illustrative Example Modified D-SQ#22

• IV) Swap Summary - Full Method

• V) Swaps - Commodities vs. Forward rates

• VI) Illustrative Example D-SQ#23

• VII) Discount Factors: 3 Methods

22.2 I) Swaps - basic definition


• A swap refers to an actuarially equivalent payment scheme

• Typically swaps refer to actuarially equivalent level payment schemes

• Swaps are used to hedge (avoid losses from) varying prices

259
260 CHAPTER 22. SWAPS

22.3 Swaps - Basic Timeline Approach


• Given: You are given a series of market forward payments for a com-
modity at a future time

• Given: You are also given a collection of spot rates (n-year interest
rates from t = 0, today)

• To find: An actuarially equivalent series of level payments

• Method: You can do this using a simple Equation of Timelines

22.4 II) Illustrative Example D-SQ#22


• A farmer expects to sell 50 tons of pork bellies at the end of each of
the next 3 years.

• Suppose that the pork bellies forward price for delivery

• in 1 year is 1,600 per ton.

• in 2 years, the forward price is 1,700 per ton.

• in 3 years, the forward price is 1,800 per ton.

• Suppose that interest rates are determined from the following table:

• r1 = 5%, r2 = 5.5%, r3 = 6%

• If the farmer uses a commodity swap to hedge the price for selling
pork bellies,

• what is the level amount he would receive each year (i.e. the swap
price)

• for all 50 tons?


22.5. SOLUTION - TIMELINE #1 261

22.5 Solution - Timeline #1


• We use an Equation of Timeline method

• The first timeline is the timeline for the actual prices

1600 1700 1800


0 1 2 3
r1 = 5% r2 = 5.5% r3 = 6%

22.6 Solution - Timeline #2


• The second timeline is the timeline for the actuarially equivalent level
price

X X X
0 1 2 3
r1 = 5% r2 = 5.5% 53 = 6%

22.7 Solution - Swap equation of Value


• X represents the unknown level swap value sought

• We want timeline #2 to be actuarially equivalent to timeline #1

• EOV : Timeline #2 = Timeline #1

22.8 Solution - Algebraic Details on Swap equa-


tion of Value
• But the EOV for Timeline #1 is: P V1 = 1600v1 + 1700v22 + 1800v33

• The EOV for Timeline #2 is: P V2 = Xv1 + Xv22 + Xv33


262 CHAPTER 22. SWAPS

• Here vi , i = 1, 2, 3 is the present value or discount factor

• The relationship between vi and ri is given by the following basic


equation


1
vii = = Price of a 1 dollar n-year zero coupon bond (22.1)
(1 + ri )i

22.9 Swap Price for 1 vs. 50 Tons


• Plugging in and solving we obtain X = 1695.81

• This is per ton. So the swap price for 50 tons is 50 × 1695.81 =


84790.38.

• 50 is called the notional amount

22.10 Operational Details


• In the above I have simply numerically solved the problem

• Operationally swaps have very specific mechanisms

• Swaps typically happen through a 3rd party

• Just as an exchange is a 3rd party that allows stocks and options to


be traded,

• So to swap transactions are facilitated by a 3rd party

• The following terms are also relevant: prepaid swap, back-to-back trans-
action, swap term, swap tenor, asset swap, swap spread, deferred swap,
accretizing swap, amortizing swap, swaption

• You can get a good idea of their meaning by googling them

• We are focusing on the numerical solutions and mathematical methods


in this chapter

• We also briefly mention LIBOR and related matters below.


22.11. III) ILLUSTRATIVE EXAMPLE MODIFIED D-SQ#22 263

22.11 III) Illustrative Example Modified D-SQ#22


• To illustrate the importance of notional amounts we modify the above
problem

• Instead of wanting 50 tons per year for all years we suppose we want

• 10 tons the first year

• 20 tons the 2nd year

• 30 tons the 3rd year

• How would the solution be different.

22.12 Notional Amount


• In the unmodified D-SQ#22 we first solved for the amount per ton

• Since we wanted 50 tons for each year, we multiplied the amount per
ton by 50

• This (50) is called the notional amount

22.13 Solution Method for modified D-SQ#22


• We still use an equation of two timelines

• We still use the relation between present value, vi and spot rates ri

• However we must include the notional amounts in the two timelines


as follows

22.14 Solution - Timeline #1


1600 × 10 1700 × 20 1800 × 30


0 1 2 3
r1 = 5% r2 = 5.5% r3 = 6%
264 CHAPTER 22. SWAPS

22.15 Solution - Timeline #2


X × 10 X × 20 X × 30
0 1 2 3
r1 = 5% r2 = 5.5% 53 = 6%

22.16 Solution - Algebraic Details on Swap equa-


tion of Value
• We now modify the EOV to reflect the amounts

• The EOV for Timeline #1 is: P V1 = 1600 × 10v1 + 1700 × 20v22 +


1800 × 30v33

• The EOV for Timeline #2 is: P V2 = X × 10v1 + X × 20v22 + X × 30v33

• Plugging in we obtain P V1 = 91124.49, P V2 = 52.681436X −→ X =


1729.73

22.17 Summary of solutions


• In the unmodified D-SQ#22,

• the basic amount per ton is 1695.81

• the level amount to be paid each year is 1695.81 × 50 = 84790.38

• In the modified D-SQ#22, . . .

• the basic amount per ton is 1729.73

• The amounts paid at t = 1, 2, 3 respectively are

• 1729.73 × 10, 1729.73 × 20, 1729.73 × 30

• In each case the basic amount is multiplied by the notional amounts


22.18. IV) SWAP SUMMARY - FULL METHOD 265

22.18 IV) Swap Summary - Full Method


• A swap is an attempt to find an actuarially equivalent (level) value

• Method memorization is superior to formula memorization here

• To solve a problem set up a timeline with appropriate forward rates

• Multiply by notional amounts if necessary

• Then set up an identical timeline with level amounts, X

• Again multiply by notional amounts if necessary

• Convert spot rates to present values and solve for the basic amount X

• If necessary multiply by notional amounts to obtain future payment


rates

22.19 A subtltety
• D-SQ#4 presents an interesting problem

• It points out that definitionally a swap can refer to any actuarially


equivalent sequence of payments

• This is true

• But I feel it misleading (I don’t think the problem is good)

• The standard use of a swap is to find an actuarially equivalent sequence


of level payments

22.20 V) Swaps - Commodoties vs. Forward rates


• In the previous example the concept of swap - actuarially equivalent
level payments,

• This concept, was applied to a sequence of payments for commodoties

• One can also apply the concept of swap - actuarially equivalent level
payments,
266 CHAPTER 22. SWAPS

• One can apply this concept, to a sequence of forward rates

• The mathematics is identical

• The objects to which the swap method is applied are different

22.21 Swap - Forward Rates


• In a commodity swap the t = i value is the expected cost of the
commodity at t = i, the forward rate for the commodity

• In an interest swap the t = i value is the expected one-year forward


rate fi−1,i

• It is sometimes convenient to equivalently use the one-year forward


rate factor 1 + fi−1,i

• It is instructive to work out why use of the factor vs. rate is eqiuvalent

22.22 Unusual Feature


• Here we have an unusual feature: The spot rates are used for two
purposes
 i
i 1
• 1) They are used to calculate present values: vi = 1+ri

• 2) They are used to calculate forward rates: (1 + ri−1 )i−1 (1 + fi−1,i ) =


(1 + ri )i

22.23 VI) Illustrative Example D-SQ#23


• You are given the following spot rates from the latest upward-sloping
yield curve:

• r1 = 4.00%, r2 = 4.50%, r3 = 5.25%, r4 = 6.25%, r5 = 7.50%

• You enter into a 5-year interest rate swap (with a notional amount of
100,000)
22.24. LIBOR 267

• to pay a fixed rate and to receive a floating rate based on future 1-year
LIBOR rates.

• If the swap has annual payments, what is the fixed rate you should
pay?

22.24 LIBOR
• LIBOR is an acronym for London Interbank Offered Rate

• The Libor rate is the average interest rate that leading banks in Lon-
don charge when lending to other banks.

• In other words each bank borrowing money from another bank pays
the lending bank interest

• The LIBOR is the average of these rates among major banks and is
published

• Many financial entities use the LIBOR rates as a benchmark for their
own rates (which may be higher)

22.25 LIBOR and T-Bill rates


• We often in this course, speak about the interest rate

• But how does one find the interest rate

• The LIBOR is one way that the interest rate is established

• Another method is to use the rates of United States Treasury Bills


(T-Bills)

• The US routinely loans money (zero coupon bonds)

• The maturity value can be 30 days, 90 days, and several other values
up to 10 years

• The 10 year loans of the US government are called bonds

• The loans with smaller maturity, such as 30 and 90 days, are call
T-bills

• The underlying mathematics is the same as any zero coupon bond


268 CHAPTER 22. SWAPS

• It is an instructive exercise to look up current t-bill rates and LIBOR


rates

• Because of the confidence in the US dollar and the British economy


these rates are respected as the current market rates

22.26 Preliminaries to solving the problem


• The notional amount of 100,000 does not matter

• Why? It is not varying and gets cancelled from both sides of the
equation

• The LIBOR rates also don’t enter the problem

• Instead, regard the problem abstractly as a pure swap

22.27 Forward rates


• We need to compute the forward rates

• (1 + f0,1 ) = (1 + r1 ) −→ f0,1 = r1

(1+r2 )2
• (1 + r1 )1 (1 + f1,2 ) = (1 + r2 )2 −→ 1 + f1,2 = 1+r1 = 1.050024 −→
f1,2 = 5%

• Other forward rates are computed similarly

• We could have also just calculated the factors

22.28 Solution - Timeline #1


4% 5.002% 6.766% 9.307% 12.649%


0 1 2 3 4 5
r1 = 4% r2 = 4.5% r3 = 5.25% r4 = 6.25% r5 = 7.50%
22.29. SOLUTION - TIMELINE #2 269

22.29 Solution - Timeline #2


X X X X X
0 1 2 3 4 5
r1 = 4% r2 = 4.5% r3 = 5.25% r4 = 6.25% r5 = 7.50%

22.30 EOV
• P V1 = 4%v1 +5.002%v22 +6.766%v33 +9.307%v44 +12.649%v55 , vii =
1
(1+ri )i

1
• P V2 = Xv1 + Xv22 + Xv33 + Xv44 + Xv55 , vii = (1+ri )i

• P V1 = P V2 −→ X = 7.20%

• Note: The published solutions give a computational shortcut; however,


my goal was to emphasize the underlying concepts; but if you are
studying for an SOA exam where time matters you should master this
shortcut for this problem

• The appendices contain a worked out solution to D-SQ#23

22.31 VII) Discount factor: 3 Methods


• Computations require present values vii

• There are 3 ways present values can be given


1
• 1) vii = (1+ri )i

• 2) Let Pi denote the price of an i-year zero coupon bond with a re-
demption of 1 dollar

• Then Pi = vii (Why?)

• 3) vi−i = (1 + r1 )(1 + f1,2 ) . . . (1 + fi−1,i )


270 CHAPTER 22. SWAPS

• To formulate a solution to a problem one should always use equations


of values with vii and then convert as appropriate
Chapter 23

Appendix

23.1 Derivative Tables


• A derivative is a combination of calls,puts,buys, shorts and forwards

• Your goal in a derivative is too know . . .

• 1) The derivative name

• 2)Which calls, puts and buys are used

• 3) The derivative Graph

• 4) The derivative characteristics

23.2 Derivative Characteristics


• The derivative characteristics can always be derived from the graph

• We have discussed these characteristics in the Chapter on Graphs

• A derivative is profitable if y values are greater than 0

• A derivative has losses if y values are below 0

• A derivative has a cap if the limit of y as x goes to infinity is constant

• Other definitions were presented in the Chapter on Graphs

271
272 CHAPTER 23. APPENDIX

N ame Graph StrickP rices Option#1 Option#2 Option#3 Option#4


F loor HU K1 Asset P (K)AT M ?
Cap DH K1 AssetShort C(K)AT M ?
CoveredCall UH K1 Asset -C(K) ATM?
Coveredput HD K1 Assetshort -P(K) ATM?
Collar DHD K1 < K2 +P(K1) -C(K2)
W rittenCollar U HU K1 < K2 -P(K1) +C(K2)
Collaredstock HU H K1 < K2 Asset(K1???) +P(K1) ATM? -C(K2)
0-Cost Collar DHD K1 < K2 +P(K1) -C(K2)

Table 23.1: Insurance Derivatives

N ame Graph StrickP rices Option#1 Option#2 Option#3 Option#4


Iron Butterf ly HU DH K1 < K2 < K3 +P(K1) OTM -P(K2) ATM -C(2) ATM +C(K3) OTM
Long Call butterf ly HU DH K1 < K2 < K3 +C(K1) -2C(K2) ?K2 ATM +C(K3)
Short Call butterf ly HDU H K1 < K2 < K3 -C(K1) +2C(K2) ?K2 ATM -C(K3)
Long P ut Butterf ly HU DH K1 < K2 < K3 +P(K1) -2P(K2) ?K2 ATM +P(K3)
Short P ut Butterf ly HDU H K1 < K2 < K3 -P(K1) +2P(K2) ?K2 ATM -P(K3)
Collaredstock K1 < K2 Asset(K1???) +P(K1) ATM? -C(K2)
0-Cost Collar K1 < K2 +P(K1) -C(K2)

Table 23.2: Butterfly Derivatives

23.3 Notation
• Graph Column: H = Horizontal, U = U P, D = DOW N, (Line seg-
ments left to right)

• ATM means AT THE MONEY; OTM means out of the money; ITM
means In the Money

• A question mark indicates lack of certainty on an attribute

• C(K) = C(K, T ), P (K) = P (K, T ) These refer to premiums with


common expiration dates

• a plus sign indicates buy, long, or purchase

• a negative sign indicates sell, short or write

23.4 Long Asymmetric Butterfly


• We omitted the long Asymmetric butterfly from the Butterfly table
23.4. LONG ASYMMETRIC BUTTERFLY 273

N ame Graph StrickP rices Option#1 Option#2 Option#3 Option#4


Long Bull CallSP READ HU H K1 < K2 +C(K1) -C(K2)
Long Bull P utSP READ HU H K1 < K2 +P(K1) -P(K2)
Long Bear CallSP READ HDH K1 < K2 -C(K1) +C(K2)
Long Bear P utSP READ HDH K1 < K2 -P(K1) +P(K2)
Box Spread HHH K1 < K2 +C(K1) -C(K2) -P(K1) +P(K2)
Ratio Spread K1 <?K2 Buy m + C(K1) Sell n − C(K2)
Straddle DU K1?AT M ?? +C(K1) +P(K1)
W rittenStraddle UD K1?AT M ?? -C(K1) -P(K1)
Strangle DHU K1, K2 +P(K1) OTM +C(K2) OTM

Table 23.3: Spread Derivatives

• Its definition is complicated

• Its graph is HUDH but the UD part is not symmetric (hence the name)

• There are three strike prices: K1 < K2 < K3

• K2 is a weighted average: K2 = L × K1 + (1 − L) × K3

• You create the asymmetric butterfly by

• Buying n × L puts at strike price, K1

• Shorting n puts at strike price, K2

• Buy n × 1 − L puts at strike price K3


274 CHAPTER 23. APPENDIX
SOLULTIONS TO SELECTED PROBLEMS
The following pages contain worked out solutions to SOA problems using the methods in the
lecture notes. I plan to have a complete set of solutions using a uniform notation for version 3 by
the end of the summer or year.

Please find on the next page a table of contents. To illustrate use of the table: We see that
problem M00#26, is illustrative of Loan problems. It particularly illustrates the technique of
writing down examples of the first few rows and detecting a pattern (in contrast to other
problems where you plug in a formula)

Thoughout these problems the following applies

 The reader must look up the problem prior to reading the solution
 The abbreviations used follow the abbreviations listed in the introduction
 The problems themselves are copyright SOA and are reprinted with
permission
 The SOA also has copyrighted solutions
 My solutions reflect my methods; sometimes all I have added to the SOA
solutions are timelines and calculator TV lines; sometimes (as in inflation
problems) my approach is totally different
 My own solutions are copyright (like the rest of these notes; that simply
means that anyone using the notes should retain identification of the
author (me) and not use them for monetary gain)
 The solutions are not direct to the answer; rather they illustrate the
thinking process that a student must go through to arrive at an answer.
This includes choosing between competing methods. It also includes
fundamental items that must be memorized.
Area Problem Topic-Subtopic
Time Value of Money M01#49 Force – Nominal –
Discount
QIT#1 Force – Nominal –
Comparison

Dollar Weighting QIT#5 Dollar Weighting

Annuities QIT#2 Annuity-Comparison-


Difference of Squares
QIT#6 Annuity-Perpetuity-
Increasing-Algebra
M03#45 Annuities-Refinancing-
Inflation

QIT#93 Annuities-Level yearly,


increasing monthly

Loans N00#34 Loans – Refinancing


M00#24 Refinancing – Deferral

M00#26 Loans – Examples /


Pattern Seeking
M03#49 Loans – Examples /
Pattern Seeking

N05#16 Loans – Reinvestment


QIT#7 Loans – Reinvestment
(Decreasing)

QIT#9 Amortization – 13
Formulae – Examples /
Patterns – English
Conventions

Bonds QIT#10 Bonds – 13 Formulae –


Emglish Conventions
QIT#74 Bond Comparison

QIT#55 Callable Bonds

Swaps DSQ#23 Interest Swaps


Solution to M01#49,

( c ) Dr Hendel Sp 2015, Solutions Follow Dr Hendel’s approach


Tawny Timeline

Time 0 1 2 3 5
A(t) X X(1.05)2t
Express as
function of t=5

Fabio Timeline

Time 0 1 2 3… 5
A(t) 1000 1000(1+ti) = Z

We have

To do the problem you need to use the following force formula

However you can’t use that formula unless you express A(t) as a function of t. Why? Because if you use
t=5, you have a constant and can’t differentiate.

The problem is testing your capacity to do abstraction. Now let us do the differentiation (Which you are
suppose to know!!!)

We use the fact that X is a constant (relative to t).

But then

Note that this is the traditional formula for force of compound interest

Now we apply the same technique to Fabio.


Plugging into the EOV (which is why you state it first), we have

But now, and only now, we plug in t=5. We can multiply both sides by the denominator and obtain a
linear equation and solve

The problem asks to solve for Z. We however had to solve for i first. We obtain

Summary of tips:

 Use timelines
 Use unknowns for what you don’t know
 Express in variable form if you have to differentiate (This is a key reason this is a level 7
problem)
 Express EOV up front
 Know your formulas (Have you memorized chapter 2 yet!  All formulas in Chapter 2 of my
notes are very important and will appear on exams)
Sample Problems Question #1 ( c ) Sp 2015 Dr Hendel;
Solution reflects Dr Hendel’s approach
Force-Nominal-Comparison
Time 0 1 2… 14.5
Payment 100 A1(14.5)
Timeline 1: Bruce. Period=half year. Interest / period =2 %

Time 0 1 2 .. 7.25
Payment 100 0 0 … A2(7.25)
Timeline 2: Peter. Period = one year. Force of interest δ

EOV: A1(14.5) = A2(7.25)

Timeline equations:
A1(14.5) = 100(1.02)14.5= 133.26

We now combine the timeline equations with the EOV


QIT#5
Solutions ( c ) Spring 2015, Dr Hendel
Solutions reflect Dr Hendel’s approach
Time/Dollar Weighted
This solution uses the 5-6 steps in my notes.

Step 1: Detailed timeline

Date 1/1 2/1 3/1 4/1 5/1 6/1 7/1 8/1 9/1 10/1 10/15 11/1 12/1 1/1
Deposit 75 10 10- 10 10 10 10- 10 10 10 -80 10- 10 60
5 25 35

Step 2: EOV at t=1


EOV: 75(1+i)12/12 +10(1+i)11/12+5(1+i)10/12+10(1+i)9/12+10(1+i)8/12+10(1+i)7/12-
15(1+i)6/12+10(1+i)5/12+10(1+i)4/12 +10(1+i)3/12-80(1+i)2.5/12-25(1+i)2/12+10(1+i)1/12=70

Step 3: Bernoullie:
Using the Bernoulli approximation we have

75(1+12/12 i) + 10 (1+ 11/12 i) + 5(1+ 10/12 i) + 10(1+9/12 i) + 10(1+ 8/12 i) + 10(1+7/12 i) - 15 (1+6/12
i) + 10(1+5/12 i) + 10(1+4/12 i) + 10(1+3/12 i) -80(1+2.5/12 i)-25(1+2/12 i)+10(1+1/12 i)=70

Step 4: Gathering like terms:


(75+10+5+10+10+10-15+10+10+10-80-25+10) + i/12(75*12+10*11+5*10+10*9+10*8+10*7-
15*6+10*5+10*4 +10*3 -80*2.5-25*2+10*1) = 60

Step 5: Linear Form: 50 + i/12 (1090) = 60

Step 6: Solve: i= 10/1090*12 = 11.01%


Sample Problems Question #2
Solution ( c ) Sp 2015 Dr Hendel;
Solution reflects Dr Hendel’s approach
Money Growth – (a/s) – Conversions – Difference of Squares

TIMELINES

t 0 4 8 12... 20 40
t 0 1 2 3… 5 10
Payment 100 100 100 … 100… 100
Accumulated X/5=A(20) X=A(40)
Value
General Timelines @ rate i per year; broken up into two timelines till 5 and 10

t 0 4 8… 16 20
t 0 1 2… 4 5
Payment 100 100 100… 100 A(5)
Timeline 2: @ i per year.

t 0 4 8… 36 20
t 0 1 2… 9 10
Payment 100 100 100… 100 A(10) =X
Timeline 3: @ i per year.

CONVERSION BOX
i is rate per year
j is rate per 4 years
(1+i)4 = 1+j
TIP: We do not need i. We only need vj. This will save you
time

EOV: A(10) = 5 A(5) = X


Timeline equations:
Combining the EOV with the timeline equations

We want X= A(10). We simply need (1+j) and j.

Comments and tips


 The problem could be worded better. It would be better to say “ Accumulated value at 20 just
before the 100 payment at 20” vs. the current text which states “the accumulated value at 20”
By adding the underlined phrase the problem statement would provide greater clarity
 Many students try to use one timeline, timeline 1. By using 3 timelines, I have a leisurely pace
and avoid possible errors.
 Notice the use of the difference of squares: (1-x2) = (1+x)/(1-x) or (x2-1)/(x-1)=x+1. This shocks
students but it is common on many exams. Quotients of annuity symbols with indexes 2n and n
have this property
 Notice how we didn’t have to compute i. We wanted X and for that j was sufficient.
 Finally, notice how this problem is naturally formulated in terms of s rather than a. You have to
be prepared to use both approaches.
QIT#6 Solution reflects Dr Hendel’s approach ( c ) Dr
Hendel Sp 2015
Annuity-Perpetuity-Increasing-Algebra
TIMELINE First we make a timeline
T 0 1 2 3 4 … n n+1 n+2 n+3
Payment 77.1 0 1 2 3 … n-1 n n N
Cut off Cutoff
here here

EOV
Next we observe several possible EOV based on the cut off points

How do you decide which EOV to use? Or, perhaps both of them are OK?

The basic idea is to take the EOV where the manipulations are most straightforward

After studying the two EOV we can see that EOV with cutoff at $n$ allows cancellation. We have

TV LINE
We can solve this last equation with a TV calculator line as shown below

N I PV PMT FV
CPT 10.5 -77.1 x 10.5\% -1 0
= 8.0955
N=19
FM – M03#45 Annuities Inflation Refinancing
( c ) 3/24/2015 Dr Hendel – Solution reflects Dr Hendel’s approach to these problems

This problem requires 6 timelines and 5 equations of value

TIMELINE 1
0----100 (t=1)----100(t=2) ----…

An exchange is made at t = 5. So we need the the outstanding balance (OB) at t=5. This
outstanding balance (OB) is the present value at t=5 of all future payments!

0----100(t=1/old t =6) -----100 (t=2/old t=7) ---- 100(t-3/old t=8)…

To envision all timelines I use a trick of absolute dates and relative dates. The difference
between the absolute and relative dates on any timeline is constant. Here is an outline for this
problem

Timeline 2000 2001 2005 2006 2015 2016 2029 2030


I 0 1 5 6 15 16 29 30
II 0 1 10 11 24 25
VI 0 1 14 15

TIMELINE II
Remember the trick of reinvestment: We are standing at 2005 so that is 0.

0----------X (t=1)-------------1.08X (t=2) -------------1.082X (t=3) …. 1.0824 X (t=24)

This timeline is equivalent to TIMELINE III evaluated at the modified rate of i’ where 1/(1+i’)
= 1.08/1.08=1  i’=0%.
TIMELINE III
0----------------X(t=1)--------------X(t=2)-------------X(t=3)…..X(t=24)

We now have to find the outstanding balance of TIMELINE II at t=10. We also need the
payments at t=10 and t=11. For this we use the pattern technique.

 Payment at t=1, X (1.08)0


 Payment at t=2, X (1.08)1
 Payment at t=3, X (1.08)2 Pattern of X(1.08)t-1 at time t.
 Payment at t=11 X(1.08)10
 Payment at t=12 X(1.08)11

Notice that we use Timeline II to compute the outstanding balance. If we used TIMELINE III
to compute the outstanding balance we would use a level payment of X which is incorrect

 Outstanding balances must be computed using the original timeline (II)


 However the level timeline(III) is a fiction that allows us to compute PVs

TIMELINE IV
t=0-------------t=1, X(1.08)10----------t=2, X(1.08)11….t=15, X(1.08)24

We still use the same i’. Why? Because inflation and interest rate are the same so modified rate
is same.

We now convert TIMELINE IV to TIMELINE V


t=0-------------t=1, X(1.08)10----------t=2, X(1.08)10….t=15, X(1.08)10

TIMELINE VI
Remember: We are now standing in 2015. The 1619.19 is exchanged for a perpetuity. We have
the following timeline

1619.19-----------Y(t=1)-------------Y(t=2)…
QIT#93
Solutions ( c ) Dr Hendel, Spring 2015
Solutions Reflect Dr Hendel’s Approach
Annuity – Increasing Yearly/Level Monthly

TIMELINE #1
0—2000(t=1)--2000(t=2)---2000(t=12)---2000(1.02)(t=13)….2000(1.02)(t=24)----2000(1.02)24(t=275-300)

i=1/2% per period.

Tip: Convert immediately: So 300 periods and ½% per period

Tip: Do you see pattern

2000(1.02)0 first year (1-0=1)


2000(1.02)1 2nd year (2-1=1)
2000(1.02)2 3rd year (3-2=1)

So year minus exponent equal 1 2000(1.02)24 for 25th year

TIMELINE #2

Fundamental technique: The first 12 payments of 2000 have a value of at t=0. Similarly the next 12
payments of 2000(1.02) have a present value at t=12 of

We can now factor out of TIMELINE #2 the factor of

We have left TIMELINE #3


1 (t=0)-------------1.02 (t=1)--------------------1.022(t=2)--------------------1.023(t=3)-------…-------1.0224(t=24)

Interest j with 1+j = 1.00512= 1.061678%

Timeline #3 is equal to Timeline #4 at the modified interest rate 1/(1+i’) = 1.02/(1.061678)i’=1.040861

Tip: In computing i’ remember to use j not i. Why? Because i is the rate per month while j is the rate per year.

TIMELINE 4
1(t=0)-----------------1(t=1)---------------1(t=2)----------------------------------------------------1(t=24) @ i’=1.040861

We have left to compute


N00#34 Solution Dr. Hendel ( c ) Mar 15 2015, Lecture I – Solutions reflect Dr Hendel’s approach

The way you recognize refinancing problems is by a recalculation of PVs at a time different than 0. The
recalculation may involve different i,n. It may also involve conversions.

This problem has 3 timelines and 5 equations. So it affords us an opportunity to review how to approach
solving several equations in several variables

Timeline 1:

150000--------1-----------2-----------3------------i= 2%/quarter-------12----13----14---------n=4x10=40 periods

Notice how I converted immediately. Neither year nor the nominal 8% interest rate are mentioned. This
is the best way to avoid accidental errors.

Timeline 2:

Notice the problem. Timeline 2 begins at the old t=12 but is a new timeline so t=0. The best way to avoid
confusion is to use two sets of labels. I have found this very helpful.

OLBI12-------------R2----------R2----------------------------i=1.5%/quarter-------------------------40-12=28 periods
12-------------------13----------14---------------------20-------------------------------------------------------40
0---------------------1------------2----------------------8------------------------------------------------------28

Notice the difference of the two labels is a constant 12 = 12-0 = 13-1=14-2=40-28.

We will need OLBI12

The symbol refers to what is already in memory. (Recall we entered 2% when calculating R. So I and
PMT are retained. Also note that when computing the OLB at t =12 we use N=28 (40-12) since we use a
prospective approach, with 28 periods left.

These 2 equations are the essense of refinancing, computing PV at two different 0 points. It is not
conceptually difficult. But you have to have a system to avoid error.

Timeline 3:

First, the new rate is 1.75%.

Second, we have an extra payment of X at t=20 of first timeline, which is t=8 of second timeline. How do
we deal with this. We have two issues.
 Is it OLB120 or OLBII8. One of the tricks of refinancing is forgetting previous timelines. In this case
we are on timeline II, so we have forgotten timeline I. So we use OLBII8
 How do we deal with X. To answer this recall the meaning of OLBII8: It is what is owed at time 8.
If you like it is the new loan at time 8. If I owe OLBII8 and I immediately pay back X at the time of
the loan then my loan is really OLBII8 – X. This is the fundamental method of dealing with paying
off. Simply subtract from the OLB.

We need OLBII8.

Solution Time:

We have 3 EOVs. So we have 3 equations. But we have 5 unknowns: R1, R2, X, OLBI12, OLBII8. We have
two more equations for OLBI12, OLBII8. But even then we have five equations in 5 unknowns. How do you
solve them.

Trick for solving 5 equations in 5 unknowns: If some equations have one unknown solve them first. Then
you have 4 equations in 4 unknowns. Repeat the process (find the equation with one unknown). This has
been done throughout the document. We could have waited to the end to solve OR we could solve as
each equation comes up. That is a matter of taste. But an important principle of solving many equations
in many unknowns is when each equation gives one more unknown.

Summary: Go back through the document. Make sure you understand the basic characteristic of
refinancing, a recalculation at a different 0 point.
SOLUTION M00#24 ( c ) 2015 Dr Hendel, AMORTIZATION – REFINANCING Reflects my approach to these problems

1st Loan ;Original Loan

12000-(0)----750 (1) ----750 (2)-------3%/quarter Payments per 6 months @ j% Timeline


1.032 = 1+j j=6.09% Conversion

Refinanced loan; 2nd loan

New 0 L2----t=9 R (1)------R (every month)--------------R (2)------------R (3)-----3/4%------- R(30) Timeline 2

Notice how the two loan timelines are straightforward. The hard part of the problem? The connection between the two
loans. Here are some of the issues

 The last payment is the 8th payment


 The first payment of the new loan is at the time of the 9th payment of the old loan
 The original loan is in half years while the refinanced loan is in months
 The first loan is at 6.09% per half year; the 2nd load is at ¾% per month
 What does it mean that the refinancing happens 3 months before the 1st payment

Before proceeding further make sure you understand why you are confused by the above
To solve this dilemma you need a 3rd timeline for the 6 months between the 8th and 9th payment

t=4 years 4 years 1 month 4 years 2 months 4 years 3 months 4 years 4 months 4 years 5 months 4 years 6 months
th
8 payment 0 point of 1st payment
1st loan new loan (0 new loan
point is one
period
before 1st
payment-
deep)
Interest rate 0.9902% 0.9902% 0.9902% ¾% Why? ¾% ¾%
(1+j)6 Because loan
=1.0609 is refinanced
3 months
before 9th
payment is
due
OLB8I 1.0099 OLB8I 1.00992 1.00993 1.00993
1.00993
OLB8I OLB8I 1.0075 OLB8I
1.00752
OLB8I=L2
To understand what is going on I made a timeline with 4 rows of labels. Notice how Chapter 2 is the hard part of the
problem not amortization as you might expect. Here are the calculations

Calculating R is now straightforward


Solution M00#26, Dr Hendel, ( c ) Mar 14 2015, Lec J
Loans – Amortization Table - Examples
L = 19,800

Let us look at the first row of the amortization table

Notice how I left the numbers in expression form rather than simplifying to one number.
The purpose of doing this is to facilitate seeing patterns. Also notice how I don’t care about R which of
course equals I + P. Now let us look at row 2

We do not yet see all patterns. The rule of thumb is to do 3-4 examples. If you don’t see anything by
then, give up. So let us do row 3.

I now see patterns. Let us focus on all 3 patterns

At t = 16, Bank X sells to Bank Y all i) future interest and ii) principal payments at a new interest rate.
The reason that the sales price is not OLB16 is because the interest has changed.

The new interest rate is j with (1+j)6 = 1+ 14%/2, o r j = 1.1340%. Notice the two conversions: You have
to convert both the nominal and the compound rates.

1) The current value at t =16 of principal is a level stream of 20 payments (from t=17 to t=36) of
550.

The current value at t=16 of interest is current value of 198-16*5.5, 198-17*5.5,…,198-35*5.5

2) So there is a stream of 20 payments of 198


We have to subtract the PV of 16 x 5.5, 17 x 5.5, 18 x 5.5, …, 35 x 5.5. This almost looks like an increasing
annuity. In fact it seems to be the difference of 1,2,3,…,35 (times 5.5) minus 1,2,3…,15 (times 5.5).
However the difference of 1,2,3,…,35 minus 1,2,3…15 is the value of 16,17,18,…,35, 16 periods from
now. So we need a discount factor, actually an accumulation factor.

3)

Adding (1) and (2) but subtracting (3), we get 9792.39 + 3525.26 - 2460.38 = 10857.27.

The published SOA solutions noticed a further pattern. They noticed that 198 = 36 x 5.5. So

But then I17 – I35 is simply 20*5.5 to 1*5.5. So they subtract 5.5 (Da)20

The reason I did it without this extra pattern is in order to emphasize that you can solve a problem
without noticing every pattern. You do have to notice some basic patterns; but once you notice the
minimum patterns you can solve. You may, as in this example, have to combine annuities and use
discount or accumulation factors.
M03#49 Dr Hendel ( c ) Sp 2015
Solutions reflect Dr Hendel’s approach
This problem requires pattern recognition. That is the hard part
because you can’t use the formulas till you find the pattern.
The only way to find patterns is to do examples and that takes
time which you have to spend
TIP: Use variables not numbers since it helps you see the
pattern.
T R I P OLB
0 1000 = L
1 10% L 10% L 0 1000
2 10% L 10% L 0 1000

10 10% L 10% L 0 1000= L

11 15% L 10% L 5% L L -5%L=95% L


12 15% 95%L 10% 95%L 5% 95% L (95%)2 L
13 15% (95%)2 L 10% (95%)2 L 5% (95%)2 L (95%)3 L

20 15% (95%)9 L 10% (95%)9 L 5% (95%)9 L (95%)10 L

N I PV PMT FV
10 10 -(95%)10 CPT=97.44 0
1000
Reinvestment N05,#16, ( c ) Dr Hendel Spring 2015; Solutions reflect Dr Hendel’s approach

N05#16 Three key ideas in revinestment problems


 Separate timeline and EOV for each investment (each bank)
 Summary timeline with A(0), A(n) and A(n)=A(0)(1+i)n; chapter 2 problem
 Use high level symbols (e.g. A(n) with subscripts) to describe timelines

TIMELINE 1
925 45 45 … 1045
0 1 2 … 10 x 2

TIMELINE 2
45 45 …
0 1 2 … 20

Note the subtlety that the A(20) value is stated 3 ways

 Using high level A symbol


 Using actuarial notation (s)
 Using calculator (1844.80)

These 3 steps need not be done simultaneously. You can simply state A(20) and later identify its value

SUMMARY TIMELINE
0 1 2 20
TLine 1 925 1000
TLine 2 1272.59
Summary 925 2272.59
QIT#7 ( c ) Dr Hendel Sp 2015; Solutions Illustrate Dr
Hendel’s Approach
Reinvestment – Decreasing - Examples
TIMELINE
Fund X Timeline 6%
1
T 0 1 2 3 4… 8 9 10
Deposits 1000 1060 954 848 …
900 800 700 … 100 0

Fund Y Timeline 9%
2
Deposits 160 154 148

Equivalent Timeline 166 166 166 166 166


to Fund Y 3
Minus 6x1 6x2 6x3 6x4 6 x 10
Timeline 1: The 1000 accumulates 6% x 1000 = 60 so we have 1060 at time t=1. We withdraw the 60 and
an additional 100,
so we have left 900. The900 at t=1 accumulates to 6% x 900 = 54 at time t=2. We withdraw the 54 and
an additional 100
Notice the important of using examples to see the pattern of cash flows.

Timeline 2: This timeline has the interest and principle withdrawn from timeline 1

Timeline 3: This is a standard trick for decreasing cashflow. (1) First raise the first amount by the
constant decrease.
So the 160 becomes 166. Make the 166 a level annuity and the decreased amount becomes an increasing
annuity times 6

EOV

TV LINE
N I PV PMT FV
10 9 CPT=1065.33 -166 0
BGN
Retain Retain CPT=2.7711 -1 10 Brovender
Divide .09=30.79
X 6 =184.74
Subtract 880.59
10
X 1.09 2084.67

NOTE: Why did I use PV and then multiply by 1.0910? Couldn’t I have just used future value?
Answer: By using PV, I can use Brovender’s trick simplifying calculations.
QIT#9 ( c ) Sp 2015 Dr Hendel, Solution Reflects Dr
Hendel’s Approach
Amotization – Examples – 13 Basic Formulae – English
Conventions

English-Algebraic Derivations
Text Of Problem with Key Phrases Underlined

A 20-year loan of 1000 is repaid with payments at the end of each year.
Each of the first ten payments equals 150% of the amount of interest due. Each of the
last ten payments is X. The lender charges interest at an annual effective rate of 10%.
Calculate X.
Figure 1: Text of QIT#9. Problem is ( c ) SOA and reprinted from their website with permission.

We present below the English-Algebraic correspondence

English Algebra
20 year loan N=20
1000 L=1000
Payments equals 150% of amount of interest do Rt = 150% It, t=1,2,3,…
Last 10 payments is X Rt = X, t=11,12,13,…
Interest at an annual effective rate of 10% i=10%

Table 1: Correspondence of English phrases and algebraic equivalents.

AMORTIZATION TABLES
We now look at examples and explore patterns using a standard table and formulae

T R I P OLB
0 1000 = L
1 150% I 10% L 150% x 10% L L-5% L =95% L
– 10% L = 5%
L
2 150% I 10% 95% L 150% x 10% (95%)2 L Exponent 2 = t=2
95% L – 10%
95% L = 5%
95% L
3 150% I 10%(95%)2 L 5% (95%)2 L (95%)3 L Exponent 3= t=3
… … … … … …
10
(95%)10 L Justified
by Pattern
of
examples
11 X
12 X
13 …
14-19 …
20 X
Table 2: We break the problem into two amortization tables. The first 10 rows correspondes to
one amortization table.
The 2nd 10 rows corresponds to the 2nd amortization table. Notice how we have to use
examples and patterns in the
first 10 rows in order to see the pattern. The purpose of this is to derive a formula for the OLB10

EOV: There are two EOV; one for the first 10 rows and one for the 2 nd
10 rows.

TV LINE
N I PV PMT FV
10 10 -598.74 CPT=97.44 0
QIT#10 / M03#42 ( c ) Sp 2015, Dr Hendel, Solutions
Reflect Dr Hendel’s Approach
Bonds – 13 Formulae – Plug in – English Conventions
Text of Problem
A 10,000 par value 10-year bond with 8% annual coupons is bought at a premium
to yield an annual effective rate of 6%.
Calculate the interest portion of the 7th coupon.
Figure 1: Text of QIT#10. Text is copyright SOA and reprinted with permission

English Algebra correspondence


English Algebra
10000 F = 10000
Par value C=F
10 year bond N=10
8% annual coupons r=8%
Yield …6% i=6%
Interest portion of 7th coupon I7
Table 1: Correspondence between underlined English phrases in Figure 1 and algebraic equations

This is an easy problem: Formula plug in

Although not necessary for the problem we explain the terms “bought at a premium” and related terms

These are English conventions

At Issue: Bond Amount of Amount of


Bond bought at premium/discount in premium/discount in
bought at premium bond1 t-th coupon
or discount
Premium P>C |P–C| Pt
Discount C>P |P–C| -Pt
NOTES: (1) Absolute amount of premium in t-th coupon is called write down; Absolute amount
of discount in t-th coupon is called write-up.

EOV: Approach #1:


Fr = I7 + P7
Fr = 8% x 10000 = 800
P7 = (Cg – Fr) vn+1-7 = (Cg-Fr) v4 = C(i-r) v4 = 10000 (8%-6%) 1.06-4 = 158.42
I7 = Fr – P7 = 800 – 158.42 = 641.58

EOV: Approach #2
I7 = I OLB6 = 6% OLB6

The OLB6 may be computed with the following TV line

TV LINE
N I PV PMT FV
4 6% CPT=10693.02 8% x 10000 = 800 10000
Sample Problems, #74 Bond Comparison
( c ) Dr Hendel Spring 2015;
Solutions reflect Dr Hendel’s approach
Bonds – Pricing - Comparison
Note: This problem can be done using either the Fr or C(g-i) formulae.

TIPS:
 Do all conversions up front
 Write EOV up front

Conversions
Phrase in problem Conversion
10 years n = 20
Coupon rate of i+4% i/2 + 2%
Coupon rate of i-4% i/2 – 2%
Yield …nominal rate of i convertible i/2

Equations for prices

Tips for solving equations:

 Count equations and unknowns: (2 equations; 1 unknown)


 Subtract or divide

We can solve for i using the calculator TV Line

N I PV PMT FV
20 CPT =4.2 -5341.12/400 1 0
So i/2 = 4.2%  i = 8.4%
QIT#55
Solutions © Sp 2015, Dr Hendel
Solutions Reflect Dr Hendel’s Approach
Bonds - Callable

Tip: As I indicated in class, these problems can be confusing since there are so many things to
check. I present in this solution an organizational technique. The organization is based on
roman numerals I,II,III and upper case letters A,B,C.

IA: Calculate initial price of bond.


Remember, when the bond is bought, the buyer does not know when the seller will call it. So
the price of the bond is based on t=0. The investor wants a minimum yield (over all prices and
redemption values) of 3% so that is what we use. But we have to check the yield at other prices.
This is done in B.

N I PV PMT FV
40 3 CPT=1261.80 40 1100

1B: Check yield of part (A) with other possible call dates and call values.
Remember, the coupons remain the same. Peeking through the problem we see that we must
check 2 x 15 = 30 through 39. Since the premium will be the same on these dates, 1200, we
only have to calculate 1 or 2 points. (Thoughout the solution I am checking 2 points but
indicating which one I didn’t have to do if applicable)

N I PV PMT FV
40 3 CPT=-1261.80 40 1100

30 CPT =3.07 Retain Retain 1200


39 CPT =3.10 Retain Retain 1200

NOTE: Since the bond is (bought) at a premium C=1200 <1261.80=P, the earliest yield, t = 30,
is the lowest, lower than 39. But a safe approach is to always check two points.

IIA: Calculate the bond price using the alternate call premium.
N I PV PMT FV
30 3 CPT=1278.40 40 1200
IIB: Check yield of part (A) with other possible call dates and call values.
Peeking through the problem we see that we must check N=39, C=1200 and N=40, C=1100.
Since the bond is bought at a premium we need not check N=39, C=1200 but do so anyway

N I PV PMT FV
30 3 CPT=-1278.40 40 1200

39 CPT =3.04 Retain Retain Retain


40 CPT =2.94 Retain Retain 1100

IIIA: Calculate price with N=39, C=1200 and I=3.


N I PV PMT FV
39 3 CPT = - 40 1200
1291.23
IIIB: Check yield of part (A) with other possible call dates and call values.
Peeking through problem we must check N=30, C=1200 and N=40,C=1100. Note: Since bond
is (bought) at premium (1291.23=P>1200=C) we need not check N=30 but do so anyway.

N I PV PMT FV
39 3 CPT = -1291.23 40 1200

30 CPT =2.95 Retain Retain Retain


40 CPT=2.90 Retain Retain 1100

Answer to question: Look over all the (B) parts. Remember, the buyer does not know which
call date will be used. The buyer wants at least a 3% yield. Only one of the B tables gives a 3%
yield for all redemption values and prices. Hence we use table IA: The maximum, indeed only
price, is 1261.80. Why? By Table IB, the buyer may get either 3%, 3.07% or 3.10% depending
on when the seller calls. So indeed the buyer gets at least 3% as required.
Quick Method for Solving Interest Swap Problems , Problem 23 Sample Derivative problems
(c ) Dr Hendel Sp 2015; Solution reflects Dr Hendel’s approach

GIVEN SPOT RATES:


t 1 2 3 4 5
rt 4% 4.5% 5.5% 6.25% 7.5%
Find a level swap rate for one year interest rates
Solution
We want an equation of the following timelines

t 1 2 3 4 5
1+f0,1 1+f1,2 1+f2,3 1+f3,4 1+f4,5
Timeline I: Timeline of 1-year forward rate factors

t 1 2 3 4 5
1+i 1+i 1+i 1+i 1+i
Timeline II: Timeline of level one year swap rate factors

t 1 2 3 4 5
1 1 1 1 1
Timeline III: Timeline of level ones (Warning: This is not calculable on TV line; why?)

t 1 2 3 4 5
i i i i i
Timeline IV: Timeline of level swap rates (Warning: This is not calculable on TV line; why?)

t 0 1 2 … 5
1 0 0 0 -1
Timeline V: Deposit 1 in bank at time 0 and withdraw that 1 at time 5 (Withdraw all interest)

EOV TL =TL I II = TLIII + TLIV, TLIV = TLV


with the symbol TLx indicating Present values of Timeline x using implied forward rates.

TLIV = TLV = 1- 1/(1+r5)5 = 1-1/1.0755= 0.3034;


TLIII = TLIV/i

Note: (1+ri-1)i-1 (1+fi-1,i) = (1+ri)i  1+fi-1,i = (1+ri)i / (1+ri-1)i-1 (1+fi-1,i)/ (1+ri)I = 1/(1+ri-1)i-1
Hence TLI = 1 +1/1.04 + 1/1.0452 +1/1.0553 + 1/1.06254 = 4.5135

Combining all together with EOV we have 0.3034/i+0.3034 =4.5135 i= 7.21%

You might also like